Chapter 40 - Mechanisms of Endocrine Control, Patho Chapter 31-33, Patho chapter 31, patho phys ch. 31 and 33, Patho Ch 38 Disorders of Special Sensory Function, CH 38 PATHOPHYSIOLOGY, Chap 19 Disorders of Visual Function, Chapter 38: Disorders of Sp...

Réussis tes devoirs et examens dès maintenant avec Quizwiz!

A client has developed a tumor of the posterior pituitary gland. The client is at risk for problems with secretions of:

Antidiuretic hormone (ADH) and oxytocin

A client has developed a tumor of the posterior pituitary gland. The client is at risk for problems with secretions of:

Antidiuretic hormone (ADH) and oxytocin Explanation: The posterior pituitary secretes ADH and oxytocin/vasopressin, while the anterior pituitary secretes the hormones listed in the other choices. Page 762

A client asks the nurse what causes the secretion of growth hormone (GH) and adrenocorticotrophic hormone (ACTH) to fluctuate. The best response by the nurse would be:

Sleep-wake cycles

8. Which medical term is a synonym for ophthalmodynia? a. Ophthalmalgia b. Ophthalmectomy c. Ophthalmoplegia d. Ophthalmoscopy e. Ophthalmitis

A. Ophthalmalgia

What is the role of glucagon in initiating the breakdown of glycogen?

First messenger

Which physiologic process allows hormones to exert influence upon some cells and not others?

Hormone receptors

Which hormone will elicit a more rapid response than the others?

Norepinephrine

Conductive hearing loss can occur for a variety of reasons, including foreign bodies in the ear canal, damage to the ear drum, or disease. What disease is associated with conductive hearing loss?

Paget disease.

A male client has been diagnosed with a low sperm count. Which endocrine imbalances could contribute to this condition? Select all that apply.

Lack of follicle-stimulating hormone (FSH) Insufficient androgens

Which body function is regulated by several hormones?

Lipolysis

33. Ossification in the bony labyrinth that leads to progressive deafness is called: a. Tinnitus b. Otosclerosis c. Tympanosclerosis d. Tympanorrhexis e. Mastoiditis

B. Otosclerosis

The nurse is caring for a client whose hearing is impaired due to impacted earwax. The nurse understands that the associated deafness is due to which of the following?

Conduction disorder.

What are the hallmark signs of diabetes mellitus?

Polyuria, polydipsia, and polyphagia

What are the hallmark signs of diabetes mellitus? a) Polyuria, polydipsia, and pheochromocytoma b) Polyuria, polydipsia, and polyphagia c) Polycythemia, polydipsia, and pheochromocytoma d) Polyuria, polyphagia, and polycythemia

Polyuria, polydipsia, and polyphagia The most commonly identified signs and symptoms of diabetes are referred to as the three polys: (1) polyuria (i.e., excessive urination), (2) polydipsia (i.e., excessive thirst), and (3) polyphagia (i.e., excessive hunger). Pheochromocytoma and polycythemia are not hallmark signs of diabetes mellitus.

Which cranial nerve is designated as the vestibulocochlear nerve?

Cranial nerve VIII Explanation: The vestibulocochlear nerve is cranial nerve VIII.

It is important to differentiate between the kinds of hearing loss so they can be appropriately treated. What is used to test between conductive and sensorineural hearing loss?

Tuning fork

It is important to differentiate between the kinds of hearing loss so they can be appropriately treated. What is used to test between conductive and sensorineural hearing loss?

Tuning fork Explanation: Tuning forks are used to differentiate conductive and sensorineural hearing loss. Audioscope, audiometer, and tone analysis do not differentiate between conductive and sensorineural hearing loss.

44. What is the snail-shaped tube in the inner ear called? a. Auricle b. Pinna c. Cochlea d. Auditory meatus e. Oval window

C. Cochlea

41. Astigmatism can be described as: a. Esotropia b. Exotropia c. Defective curvature of the cornea and lens d. Localized infection of the eye e. A tumor pressing on the nerve.

C. Defective curvature of the cornea and lens

13. Which term means double vision? a. Myopia b. Hyperopia c. Diplopia d. Nearsightedness e. Farsightedness

C. Diplopia

Which cranial nerve is designated as the vestibulocochlear nerve?

Cranial nerve VIII.

Dilutional hyponatremia occurs in which disorder?

Syndrome of inappropriate antidiuretic hormone secretion (SIADH)

Accidental removal of one or both parathyroid glands can occur during a thyroidectomy. Which of the following is used to treat tetany?

Calcium gluconate

A patient has shingles. What is the best time frame for the initiation of oral and intravenous antiviral drugs after the appearance of a rash to reduce the incidence of ocular complication?

3 days

A client comes to a scheduled appointment in the endocrine clinic. The primary care physician referred the client, suspecting acromegaly. Knowing the usual testing involved, the nurse should educate the client about which lab/diagnostic procedure?

A suppression test

Which manifestation indicates a client is at risk for developing diabetes mellitus?

2 hour oral GTT 175 mg/dL (9.7 mmol/L)

The actions of parathyroid hormone (PTH) are increased in the presence of which vitamin?

D

Which is a clinical manifestation of diabetes insipidus?

Excessive thirst

59. horny tissue; har; cornea a. Acous/o k. kerat/o b. Blephar/o l. myring/o c. Chromat/o m. ophthalm/o d. Conjunctiv/o n. ot/o e. Choroid/o o. salping/o f. Core/o p. -metry g. Corne/o q. -opsia h. Dacry/o r. -ptosis i. dipl/o s. -rrhea j. irid/o t. -tropia

k. kerat/o

The diagnosis of type 1 diabetes would be confirmed by which principle?

Insulin is not available for use by the body.

64. turning a. Acous/o k. kerat/o b. Blephar/o l. myring/o c. Chromat/o m. ophthalm/o d. Conjunctiv/o n. ot/o e. Choroid/o o. salping/o f. Core/o p. -metry g. Corne/o q. -opsia h. Dacry/o r. -ptosis i. dipl/o s. -rrhea j. irid/o t. -tropia

t. -tropia

82. sensation of moving around in space or a feeling of spinning or dizziness a. achromatopsia k. macular degeneration b. anacusis l. meniere disease c. astigmatism m. myringoplasty d. audiometry n. myringotomy e. cataract o. otoscopy f. diabetic retinopathy p. photophobia g. exotropia q. rinne test h. glaucoma r. strabismus i. hordeolum s. tinnitus j. iridoplegia t. vertigo

t. vertigo

47. Hyperopia is a condition also known as: a. Nearsightedness b. Farsightedness c. Strabismus d. Astigmatism e. Glaucoma

B. Farsightedness

Which assessment would a nurse perform on a client with Cushing's syndrome who is at high risk of developing a peptic ulcer?

Observe stool color.

Which type of receptor is mediated through vision in dim light?

Photoreceptors

Presbycusis is degenerative hearing loss associated with aging. What is the first symptom of this disorder?

Reduction in ability to understand speech

Patients with hyperthyroidism are characteristically:

Sensitive to heat

Which type of characteristic do thyroid and steroid hormones exert on target cells by way of nuclear receptors?

The ability to cross the cell membrane of target cells

The nurse is preparing a client for a surgical procedure to create an endolymphatic shunt. The nurse understands that this procedure aims to relieve which of the following symptoms?

Vertigo

Age-related cataracts are characterized by what?

Visual distortion

A patient has been diagnosed with bacterial conjunctivitis. This type of infection is usually characterized by large amounts of what color drainage?

Yellow-green.

A nurse who works in the office of an endocrinologist is orienting a new staff member. Which teaching point should the nurse include in the orientation?

"A single hormone can act on not only one process or organ but often on several different locations or processes."

The nurse is performing a health history for a seventy-two-year-old client. When reviewing the client's medications, the client states, "I take a baby aspirin every day, but I'm worried. I heard that aspirin can cause hearing problems!" Which of the following is an appropriate response by the nurse?

"A baby aspirin is a low dose that is considered to be safe."

The nurse is performing a health history for a 72-year-old client. When reviewing the client's medications, the client states, "I take a baby aspirin every day, but I'm worried. I heard that aspirin can cause hearing problems!" What is an appropriate response by the nurse?

"A baby aspirin is a low dose that is considered to be safe." Explanation: A frequently overlooked ototoxic agent is aspirin. Little is known about what level of dosage causes ototoxicity, but it is generally believed that 81 mg of aspirin on a daily basis is safe.

A nurse who works in the office of an endocrinologist is orienting a new staff member. Which teaching point should the nurse include in the orientation? "The regulation of homeostasis requires that hormones be absent from the body when their effect is not needed." "A single hormone can act on not only one process or organ but often on several different locations or processes." "It's uncommon for production of hormones to be far removed from the tissue where they ultimately exert their effect." "Sometimes hormones act locally on the area where they were produced, as in the case of endocrine actions."

"A single hormone can act on not only one process or organ but often on several different locations or processes."

A nurse who works in the office of an endocrinologist is orienting a new staff member. Which teaching point should the nurse include in the orientation?

"A single hormone can act on not only one process or organ but often on several different locations or processes." Explanation: A single hormone can exert various effects in different tissues, or conversely, a single function can be regulated by several different hormones. Hormones act both distantly from their source and more locally, as in the case of autocrine and paracrine actions. Hormones are normally present at all times.

A nurse is teaching a client with adrenal insufficiency about corticosteroids. Which statement by the client indicates a need for additional teaching?

"I may stop taking this medication when I feel better."

A client diagnosed with type 2 diabetes has been instructed about managing his condition with diet. The nurse determines further teaching is necessary when the client states:

"I must avoid all candies and cookies, but can eat unlimited amounts of pasta and breads."

The nurse is teaching a teenaged client strategies to avoid recurrent ear infections. The nurse understands that further teaching is required when the client states which of the following?

"I will dry the inside of my ears thoroughly with cotton-tipped applicators after swimming."

A 2-year-old child has been diagnosed with uncomplicated acute otitis media by a nurse practitioner. When explaining the plan of care, which description is most appropriate?

"Let's keep a close eye on his condition for a couple of days before starting antibiotics."

A 2-year-old child has been diagnosed with uncomplicated acute otitis media by a nurse practitioner. When explaining the plan of care, which description is most appropriate?

"Let's keep a close eye on his condition for a couple of days before starting antibiotics." Explanation: Observation without antimicrobial agents is an option in a child over 6 months of age with uncomplicated AOM. These approaches involve waiting for 24 hours to 72 hours (greater than 6 months of age) to see if symptoms improve before institution of antibiotic therapy. Antiviral medications and decongestants are ineffective and rinsing of the ears is not curative.

The nurse has just completed teaching a client, newly diagnosed with type 1 diabetes, about the treatment options. Which response by the client leads the nurse to conclude that addidtional teaching is needed?

"So I can stop my insulin if I start an exercise program."

A 69-year-old patient comes to the clinic for a routine checkup. Upon examination the nurse practitioner informs the patient that she has cataracts. The patient then tells the nurse that she already knew that and her physician told her that she could use bifocals and that would take care of the problem. What would be the best response by the nurse practitioner?

"Surgery is the only effective treatment for cataracts."

A 70 year-old woman with a diagnosis of benign paroxysmal positional vertigo (BPPV) is receiving teaching from her physician about her diagnosis. The client is eager to avoid future episodes of vertigo and has asked the physician what she can do to prevent future episodes. How can the physician best respond?

"There are some exercises that I'll teach you to help reorient your inner ear and prevent vertigo."

A young child develops type 1A diabetes. The parents ask, "They tell us this is genetic. Does that mean our other children will get diabetes?" The best response by the health care provider would be: a) "Probably not since genetically your other children have a different cellular makeup, they just might not become diabetic." b) "If you put all your children on a low-carbohydrate diet, maybe they won't get diabetes." c) "We don't know what causes diabetes, so we will just have to wait and see." d) "This autoimmune disorder causes destruction of the beta cells, placing your children at high risk of developing diabetes."

"This autoimmune disorder causes destruction of the beta cells, placing your children at high risk of developing diabetes." Type 1 diabetes is subdivided into two types: type 1A, immune-mediated diabetes, and type 1B, idiopathic diabetes. Type 1A diabetes is characterized by autoimmune destruction of beta cells. The other choices are not absolutely correct. The fact that type 1 diabetes is thought to result from an interaction between genetic and environmental factors led to research into methods directed at prevention and early control of the disease. These methods include the identification of genetically susceptible persons and early intervention in newly diagnosed persons with type 1 diabetes

A client reports occasional ringing in the ears that worsens toward the end of the day. The most important question for the nurse to ask would be:

"What prescription medication and over-the-counter medication do you take?" Explanation: Medications and stimulants, such as aspirin, caffeine, and nicotine, can cause tinnitus. The client should be questioned to determine if this is a potential cause. Impacted cerumen is a benign cause of tinnitus, which resolves after the earwax is removed. Respiratory infections may cause fluid in the ears and decreased hearing.

A new mother brings her infant to the clinic reporting that the child is not sleeping or eating much. Upon assessment, the health care provider notes that the infant's ear canal is reddened with a bulging tympanic membrane. Which other data collected would lead to the diagnosis of acute otitis media (AOM)? Select all that apply.

"Yes, he has been pulling at his ear." "He's been very irritable and fussy the past couple of days." Explanation: AOM is characterized by an acute onset of otalgia (ear pain), fever, and hearing loss. Younger children often have nonspecific signs and symptoms that manifest as ear tugging, irritability, nighttime awakening, and poor feeding. Key diagnostic criteria include ear pain that interferes with activity or sleep, tympanic membrane erythema (redness), and middle ear effusion. A child with otitis media with effusion (OME) may develop delayed speech and language skills.

During an assessment of a client's functional health pattern, which question by the nurse directly addresses the client's thyroid function? -"Do you have to get up at night to empty your bladder?" -"Have you experienced any headaches or sinus problems?" -"Do you experience fatigue even if you have slept a long time?" -"Can you describe the amount of stress in your life?"

-"Do you experience fatigue even if you have slept a long time?" With the diagnosis of hypothyroidism, extreme fatigue makes it difficult for the person to complete a full day's work or participate in usual activities.

A patient is ordered desmopressin (DDAVP) for the treatment of diabetes insipidus. What therapeutic response does the nurse anticipate the patient will experience? -A decrease in blood pressure -A decrease in blood glucose levels -A decrease in urine output -A decrease in appetite

-A decrease in urine output Desmopressin (DDAVP), a synthetic vasopressin without the vascular effects of natural ADH, is particularly valuable because it has a longer duration of action and fewer adverse effects than other preparations previously used to treat the disease. DDAVP and lypressin (Diapid) reduce urine output to 2 to 3 L/24 hours. It is administered intranasally; the patient sprays the solution into the nose through a flexible calibrated plastic tube. One or two administrations daily (i.e., every 12 to 24 hours) usually control the symptoms (Papadakis, McPhee, & Rabow, 2013). Vasopressin causes vasoconstriction; thus, it must be used cautiously in patients with coronary artery disease.

What interventions can the nurse encourage the client with diabetes insipidus to do in order to control thirst and compensate for urine loss? -Come to the clinic for IV fluid therapy daily. -Limit the fluid intake at night. -Consume adequate amounts of fluid. -Weigh daily.

-Consume adequate amounts of fluid. The nurse teaches the client to consume sufficient fluid to control thirst and to compensate for urine loss. The client will not be required to come in daily for IV fluid therapy. The client should not limit fluid intake at night if thirst is present. Weighing daily will not control thirst or compensate for urine loss.

Antithyroid medications are contraindicated in late pregnancy due to the fact that which of the following may occur? Select all that apply. -Fetal hypothyroidism -Fetal bradycardia -Goiter -Cretinism -Fetal tachycardia

-Fetal hypothyroidism Antithyroid medications are contraindicated in late pregnancy because the fetus may develop fetal hypothyroidism, fetal bradycardia, goiter, and cretinism.

The nurse knows to assess a patient with hyperthyroidism for the primary indicator of: -Fatigue -Weight gain -Constipation -Intolerance to heat

-Intolerance to heat With hypothyroidism, the individual is sensitive to cold because the core body temperature is usually below 98.6°F. Intolerance to heat is seen with hyperthyroidism.

A client is receiving long-term treatment with high-dose corticosteroids. Which of the following would the nurse expect the client to exhibit? -Weight loss -Pale thick skin -Moon face -Hypotension

-Moon face Clients who are receiving long-term high-dose corticosteroid therapy often develop a cushingoid appearance, manifested by facial fullness and the characteristic moon face. They also may exhibit weight gain, peripheral edema, and hypertension due to sodium and water retention. The skin is usually thin, and ruddy.

A patient taking corticosteroids for exacerbation of Crohn's disease comes to the clinic and informs the nurse that he wants to stop taking them because of the increase in acne and moon face. What can the nurse educate the patient regarding these symptoms? -The symptoms are permanent side effects of the corticosteroid therapy. -The moon face and acne will resolve when the medication is tapered off. -Those symptoms are not related to the corticosteroid therapy. -The dose of the medication must be too high and should be lowered.

-The moon face and acne will resolve when the medication is tapered off. Cushing syndrome is commonly caused by the use of corticosteroid medications and is infrequently the result of excessive corticosteroid production secondary to hyperplasia of the adrenal cortex. The patient develops a "moon-faced" appearance and may experience increased oiliness of the skin and acne. If Cushing syndrome is a result of the administration of corticosteroids, an attempt is made to reduce or taper the medication to the minimum dosage needed to treat the underlying disease process (e.g., autoimmune or allergic disease, rejection of a transplanted organ).

A client has been experiencing a decrease in serum calcium. After diagnostics, the physician proposes the calcium level fluctuation is due to altered parathyroid function. What is the typical number of parathyroid glands? -four -three -two -one

-four The parathyroid glands are four (some people have more than four) small, bean-shaped bodies, each surrounded by a capsule of connective tissue and embedded within the lateral lobes of the thyroid.

A nurse is caring for a client with hypoparathyroidism. During assessment, the nurse elicits a positive Trousseau's sign. What does the nurse observe to verify this finding? -hand flexing inward -cardiac dysrhythmia -moon face and buffalo hump -bulging forehead

-hand flexing inward The nurse observes the client for spasm of the hand (carpopedal spasm), which is evidenced by the hand flexing inward.

A client presents with a huge lower jaw, bulging forehead, large hands and feet, and frequent headaches. What could be causing this client's symptoms? -hyperpituitarism -hypopituitarism -panhypopituitarism -panhyperpituitarism

-hyperpituitarism Acromegaly (hyperpituitarism) is a condition in which growth hormone is oversecreted after the epiphyses of the long bones have sealed. A client with acromegaly has coarse features, a huge lower jaw, thick lips, a thickened tongue, a bulging forehead, a bulbous nose, and large hands and feet. When the overgrowth is from a tumor, headaches caused by pressure on the sella turcica are common.

Although not designated as endocrine glands, several organs within the body secrete hormones as part of their normal function. Which organ secretes hormones involved in increasing blood pressure and volume and maturation of red blood cells? -kidneys -cardiac atria -brain -liver

-kidneys The kidneys release renin, a hormone that initiates the production of angiotensin and aldosterone to increase blood pressure and blood volume. The kidneys also secrete erythropoietin, a substance that promotes the maturation of red blood cells.

Cardiac effects of hyperthyroidism include -decreased pulse pressure. -decreased systolic BP. -bradycardia. -palpitations.

-palpitations. Cardiac effects may include sinus tachycardia, increased pulse pressure, and palpitations. Systolic BP is elevated.

Cardiac effects of hyperthyroidism include -decreased pulse pressure. -decreased systolic blood pressure. -bradycardia. -palpitations.

-palpitations. Cardiac effects may include sinus tachycardia, increased pulse pressure, and palpitations. Systolic blood pressure is elevated.

A nurse educator is teaching a chapter on "The Function of the Endocrine System." Which hormone would not be included as one of the six hypothalamic hormones? -prolactin -corticotropin-releasing hormone -thyrotropin-releasing hormone -gonadotropin-releasing hormone

-prolactin Hypothalamic dopamine inhibits the release of prolactin from the anterior pituitary gland. Corticotropin-releasing hormone (CRH) causes the anterior pituitary gland to secrete adrenocorticotropic hormone (ACTH). Thyrotropin-releasing hormone (TRH) stimulates the release of thyroid-stimulating hormone (TSH) from the anterior pituitary gland. Gonadotropin-releasing hormone (GnRH) triggers sexual development at the onset of puberty and continues to cause the anterior pituitary gland to secrete luteinizing hormone (LH) and follicle-stimulating hormone (FSH).

When explaining about structural classifications to a group of students, the instructor discusses the peptides and proteins. The instructor talks about small hormones and hormones as large and complex as growth hormone (GH), which has approximately how many amino acids involved?

200 amino acids

When explaining about structural classifications to a group of students, the instructor discusses the peptides and proteins. They talk about small hormones and hormones as large and complex as growth hormone (GH) which has approximately how many amino acids involved?

200 amino acids. Explanation: Growth hormone is very large and complex protein which has ~ 200 amino acids. Page 754

When explaining about structural classifications to a group of students, the instructor discusses the peptides and proteins. They talk about small hormones and hormones as large and complex as growth hormone (GH) which has approximately how many amino acids involved?

200 amino acids. Explanation: Growth hormone is very large and complex protein which has ~ 200 amino acids.

A nurse is assigning beds to four new clients being admitted to the cardiac telemetry floor. Which client should she assign to the bed at the end of the hall, away from the nurses' station?

A 24-year-old client with unstable hyperthyroidism with sinus tachycardia

29. Which test measures hearing acuity at various sound frequencies? a. Audiometry b. Rinne c. Weber d. Tonometry e. Otoscopy

A. Audiometry

2. Which term is a synonym for sty? a. Hordeolum b. Exotropia c. Vertigo d. Cerumen e. Choroiditis

A. Hordeolum

The nurse is caring for a client who reports, "My ears are constantly ringing!" The nurse will evaluate the client further for the use of which of the following medications?

Acetylsalicylic acid (aspirin)

Which disorder results from excessive secretion of somatotropin (growth hormone)?

Acromegaly

A client reports sudden, acute left eye pain with blurred vision and a headache on the affected side. The client is most likely experiencing:

Acute angle-closure glaucoma.

The nurse understands that an antimicrobial drug is most likely to be prescribed for which of the following conditions?

Acute otitis media

Which of the following hormones are derivatives of cholesterol?

Aldosterone and testosterone Explanation: Steroids such as aldosterone and testosterone are a classification of hormones that are derived from cholesterol. Epinephrine and norepinephrine are both amino acids while insulin and glucagon and classified among peptides, polypeptides, proteins, and glycoproteins. Eicosanoids and retinoids consist of fatty acid compounds. Page 755

Hypoglycemia has a sudden onset with a progression of symptoms. What are the signs and symptoms of hypoglycemia?

Altered cerebral function and headache

What is ocular muscle imbalance resulting in "lazy eye" called?

Amblyopia

A client has blindness in the right eye. The nurse documenting in the electronic medical record would use which term for blindness in one eye?

Anopia Explanation: Blindness in one eye is called anopia. If half of the visual field for one eye is lost, the defect is called hemianopia; if a quarter of the field is lost, it is called quadrantanopia. Loss of the temporal or peripheral visual fields on both sides results in a narrow binocular field, commonly called tunnel vision.

A client develops fever, headache, and burning/itching in the periorbital area. After a few days, a vesicular rash appears around the eyelid margins. The health care provider will likely prescribe:

Antiviral medication for herpes zoster ophthalmicus

Otitis externa is an inflammation of the outer ear. What fungi cause otitis externa?

Aspergillus Explanation: The most common bacterial pathogens are gram-negative rods (Pseudomonas aeruginosa, Proteus spp.) and fungi (Aspergillus) that grow in the presence of excess moisture. The other answers are not fungi.

The nurse is performing a Romberg test on a client. Select the statement that best describes purpose of the test.

Assess for static vestibular function

The nurse is performing a Romberg test on a client. Select the statement that best describes purpose of the test. a) Assess for neurological weakness b) Assess for transient ischemic attacks c) Assess for static vestibular function d) Assess for conductive hearing loss

Assess for static vestibular function The Romberg test is used to demonstrate disorders of static vestibular function. It is done by having a person stand with his or her feet together and arms extended forward so that the degree of sway and arm stability can be observed. (less)

The nurse is explaining to the parent of a 5-year-old that the child has otitis media with effusion (OME), noted by otoscopic exam, following an upper respiratory infection. Unlike acute otitis media (AOM), OME does not require treatment with antibiotics because it is usually which of the following?

Asymptomatic and often self-limiting

The nurse is explaining to the parent of a 5-year-old that the child has otitis media with effusion (OME), noted by otoscopic exam, following an upper respiratory infection. Unlike acute otitis media (AOM), OME does not require treatment with antibiotics because it is usually which of the following?

Asymptomatic and often self-limiting.

A client seeks medical care when he wrecks his care because of poor eyesight. At the time of admission, his blood glucose level was 390 mg/dL. The client is diagnosed with diabetes (type 2). The ophthalmologist must perform an urgent intravitreal injection. The nurse explains this to the client by stating the doctor will: A) Just put a couple of drops in each of your eyes B) Put a needle with syringe into your eyeball and inject some medication to decrease active bleeding. C) Remove some of the vitreous from your eye by withdrawing it with a needle/syringe and then strip some of the membranes off your inner eye. D) Use a laser to try to seal off any bleeding vessels in your eyeball.

B) Put a needle with syringe into your eyeball and inject some medication to decrease active bleeding.

While on a cruise to the Caribbean, a person develops "motion sickness" with associated malaise, and nausea/vomiting. The nurse notes the client's BP is 88/52; pulse is 110; and skin moist with perspiration. The client diagnosis related to the clinical manifestations would most likely be: A) Light-headedness B) Vertigo C) Syncope D) Dizzy

B) Vertigo

To prevent the accumulation of hormones in our bodies, the hormones are constantly being metabolized and excreted. Where are adrenal and gonadal steroid hormones excreted?

BILE AND URINE

A client presents with copious amounts of yellow-green drainage, conjunctival redness, and chemosis to the right eye. A culture of the eye drainage reveals S. pneumoniae. The client most likely developed:

Bacterial conjunctivitis Explanation: Based on the presenting symptoms and the culture results, this is classic bacterial conjunctivitis. Allergic and chronic conjunctivitis usually involves both eyes.

A 26-year-old client who has recently traveled by airplane complains of pain in his right ear. The nurse suspects which condition?

Barotrauma Explanation: Considering the client's age and recent air travel, a likely diagnosis would be barotrauma. The other options are not affected by age and changes in pressure experienced during air travel.

A young child has been diagnosed with amblyopia. The parents ask the nurse when treatment should be considered. At what age should treatment of children with the potential for development of amblyopia be instituted?

Before 6 years

A client with severe head trauma sustained in a car accident is admitted to the intensive care unit. Thirty-six hours later, the client's urine output suddenly rises above 200 ml/hour, leading the nurse to suspect diabetes insipidus. Which laboratory findings support the nurse's suspicion of diabetes insipidus?

Below-normal urine osmolality level, above-normal serum osmolality level

A client tells the nurse that when she is in bed, changes position of her head, and rolls into a lateral position, she experiences brief periods of vertigo, usually lasting less than 1 minute. The client is most likely experiencing:

Benign paroxysmal positional vertigo

30. What is a feeling of spinning or dizziness called? a. Tinnitus b. Exophthalmos c. Vertigo d. Vitiligo e. Nystagmus

C. Vertigo

The pancreas is an endocrine organ that is composed of the acini and the islets of Langerhans. The islets of Langerhans have alpha, beta, and delta cells as well as the PP cell. Which cells secrete insulin? a) Alpha cells b) Delta cells c) PP cells d) Beta cells

Beta cells Each islet is composed of beta cells that secrete insulin and amylin, alpha cells that secrete glucagon, and delta cells that secrete somatostatin. In addition, at least one other type of cell, the PP cell, is present in small numbers in the islets and secrets a hormone of uncertain function called pancreatic polypeptide.

10. Which structure is part of the external ear? a. Malleus b. Stapes c. Auricle d. Eustachian tube e. Cochlea

C. Auricle

If glucagon binds to surface receptors on liver cells to send an intracellular message for glycogen breakdown, this process is known as which mechanism of action?

Cell surface receptors

Frustrated by her worsening tinnitus, a 55-year-old female client has sought care. Which of the following teaching points should the clinician provide to the client? A) "I know this can be very difficult to live with, but it normally fades over time." B) "I will prescribe some medication that will probably help quite well." C) "This might be a sign of a more serious neurologic problem that we will assess for." D) "Initially, there are some changes in your diet that you should implement."

D) "Initially, there are some changes in your diet that you should implement."

A client develops fever, headache, and burning/itching in the periorbital area. After a few days, a vesicular rash appears around the eyelid margins. The health care provider will likely prescribe: A) Topical antimicrobial for infection caused by overuse of contact lens B) Oral antibiotics to treat chlamydial infection C) Topical mast cell stabilizer to treat allergies D) Antiviral medication for herpes zoster ophthalmicus

D) Antiviral medication for herpes zoster ophthalmicus

An adult client presents to the emergency department after an episode of syncope. Blood pressure is low and the pulse is thready and rapid. The client frequently voids large amounts of pale, clear urine. Lung sounds are clear and skin turgor is inelastic. Which endocrine disorder is the likely cause of these manifestations?

Diabetes insipidus

Which of the following types of imaging is preferred to evaluate the bone density of a patient with hyperparathyroidism?

Dual energy x-ray absorptiometry (DEXA) scan Explanation: Preferred types of scans for endocrine tissues include MRI of the pituitary gland and hypothalamus, CT scan of the adrenal glands, DEXA scan of bone density, and ultrasound of possible nodules of the thyroid gland. Bone density may be decreased in a patient with hyperparathyroidism. Page 765

Which type of imaging is preferred to evaluate the bone density of a client with hyperparathyroidism?

Dual energy x-ray absorptiometry (DXA) scan

When caring for a client who is being screened for osteoporosis, the nurse plans to evaluate the results of which diagnostic test?

Dual-energy x-ray absorptiometry (DXA)

Which statements concerning endocrinology laboratory testing are true? Select all that apply.

Endocrine tests are generally sensitive to storage requirements. Client education regarding endocrine testing must include specific test requirements.

Which of the following is an example of a positive feedback system? a) Somatostatin and thyroid-stimlulating hormone (TSH) b) Insulin and glucagon c) Cortisol and adrenocorticotropic hormone (ACTH) d) Estradiol and follicle-stimulating hormone (FSH)

Estradiol and follicle-stimulating hormone (FSH) In positive feedback control, rising levels of a hormone cause another gland to release a hormone that stimulates the first. Release of the female hormone estradiol during the follicular stage of the menstrual cycle causes increased release of gonadotropin (FSH) which further increases release of estradiol until the follicle dies and ceases production. The other options are examples of negative feedback. (less)

Acute otitis media (AOM) is the disorder in children for which antibiotics are most prescribed. What are the risk factors for AOM? Select all that apply.

Ethnicity Premature birth Genetic syndromes

The nurse understands that vestibular rehabilitation includes which of the following?

Exercises in balance retraining

The nurse understands that vestibular rehabilitation includes which of the following?

Exercises in balance retraining.

Which of the following nursing interventions is the highest priority intervention for a nurse caring for a 26-year-old client diagnosed with benign paroxysmal positional vertigo implement?

Fall precautions.

When sensors detect a change in a hormone level, the hormonal response is regulated by which mechanism that will return the level to within normal range?

Feedback

The nurse is caring for a client who received regular insulin at 7 am. Four hours later the nurse finds the client diaphoretic, cool, and clammy. Which of these interventions is the priority?

Give the client a concentrated carbohydrate.

During a physical exam, the nurse practitioner notes that the patients' optic disk is very pale with a larger size/depth of the optic cup. At this point, they are thinking the patient may have:

Glaucoma

A nurse assessing an 8-year-old child notes that the child is 6 feet tall. Which diagnostic test will be performed to evaluate the cause of the condition?

Glucose load

Which hormone is produced by the anterior pituitary gland?

Growth hormone (GH)

Which feature(s) indicates a carpopedal spasm in a client with hypoparathyroidism?

Hand flexing inward

Which statement best explains the function of hormone receptors?

Hormone receptors recognize a specific hormone and translate the hormonal signal into a cellular response

A client tells the doctor he has noticed a recent change in his vision: he can bring distant images into focus, but near images become blurred. The client has most likely developed:

Hyperopia

A patient tells the nurse of being able to see everything in the distance very clearly. The patient has difficulty seeing clearly close up. The nurse has to document in the electronic medical record what condition for this patient?

Hyperopia

As part of the community health department, a nurse is educating a group of diabetic clients about prevention of blindness. Which topics should be covered during this class? Select all that apply.

Importance of yearly eye exams Tight control of blood glucose levels Keep BP below 130/85 (American Heart Association Guidelines) Explanation: Current guidelines recommend that persons with diabetes have yearly eye examinations. Preventing diabetic retinopathy from developing or progressing is considered the best approach to preserving vision. Growing evidence suggests that careful control of blood glucose levels in persons with diabetes mellitus may retard the onset and progression of retinopathy. There also is a need for intensive management of hypertension and hyperlipidemia, both of which have been shown to increase the risk of diabetic retinopathy in persons with diabetes.

Following a meal, a client's blood glucose level has increased. In addition, the client's pancreas has increased the amount of insulin produced and released. Which phenomenon has occurred?

Increased hormone level according to a negative feedback mechanism

Diabetics are at higher risk than are the majority of the population for injury to organ systems in the body. Which organs are most at risk? a) Liver and eyes b) Kidneys and liver c) Pancreas and eyes d) Kidneys and eyes

Kidneys and eyes Diabetic nephropathy is the leading cause of chronic kidney disease, accounting for 40% of new cases. Also, diabetes is the leading cause of acquired blindness in the United States. The liver and pancreas are not organs that diabetes attacks.

Impaired function of which eye structure is responsible for blurred vision and loss of fine tuning of focus?

Lens

A patient with distal symmetric polyneuropathy usually begins by complaining of: a) Painful cramps in their calves. b) Loss of feeling or touch in the feet. c) Seeping wounds in lower legs. d) Burning in the big toe.

Loss of feeling or touch in the feet. Peripheral neuropathy is often associated with the insensate foot. The loss of feeling, touch, sensation, and position sense increases the risk of falling, serious burns and injuries to the feet. Burning in big toe is usually associated with gout. Cramps in the legs may be electrolyte imbalances and seeping wounds in lower legs could be a result of obesity, lymphedema, or diabetes to name a few.

A patient has been diagnosed with hemianopia. The patient asks the nurse what this is describing concerning the eye?

Loss of half of the visual field in one eye

A patient has been diagnosed with hemianopia. The patient asks the nurse what this is describing concerning the eye?

Loss of half of the visual field in one eye.

Which organ system is matched with the preferred type of imaging?

Magnetic resonance imaging (MRI) of the pituitary gland

Amblyopia, or lazy eye, occurs at a time when visual deprivation or abnormal binocular interactions occur in visual infancy. Whether amblyopia is reversible depends on which factor?

Maturity of the visual system at time of onset. Explanation: The reversibility of amblyopia depends on the maturity of the visual system at the time of onset and the duration of the abnormal experience.

The nurse is assessing a client who has been diagnosed with esotropia. The nurse would expect the assessment findings to include:

Medial deviation

Select the statement that best describes Meniere disease.

Meniere disease is a disorder of the inner ear due to distention of the endolymphatic compartment of the inner ear, causing a triad of hearing loss, vertigo, and tinnitus.

Select the statement that best describes Meniere disease. a) Meniere disease is a disorder of the inner ear due to distention of the endolymphatic compartment of the inner ear, causing a triad of hearing loss, vertigo, and tinnitus. b) Meniere disease is a disorder of the inner ear due to distention of the endolymphatic compartment of the inner ear: the client will be asymptomatic. c) Meniere disease is a disorder of the middle ear due to constriction of the endolymphatic compartment of the inner ear, causing hearing loss. d) Meniere disease is a bacterial infection of the outer inner ear due to distention of the endolymphatic compartment.

Meniere disease is a disorder of the inner ear due to distention of the endolymphatic compartment of the inner ear, causing a triad of hearing loss, vertigo, and tinnitus.

Neurotransmitters like catecholamines (e.g., dopamine and epinephrine) have a reaction time of:

Milliseconds

Neurotransmitters like catecholamines (ex. dopamine and epinephrine) have a reaction time of: a) 24-36 hours. b) Less than 10 minutes. c) Milliseconds. d) 4-7 days.

Milliseconds. The neurotransmitters, which control the opening of ion channels, have a reaction time of milliseconds.

Neurotransmitters like catecholamines (ex. dopamine and epinephrine) have a reaction time of:

Milliseconds. Explanation: The neurotransmitters, which control the opening of ion channels, have a reaction time of milliseconds.

A young teenager is brought to the clinic for an eye exam. She has been complaining that she is having difficulty in school as she cannot see the blackboard as clearly as she used to. She tells the nurse that it is blurry. What should the nurse suspect is wrong with this child?

Myopia

The hormone levels in the body need to be kept within an appropriate range. How is this accomplished for many of the hormones in the body?

Negative feedback loop Explanation: The level of many of the hormones in the body is regulated by negative feedback mechanisms. The other answers are incorrect. Page 762

The hormone levels in the body need to be kept within an appropriate range. How is this accomplished for many of the hormones in the body? a) Regulated feedback loop b) Negative feedback loop c) Positive feedback loop d) Sensory feedback loop

Negative feedback loop The level of many of the hormones in the body is regulated by negative feedback mechanisms. The other answers are incorrect.

When conditions occur that impair retinal blood flow, such as hyperviscosity of the blood or a sickle cell crisis, what can occur in the eye?

Neovascularization.

A nurse is teaching a client with newly diagnosed type 1 diabetes about the importance of blood glucose control to decrease the risk of which potential chronic complications of diabetes? Select all that apply.

Nephropathy Neuropathy Retinopathy Gastroparesis

An adult client with suspected hypothyroidism is scheduled for a thyrotropin-releasing hormone (TRH) stimulation test to evaluate pituitary response. Which test results would confirm secondary hypothyroidism?

No increase in TSH

Stepping out of a mall and into the sunshine has caused a man's pupils to constrict. Place the following anatomical components of the man's pupillary reflex in the ascending chronological order that they responded to the light. Use all the options.

Oculomotor nuclei, retinal ganglionic cells, preganglionic neurons, and pretectal nuclei.

The three layers of the cornea are separated by two important basement membranes. What are characteristics of the Bowman membrane? Select all that apply.

Opaque scar can form that impairs vision, and Acts as a barrier to infection.

A patient has glaucoma. The nurse is taking a health history and knows that the most common form of glaucoma is which type?

Open angle

Conductive hearing loss can occur for a variety of reasons, including foreign bodies in the ear canal, damage to the ear drum, or disease. What disease is associated with conductive hearing loss? a) Huntington disease b) Paget disease c) Parkinson disease d) Alzheimer disease

Paget disease More permanent causes of hearing loss are thickening or damage of the tympanic membrane or involvement of the bony structures (ossicles and oval window) of the middle ear due to otosclerosis or Paget disease. -Huntington, Alzheimer, and Parkinson diseases are not associated with conductive hearing loss.

A client has a disorder that causes him to have a deficiency of all of the anterior pituitary hormones. What condition will the nurse education the client about for replacement therapy? a) Laron-type dwarfism b) Congenital GH deficiency c) Psychosocial dwarfism d) Panhypopituitarism

Panhypopituitarism The term panhypopituitarism refers to conditions that cause a deficiency of all of the anterior pituitary hormones. Laron-type dwarfism is when GH levels are normal or elevated, but there is a hereditary defect in IGF production that can be treated directly with IGF-1 replacement. Psychosocial dwarfism involves a functional hypopituitarism and is seen in some emotionally deprived children. Congenital GH deficiency is associated with decreased birth length, followed by a decrease in growth rate that can be identified by careful measurement during the first year and that becomes obvious by 1 to 2 years of age.

Which type of receptor is mediated through vision in dim light?

Photoreceptors.

A client receives steroids for several months to treat an inflammatory condition. Which action by the primary health care provider indicates an understanding of the negative feedback mechanism when the client no longer needs the medication?

Prescribing a tapering dose of the medication over weeks

What is the nursing focus when the nurse performs a hearing test on a newborn infant?

Preventing developmental delays

What is the nursing focus when the nurse performs a hearing test on a newborn infant?

Preventing developmental delays Explanation: Early detection of hearing loss can prevent developmental delay by allowing early intervention. The hearing test on a newborn evaluates the physical ability to respond, not the ability to perform language or communication skills. The test records the infant's ability to perceive sound but does not improve it. Older adults, not infants, are at risk for depression as a result of hearing loss.

Select the most accurate statement regarding measurements of urinary hormone.

Provide a better measure of hormone levels during a designated period.

The nurse is treating a client with a history of chronic alcohol abuse. The client's spouse tells the nurse that the client has no difficulty speaking and responds when spoken to but is having difficulty understanding what is being said. The spouse asks if the nurse believes the client needs a hearing aid. The nurse explains to the spouse that the client probably does not require a hearing aid but may be suffering from:

Receptive aphasia Explanation: Chronic alcohol abuse causes damage to the Wernicke's area in the brain, which controls a person's ability to understand spoken language, and which is termed receptive aphasia. Clients who have receptive aphasia can speak without difficulty and hear words spoken, but cannot understand what is being said.

An older adult client comes to the clinic complaining of seeing flashing lights and small spots. The client tells the nurse that this has been going on for over 24 hours but now it is as if there is a dark curtain whenever the client opens the eyes. The client asks the nurse if this means that blindness is imminent. What diagnosis should the nurse suspect?

Retinal detachment Explanation: The primary symptom of retinal detachment consists of painless changes in vision. Commonly, flashing lights or sparks followed by small floaters or spots in the field of vision occur. As the detachment progresses the person perceives a shadow or dark curtain across the visual field.

A client has been admitted to the postsurgical unit following a thyroidectomy. To promote comfort and safety, how should the nurse best position the client?

Semi-Fowler with the head supported on two pillows

A client has received an injection containing thyrotropin-releasing hormone (TRH) and is now being assessed for serum levels of thyroid-stimulating hormone (TSH). Which type of diagnostic testing is this client undergoing?

Stimulation testing

The nurse is discussing measures that a client may take to prevent barotrauma related to airplane travel. Which of the following measures will the nurse recommend during changes in air pressure?

Swallowing

The nurse is discussing measures that a client may take to prevent barotrauma related to airplane travel. Which measure will the nurse recommend during changes in air pressure?

Swallowing Explanation: Swallowing, yawning, and chewing gum are measures that open the Eustachian tube, which equalizes air pressure in the middle ear.

The nurse is discussing measures that a client may take to prevent barotrauma related to airplane travel. Which of the following measures will the nurse recommend during changes in air pressure? a) Swallowing b) Mouth breathing c) Forcefully coughing d) Cupping the ears with one's hands

Swallowing Swallowing, yawning, and chewing gum are measures that open the eustachian tube, which equalizes air pressure in the middle ear.

The nurse is discussing measures that a client may take to prevent barotrauma related to airplane travel. Which of the following measures will the nurse recommend during changes in air pressure?

Swallowing.

Which statement about immunoradiometric assay (IRMA) testing for measuring plasma hormone levels is most accurate?

These tests are very specific since they utilize two antibodies instead of one.

A client who has just undergone a thyroidectomy is experiencing high fever, tachycardia, and extreme restlessness. The nurse would interpret these manifestations as: a) Hypothyroidism b) Thyroid crisis c) Addisonian crisis d) Myxedematous coma

Thyroid crisis The symptoms this client is experiencing are related to thyroid crisis or storm and must be treated immediately to prevent death. Myxedematous coma is related to hypothyroidism but typically does not occur after a thyroidectomy. Addisonian crisis is related to hypoadrenalism.

Imaging has proven useful in both the diagnosis and follow-up of endocrine disorders. Two types of imaging studies are useful when dealing with endocrine disorders: Isotopic imaging and nonisotopic imaging. What is an example of isotopic imaging?

Thyroid scan

The nurse administers a glucocorticoid medication to a client with pneumonia. Which of these does the nurse teach the client is the purpose of the medication? To regulate glucose metabolism To decrease appetite To decrease airway inflammation To decrease stress of illness

To decrease airway inflammation

It is important to differentiate between the kinds of hearing loss so they can be appropriately treated. What is used to test between conductive and sensorineural hearing loss? a) Audioscope b) Tuning fork c) Tone analysis d) Audiometer

Tuning fork Tuning forks are used to differentiate conductive and sensorineural hearing loss. -Audioscope, audiometer, and tone analysis do not differentiate between conductive and sensorineural hearing loss.

It is important to differentiate between the kinds of hearing loss so they can be appropriately treated. What is used to test between conductive and sensorineural hearing loss?

Tuning fork.

A 2-year-old child who has had otitis media (OM) for 4 months and been treated with several courses of antibiotics now appears to have some hearing loss. The nurse anticipates that the most appropriate treatment for the child would be:

Tympanostomy tube insertion

Which of the following is a risk factor for the development of acute otitis media in a child?

Upper respiratory infection

Which condition or situation is a risk factor for the development of acute otitis media in a child?

Upper respiratory infection Explanation: Acute OM, which refers to an acute middle ear infection, usually follows an upper respiratory tract infection. The other options may be considered risk factors for certain types of hearing loss.

Sometimes the measurement of hormones is done through a urine sample. What is an advantage of measuring hormone levels through a urine sample rather than a blood sample?

Urine samples are easily obtained.

The parent of a 10-year-old child who has developed severe inflammation, excessive tearing, fever, clear discharge from his eyes, and is very listless arrives at the medical care unit. The parent tells the receiving nurse that the child started swimming lessons at the community pool last week. The nurse would suspect which of the following?

Viral conjunctivitis

A young patient is brought to the clinic with symptoms of a runny nose, small amount of drainage from both eyes, fever, and malaise. The mother states to the nurse, "I knew she was going to be sick. That swimming pool she played in 2 days ago was filthy." What disease should the nurse suspect this child is experiencing?

Viral conjunctivitis.

The nurse is caring for a client with diabetes who has developed gastroparesis. Which symptom does the nurse expect the client to report?

Vomiting after eating

The nurse practitioner who assesses a patient with hyperthyroidism would expect the patient to report which of the following conditions?

Weight loss

A client is scheduled for a suppression test as part of the diagnostic testing for his suspected endocrine disorder. The results of this test will help the care team determine:

Whether the client is producing excessive hormone levels. Explanation: Suppression tests are used when hyperfunction of an endocrine organ is suspected. Suppression tests are not used to gauge pituitary or hypothalamus function. Stimulation tests are used to rule out hypofunction of an endocrine organ. Page 764

A patient has been diagnosed with bacterial conjunctivitis. This type of infection is usually characterized by large amounts of what color drainage? a) Serous b) Yellow-green c) Blue-yellow d) Pink-red

Yellow-green The infection usually is characterized by large amounts of yellow-green drainage. The eyelids are sticky, and there may be excoriation of the lid margins.

In which order will the nurse take these steps to prepare NPH 20 units and regular insulin 2 units using the same syringe? (Put a comma and a space between each answer choice [A, B, C, D, E]). a. Rotate NPH vial. b. Withdraw regular insulin. c. Withdraw 20 units of NPH. d. Inject 20 units of air into NPH vial. e. Inject 2 units of air into regular insulin vial.

a, d, e, b, c

Which question during the assessment of a diabetic patient will help the nurse identify autonomic neuropathy? a. "Do you feel bloated after eating?" b. "Have you seen any skin changes?" c. "Do you need to increase your insulin dosage when you are stressed?" d. "Have you noticed any painful new ulcerations or sores on your feet?"

a. "Do you feel bloated after eating?"

A 27-year-old patient admitted with diabetic ketoacidosis (DKA) has a serum glucose level of 732 mg/dL and serum potassium level of 3.1 mEq/L. Which action prescribed by the health care provider should the nurse takefirst? a. Place the patient on a cardiac monitor. b. Administer IV potassium supplements. c. Obtain urine glucose and ketone levels. d. Start an insulin infusion at 0.1 units/kg/hr.

a. Place the patient on a cardiac monitor.

53. hearing a. Acous/o k. kerat/o b. Blephar/o l. myring/o c. Chromat/o m. ophthalm/o d. Conjunctiv/o n. ot/o e. Choroid/o o. salping/o f. Core/o p. -metry g. Corne/o q. -opsia h. Dacry/o r. -ptosis i. dipl/o s. -rrhea j. irid/o t. -tropia

a. acous/o

Although both vertigo and dizziness can result from peripheral or central vestibular disorders, vertigo is distinctly different because it causes:

an illusion of motion. Explanation: Vertigo or dizziness can result from peripheral or central vestibular disorders (proprioception) unrelated to hearing loss. Vertigo is a vestibular disorder in which a unique illusion of motion occurs. Persons with vertigo frequently describe it as a sensation of spinning or tumbling, a "to-and-fro" motion, or falling forward or backward. Light-headedness, faintness, and unsteadiness are different in that the person perceives weakness yet still has a sense of balance. Syncope (loss of consciousness) is not directly associated with the sensation of vertigo.

A client is distraught because she has suddenly lost vision in the right half of both of her eyes. The care team's subsequent diagnostic testing will address the likelihood of:

an optic nerve lesion.

A client is distraught because she has suddenly lost vision in the right half of both of her eyes. The care team's subsequent diagnostic testing will address the likelihood of:

an optic nerve lesion. Explanation: This client is reporting complete right homonymous hemianopia, which is the result of an optic nerve lesion. Multiple sclerosis, diabetes, and macular degeneration produce more gradual and insidious vision changes that do not normally result in hemianopia.

Age-related macular degeneration that is dry is characterized by:

atrophy of the Bruch membrane. Explanation: Nonexudative age-related macular degeneration is characterized by various degrees of atrophy and degeneration of the outer retina, Bruch membrane, and choriocapillary layer (choriocapillaris) of the choroid. It does not involve leakage of blood or serum; hence, it is called dry age-related macular degeneration. The other answers are characterizations of the "wet" form of macular degeneration.

An active 28-year-old male with type 1 diabetes is being seen in the endocrine clinic. Which finding may indicate the need for a change in therapy? a. Hemoglobin A1C level 6.2% b. Blood pressure 146/88 mmHg c. Heart rate at rest 58 beats/minute d. High density lipoprotein (HDL) level 65 mg/dL

b. Blood pressure 146/88 mmHg

The nurse is assessing a 22-year-old patient experiencing the onset of symptoms of type 1 diabetes. Which question is most appropriate for the nurse to ask? a. "Are you anorexic?" b. "Is your urine dark colored?" c. "Have you lost weight lately?" d. "Do you crave sugary drinks?"

c. "Have you lost weight lately?"

The nurse determines a need for additional instruction when the patient with newly diagnosed type 1 diabetes says which of the following? a. "I can have an occasional alcoholic drink if I include it in my meal plan." b. "I will need a bedtime snack because I take an evening dose of NPH insulin." c. "I can choose any foods, as long as I use enough insulin to cover the calories." d. "I will eat something at meal times to prevent hypoglycemia, even if I am not hungry."

c. "I can choose any foods, as long as I use enough insulin to cover the calories."

A client's chart documents the finding of cholesteatoma. The nurse interprets this to mean that the client has:

cystlike mass in the middle ear. Explanation: Cholesteatoma is a cystlike mass of the middle ear that often extends to involve the temporal bone.

A hospitalized diabetic patient received 38 U of NPH insulin at 7:00 AM. At 1:00 PM, the patient has been away from the nursing unit for 2 hours, missing the lunch delivery while awaiting a chest x-ray. To prevent hypoglycemia, the best action by the nurse is to a. save the lunch tray for the patient's later return to the unit. b. ask that diagnostic testing area staff to start a 5% dextrose IV. c. send a glass of milk or orange juice to the patient in the diagnostic testing area. d. request that if testing is further delayed, the patient be returned to the unit to eat.

d. request that if testing is further delayed, the patient be returned to the unit to eat

A client is being evaluated for hypothyroidism. During assessment, the nurse should stay alert for:

decreased body temperature and cold intolerance.

55. pupil a. Acous/o k. kerat/o b. Blephar/o l. myring/o c. Chromat/o m. ophthalm/o d. Conjunctiv/o n. ot/o e. Choroid/o o. salping/o f. Core/o p. -metry g. Corne/o q. -opsia h. Dacry/o r. -ptosis i. dipl/o s. -rrhea j. irid/o t. -tropia

f. core/o

74. retinal damage marked by aneurismal dilation of blood vessels that occurs as a result of diabetes a. achromatopsia k. macular degeneration b. anacusis l. meniere disease c. astigmatism m. myringoplasty d. audiometry n. myringotomy e. cataract o. otoscopy f. diabetic retinopathy p. photophobia g. exotropia q. rinne test h. glaucoma r. strabismus i. hordeolum s. tinnitus j. iridoplegia t. vertigo

f. diabetic retinopathy

The nurse is caring for a client who is hearing impaired. The nurse understands that effective communication with the client includes:

facing the client so that the client can visualize the lips. Explanation: It is important that nurses speaking to people with hearing impairment face the person and articulate so that lip-reading cues can be used. The other options are not recommended.

54. cornea a. Acous/o k. kerat/o b. Blephar/o l. myring/o c. Chromat/o m. ophthalm/o d. Conjunctiv/o n. ot/o e. Choroid/o o. salping/o f. Core/o p. -metry g. Corne/o q. -opsia h. Dacry/o r. -ptosis i. dipl/o s. -rrhea j. Irid/o t. -tropia

g. corne/o

A client with hyperthyroidism is being treated with medication that blocks the activity of thyroid-stimulating hormone. Her care team has determined that she has been overproducing TSH. This client will have lost her ability to:

have negative feedback regulation. Explanation: When the sensors detect a decrease in hormone levels, they initiate changes that cause an increase in hormone production; when hormone levels rise above the set point of the system, the sensors cause hormone production and release to decrease. Positive feedback increases activity rather than shuts if off. Metabolism of the hormone is not an issue in regulation. Page 759

The nurse documents the presence of nystagmus when assessing a client. This can be interpreted as:

involuntary eye movements that preserve eye fixation on stable objects in the visual field. Explanation: Nystagmus refers to the involuntary eye movements that preserve eye fixation on stable objects in the visual field during angular and rotational movements of the head. As the body rotates, the vestibuloocular reflexes cause a slow compensatory drifting of eye movement in the opposite direction, thus stabilizing the binocular fixation point.

61. iris a. Acous/o k. kerat/o b. Blephar/o l. myring/o c. Chromat/o m. ophthalm/o d. Conjunctiv/o n. ot/o e. Choroid/o o. salping/o f. Core/o p. -metry g. Corne/o q. -opsia h. Dacry/o r. -ptosis i. dipl/o s. -rrhea j. irid/o t. -tropia

j. Irid/o

90. paralysis of the iris a. achromatopsia k. macular degeneration b. anacusis l. meniere disease c. astigmatism m. myringoplasty d. audiometry n. myringotomy e. cataract o. otoscopy f. diabetic retinopathy p. photophobia g. exotropia q. rinne test h. glaucoma r. strabismus i. hordeolum s. tinnitus j. iridoplegia t. vertigo

j. iridoplegia

88. surgical repair of a perforated eardrum with a tissue graft a. achromatopsia k. macular degeneration b. anacusis l. meniere disease c. astigmatism m. myringoplasty d. audiometry n. myringotomy e. cataract o. otoscopy f. diabetic retinopathy p. photophobia g. exotropia q. rinne test h. glaucoma r. strabismus i. hordeolum s. tinnitus j. iridoplegia t. vertigo

m. myringoplasty

60. eye a. Acous/o k. kerat/o b. Blephar/o l. myring/o c. Chromat/o m. ophthalm/o d. Conjunctiv/o n. ot/o e. Choroid/o o. salping/o f. Core/o p. -metry g. Corne/o q. -opsia h. Dacry/o r. -ptosis i. dipl/o s. -rrhea j. irid/o t. -tropia

m. ophthalm/o

A middle-aged female client has been diagnosed with a thyroid condition. The nurse educates the client about the prescription and needed follow-up lab work, which will help regulate the dosage. The client asks, "Why do I not return to the clinic for weeks, since I am starting the medication tomorrow morning?" The nurse bases the answer on the knowledge that thyroid hormones:

may take days for the full effect to occur, based on the mechanism of action.

86. visual examination of the ear, especially the eardrum a. achromatopsia k. macular degeneration b. anacusis l. meniere disease c. astigmatism m. myringoplasty d. audiometry n. myringotomy e. cataract o. otoscopy f. diabetic retinopathy p. photophobia g. exotropia q. rinne test h. glaucoma r. strabismus i. hordeolum s. tinnitus j. iridoplegia t. vertigo

o. otoscopy

Cardiac effects of hyperthyroidism include

palpitations

A client visits the physician's office complaining of agitation, restlessness, and weight loss. The physical examination reveals exophthalmos, a classic sign of Graves' disease. Based on history and physical findings, the nurse suspects hyperthyroidism. Exophthalmos is characterized by:

protruding eyes and a fixed stare.

A nurse should expect a client with hypothyroidism to report:

puffiness of the face and hands.

63. vision a. Acous/o k. kerat/o b. Blephar/o l. myring/o c. Chromat/o m. ophthalm/o d. Conjunctiv/o n. ot/o e. Choroid/o o. salping/o f. Core/o p. -metry g. Corne/o q. -opsia h. Dacry/o r. -ptosis i. dipl/o s. -rrhea j. irid/o t. -tropia

q. -opsia

65. discharge, flow a. Acous/o k. kerat/o b. Blephar/o l. myring/o c. Chromat/o m. ophthalm/o d. Conjunctiv/o n. ot/o e. Choroid/o o. salping/o f. Core/o p. -metry g. Corne/o q. -opsia h. Dacry/o r. -ptosis i. dipl/o s. -rrhea j. irid/o t. -tropia

s. -rrhea

81. ringing in the ears a. achromatopsia k. macular degeneration b. anacusis l. meniere disease c. astigmatism m. myringoplasty d. audiometry n. myringotomy e. cataract o. otoscopy f. diabetic retinopathy p. photophobia g. exotropia q. rinne test h. glaucoma r. strabismus i. hordeolum s. tinnitus j. iridoplegia t. vertigo

s. tinnitus

While reviewing the major actions of follicle-stimulating hormone (FSH), the faculty points out that in males this hormone is responsible for the:

sperm production

A homeless individual is brought to the emergency department (ED) after the police could not wake the person. The client's breath is fruity and others state that the client has been acting "different." Initial blood work identifies a blood glucose level of 642. The client is admitted to the ICU in ketoacidosis. The client is prescribed an insulin drip (IV infusion). The rationale for this is related to: a) protecting the client from shifting potassium from the intracellular to the extracellular compartment. b) ketoacidosis, which causes vasoconstriction of blood vessels resulting in poor absorption to insulin injections. c) the client's neurological condition; subcutaneous insulin injects are not well absorbed if the client is semi-comatose. d) the fact that a lower pH of the body fluids (ketoacidosis) reduces insulin binding.

the fact that a lower pH of the body fluids (ketoacidosis) reduces insulin binding. The affinity of receptors for binding hormones is also affected by a number of conditions. For example, the pH of the body fluids plays an important role in the affinity of insulin receptors. In ketoacidosis, a lower pH reduces insulin binding. While lowering the blood glucose level, potassium can be shifted from the extracellular to intracellular compartment, resulting in hypokalemia. Neurologic status does not cause vasoconstriction of vessels or poor absorption of subcutaneous injections.

Which statement describes how water-soluble peptides, such as parathyroid hormone or glucagon, exert their effect on cells?

they bind as receptors

The nurse is caring for a client diagnosed with labrynthitis. The nurse explains that manifestations of this disorder are generally expected:

to resolve in 10 to 14 days. Explanation: Labrynthitis (also called acute vestibular neuronitis) generally resolves in about 10 to 14 days.

A nurse is caring for a client with diabetes insipidus. The nurse should anticipate administering:

vasopressin.

The nurse is teaching a teenaged client strategies to avoid recurrent ear infections. The nurse understands that further teaching is required when the client states which of the following? a) "I will take my allergy medication as it has been prescribed." b) "I will use the eardrops as prescribed for me." c) "I will dry the inside of my ears thoroughly with cotton-tipped applicators after swimming." d) "I'll wear ear plugs each time I swim."

"I will dry the inside of my ears thoroughly with cotton-tipped applicators after swimming." Otitis externa treatment usually includes the use of eardrops containing an appropriate antimicrobial or antifungal agent. -Use of ear plugs is recommended to prevent moisture. Cotton-tipped applicators and other devices should be avoided.

A female client with rheumatoid arthritis has taken high doses of aspirin for several years to control inflammatory pain. Which of the following statements leads the health care provider to suspect the client has developed ototoxicity?

"I've been getting dizzy and light-headed. I seem to have a constant ringing in my ear."

A patient tells the nurse that he has hearing loss due to otosclerosis. The nurse realizes that the patient has which of the following?

Conductive hearing loss

The nurse is teaching a client who has tinnitus about suggested dietary modifications. The most important information for the nurse to include would be:

"Cut down on takeout meals because of high amounts of monosodium glutamate."

Which of these clients' statements would be most suggestive of retinal detachment?

"I feel like there's a shadow that's blocking my vision."

14. The colored portion of the eye is the: A. Sclera B. Retina C. Iris D. Choroid E. Cornea

C. Iris

Which complications could occur when an adult client has an anterior hypophysectomy? Select all that apply.

Hypothyroidism Addison disease

A nurse who works in the office of an endocrinologist is orienting a new staff member. Which teaching point should the nurse include in the orientation?

"A single hormone can act on not only one process or organ but often on several different locations or processes." Explanation: A single hormone can exert various effects in different tissues, or conversely, a single function can be regulated by several different hormones. Hormones act both distantly from their source and more locally, as in the case of autocrine and paracrine actions. Hormones are normally present at all times. Page 753

The nurse is teaching a client who has tinnitus about suggested dietary modifications. The most important information for the nurse to include would be:

"Cut down on takeout meals because of high amounts of monosodium glutamate." Explanation: Tinnitus can be caused by some cheeses, red wine, monosodium glutamate, and caffeine. These items should be reduced in the diet to help control the symptom.

The nurse is preparing a client for a radioiodine test to assess thyroid functioning. Which question is most important for the nurse to ask the client prior to preparing this client for this diagnostic test? "Have you had a CT scan in the past month?" "Are you claustrophobic or have trouble lying still?" "Have you had abnormal thyroid hormone levels?" "Will you be able to fast for 12 hours for the test?"

"Have you had a CT scan in the past month?"

The nurse is teaching a teenaged client strategies to avoid recurrent ear infections. The nurse understands that further teaching is required when the client states:

"I will dry the inside of my ears thoroughly with cotton-tipped applicators after swimming." Explanation: Otitis externa treatment usually includes the use of eardrops containing an appropriate antimicrobial or antifungal agent. Use of ear plugs is recommended to prevent moisture. Cotton-tipped applicators and other devices should be avoided.

The nurse is teaching a teenaged client strategies to avoid recurrent ear infections. The nurse understands that further teaching is required when the client states which of the following?

"I will dry the inside of my ears thoroughly with the cotton-tipped applicators after swimming."

A nurse is caring for a client who was recently diagnosed with hyperparathyroidism. Which statement by the client indicates the need for additional discharge teaching?

"I will increase my fluid and calcium intake."

A client has developed a tumor of the posterior pituitary gland. The client is at risk for problems with secretions of: a) Somatostatin and prolactin b) Growth hormone-releasing hormone (GHRH) and dopamine c) Adrenocorticotropic hormone (ACTH) and vasopressin d) Antidiuretic hormone (ADH) and oxytocin

Antidiuretic hormone (ADH) and oxytocin The posterior pituitary secretes ADH and oxytocin/vasopressin, while the anterior pituitary secretes the hormones listed in the other choices.

Otitis media (OM), which can occur is any age group, is the most common diagnosis made by health care providers who care for children. Which bacterial pathogen causes the largest proportion of cases that result in sensorineural hearing loss?

Streptococcus pneumoniae

A client has been diagnosed with sensorineural hearing loss. Which should the care team assess to determine a plausible cause?

The client's recent medication use.

71. condition of color blindness that is more common in men. a. achromatopsia k. macular degeneration b. anacusis l. meniere disease c. astigmatism m. myringoplasty d. audiometry n. myringotomy e. cataract o. otoscopy f. diabetic retinopathy p. photophobia g. exotropia q. rinne test h. glaucoma r. strabismus i. hordeolum s. tinnitus j. iridoplegia t. vertigo

a. achromatopsia

A diabetic patient who has reported burning foot pain at night receives a new prescription. Which information should the nurse teach the patient about amitriptyline (Elavil)? a. Amitriptyline decreases the depression caused by your foot pain. b. Amitriptyline helps prevent transmission of pain impulses to the brain. c. Amitriptyline corrects some of the blood vessel changes that cause pain. d. Amitriptyline improves sleep and makes you less aware of nighttime pain.

b. Amitriptyline helps prevent transmission of pain impulses to the brain.

The nurse is interviewing a new patient with diabetes who receives rosiglitazone (Avandia) through a restricted access medication program. What is most important for the nurse to report immediately to the health care provider? a. The patient's blood pressure is 154/92. b. The patient has a history of emphysema. c. The patient's blood glucose is 86 mg/dL. d. The patient has chest pressure when walking.

d. The patient has chest pressure when walking.

A nurse is caring for a client with hypoparathyroidism. During assessment, the nurse elicits a positive Trousseau's sign. What does the nurse observe to verify this finding?

hand flexing inward

A young client has a significant height deficit and is to be evaluated for diagnostic purposes. What could be the cause of this client's disorder?

pituitary disorder

When caring for a client with diabetes insipidus, the nurse expects to administer:

vasopressin.

The nursing instructor is teaching a class on diabetes and discusses complications of the disease. She further states that diabetic retinopathy is one of the leading causes of blindness. What does she tell the students are major risk factors for developing diabetic retinopathy? Select all that apply.

• Chronic hyperglycemia • Hypertension • Smoking

Vision is a special sensory function that incorporates the visual receptors functions of which parts of the eye? Select all that apply.

• Eyeball • Optic nerve • Retina

As part of the community health department, a nurse is educating a group of diabetic clients about prevention of blindness. Which of the following topics should be covered during this class? Select all that apply.

• Importance of yearly eye exams • Tight control of blood glucose levels • Keep BP below 130.85 (Am. Heart Assoc. Guidelines)

A client is admitted in the ICU with diagnosis of hyperglycemic hyperosmolar state (HHS). The nurse caring for the client knows that the client's elevated serum osmolality has pulled water out of this brain cells based on which of the following assessment findings? Select all that apply. a) Unable to respond verbally to questions b) After the sole of the foot has been firmly stroked, the toes flex and flare out c) Increase in urine output in proportion to the increase in blood glucose d) Weakness one side of the body e) Uncontrollable twitching of a muscle group

• Weakness one side of the body • After the sole of the foot has been firmly stroked, the toes flex and flare out • Unable to respond verbally to questions • Uncontrollable twitching of a muscle group HHS is characterized by hyperglycemia (blood glucose >600 mg/dL); hyperosmolarity (plasma osmolarity >310 mOsm/L); and dehydration, the absence of ketoacidosis, and depression of the sensorium. The most prominent manifestations are weakness, dehydration, polyuria, neurologic signs and symptoms, and excessive thirst. The neurologic signs include hemiparesis (weakness on one side of the body), Babinski reflex (the sole of the foot has been firmly stroked, the toes flex and flare out), aphasia (unable to respond verbally to questions), muscle fasciculations (uncontrollable twitching of a muscle group), hyperthermia, hemianopia, nystagmus, visual hallucinations, seizures, and coma

The nurse is educating a patient diagnosed with Meniere's disease on lifestyle modifications to reduce the frequency and/or intensity of exacerbations. Which should the nurse include?

Decrease dietary intake of sodium

A 3-year-old girl has just been diagnosed with type 1A diabetes. Her parents are currently receiving education from the diabetes education nurse. How can the nurse best explain to the parents the etiology (cause) of their daughter's diabetes?

"The problem that underlies her diabetes is that her own body has destroyed the cells in her pancreas that produce insulin."

The critical care nurse has just admitted a client with diabetic ketoacidosis (DKA) whose blood glucose level is 877 mg/dL (48.67 mmol/L). The client's breath has a fruity odor and the client is confused. Which of these does the nurse set as the priority at this time?

Administration of intravenous fluids

A female client with rheumatoid arthritis has taken high doses of aspirin for several years to control inflammatory pain. Which of the following statements leads the health care provider to suspect the client has developed ototoxicity?

"I've been getting dizzy and light-headed. I seem to have a constant ringing in my ear." Ototoxicity results in sensorineural hearing loss. Vestibular symptoms of ototoxicity include light-headedness, giddiness, and dizziness; if toxicity is severe, cochlear symptoms consisting of tinnitus or hearing loss occur. -The symptoms of drug-induced hearing loss may be transient, as often is the case with salicylates and diuretics, or they may be permanent. -Hearing loss in the elderly is further characterized by reduced hearing sensitivity and speech understanding in noisy environments, slowed central processing of acoustic information, and impaired localization of sound sources. -High-frequency warning sounds, such as beepers, turn signals, and escaping steam, are not heard and localized, with potentially dangerous results. -Clinical measures for hearing loss such as whispered voice tests and finger friction tests are reportedly imprecise and are not reliable methods for screening.

The father of a third grade girl has brought his daughter to a walk-in clinic because he believes the girl has pink eye, which has been going around the students in her class. The nurse at the clinic concurs with the father's suspicion of conjunctivitis. Which follow-up explanation by the nurse is most accurate?

"The surfaces of her eyes have bacteria or a virus established, and it's important to maintain good hand hygiene until it goes away."

The father of a third grade girl has brought his daughter to a walk-in clinic because he believes the girl has pink eye, which has been going around the students in her class. The nurse at the clinic concurs with the father's suspicion of conjunctivitis. Which of the follow-up explanation by the nurse is most accurate?

"The surfaces of her eyes have bacteria or a virus established, and it's important to maintain good hand hygiene until it goes away."

Distraught at the persistent ringing in his ears and his inability to alleviate it, a 50 year-old man has visited his healthcare provider. After diagnostic testing, no objective cause (like impacted cerumen or vascular abnormality) was found. Given these testing results, which of the following teaching points by the care provider is most appropriate?

"There are some treatments like tinnitus retraining therapy, which includes the extended use of low-noise generators, which has shown good success."

Following an oral glucose tolerance, a 36-year-old mother of 4 has been diagnosed with gestational diabetes mellitus (GDM), a problem that was not present in any of her previous pregnancies. What should her primary care provider tell her about this new health problem?

"Your baby could become too large or have low blood sugars if we're not vigilant about controlling your sugars."

32. A perception of ringing, hissing, or other sounds in the ears or head when no external sound is present is called: a. Tinnitus b. Anacusis c. Presbycusis d. Anklyosis e. Atosclerosis

A. Tinnitus

The nurse is working in a pediatric clinic. Which of the following children would the nurse recognize as having isosexual precocious puberty? a) A 5-year-old African-American female with developing breasts and pubic hair b) A 14-year old Caucasian female who has not yet developed secondary sex characteristics c) A 10-year-old African-American female who has begun menstruating d) A 9-year-old Caucasian female who has small breast buds

A 5-year-old African-American female with developing breasts and pubic hair Precocious puberty is now defined as the appearance of secondary sexual development before the age of 7 years in white girls and 6 years in African-American girls. In boys of both races, the lower age limit remains 9 years. However, it is recognized that puberty can develop earlier in boys with obesity.

Which of the following individuals is most likely to be diagnosed with a central vestibular disorder?

A woman who has ongoing difficulty balancing herself when walking.

18. An excision of a part or all of the eyelid is called a(n): a. Blepharectomy b. Ophthalmectomy c. Blepharotomy d. Keratorrhexis e. Keratotomy

A. Blepharectomy

The nurse is caring for a client who reports, "My ears are constantly ringing!" The nurse will evaluate the client further for the use of which medication?

Acetylsalicylic acid (aspirin) Explanation: Tinnitus ("ringing in the ears") may be caused by aspirin.

A female client with hyperglycemia who weighs 210 lb (95 kg) tells the nurse that her husband sleeps in another room because her snoring keeps him awake. The nurse notices that the client has large hands and a hoarse voice. Which disorder would the nurse suspect as a possible cause of the client's hyperglycemia?

Acromegaly

The nurse is teaching a client about the treatment of open-angle glaucoma. The most appropriate information for the nurse to give the client would be:

Administration of topical beta-adrenergic antagonists to lower the pressure.

What is ocular muscle imbalance resulting in "lazy eye" called?

Amblyopia.

A 46 year-old male has presented to the emergency department because of the eye pain, severe headache and blurred vision that has followed an eye exam at an optometrist's office earlier in the day. The client tells the triage nurse that he received eye drops during the exam "to keep my pupils wide open." What differential diagnosis will the care team first suspect?

Angle-closure glaucoma

When discussing luteinizing hormone and follicle-stimulating hormone with students, the instructor will emphasize that these hormones are under the control of:

Anterior pituitary gland

A female client with rheumatoid arthritis has taken high doses of aspirin for several years to control inflammatory pain. Which of the following statements leads the health care provider to suspect the client has developed ototoxicity? A) "I can't go to the movies anymore. It's so noisy, I miss half the words." B) "I've been getting dizzy and light-headed. I seem to have a constant ringing in my ear." C) "I almost got hit by a garbage truck. I didn't hear its backup beeper." D) "When my grandchildren whisper, I can't hear a word they are saying."

B) "I've been getting dizzy and light-headed. I seem to have a constant ringing in my ear."

Which of the following vision deficits is a clinician justified in attributing to the normal aging process? A) Conjunctivitis B) Presbyopia C) Strabismus D) Angle-closure glaucoma

B) Presbyopia

The nurse suggests which of the following to a mother who is taking her two-month-old infant on an airplane ride?

Breast feed or bottle feed the infant during the airplane's ascent or descent.

38. Radial keratotomy is used in treatment of: a. Macular degeneration b. Night blindness c. Nearsightedness d. xerophthalmia e. mastoiditis

C. Nearsightedness

The nurse is assessing a patient's risk for sensorineural hearing loss. Which of the following places the patient at greatest risk? a) Impacted earwax b) Nystagmus c) Chronic noise exposure d) Otitis media

Chronic noise exposure Chronic exposure to noise is a risk factor for sensorineural hearing losses. Impacted earwax and otitis media are risk factors for conductive hearing loss.

Which statements concerning endocrinology laboratory testing are true? Select all that apply.

Client education regarding endocrine testing must include specific test requirements. Endocrine tests are generally sensitive to storage requirements.

One of the causes of conductive hearing loss is: A) Sudden loud noise B) Ototoxic medication C) Auditory nerve damage D) Excess middle ear fluid

D) Excess middle ear fluid

The nurse is educating a client diagnosed with Meniere disease on lifestyle modifications to reduce the frequency and/or intensity of exacerbations. Which should the nurse include?

Decrease dietary intake of sodium Explanation: A lifestyle change that would be recommended for a client with Meniere disease is to consume a low sodium diet, related to the medication treatment (diuretics). The other modifications are not relevant to Meniere disease.

A patient experiences an increase in thyroid hormone as a result of a thyroid tumor. Which of the following hormonal responses demonstrates the negative feedback mechanism?

Decreased thyroid-stimulating hormone (TSH) Explanation: Negative feedback occurs when secretion of one hormone causes a reduction in the secretion of the hormone that stimulates production of the first hormone. In this case, TSH, which is manufactured by the anterior pituitary gland, would normally stimulate release of thyroid hormones, but with the increase of those hormones by the secreting tumor, enough thyroid hormones flood the system that there should be a reduction in TSH levels.

Which manifestation would the nurse observe when assessing a client diagnosed with a benign secreting tumor of the adrenal medulla?

Elevated catecholamine levels and tachycardia

The endocrine system is closely linked with the nervous system. What neurotransmitter can also act as a hormone?

Epinephrine

A nurse should perform which intervention for a client with Cushing's syndrome?

Explain that the client's physical changes are a result of excessive corticosteroids.

A 40-year-old man who is morbidly obese and leads a sedentary lifestyle has recently been diagnosed with type 2 diabetes. Which aspects of the man's obesity likely contributed to his new health problem?

Free fatty acids contribute to problems such as beta cell dysfunction and insulin resistance

A nurse caring for a client who has undergone an antithyroid peroxidase (anti-TPO) antibody test with elevated results should be prepared to educate the client about which disease process?

Hashimoto thyroiditis

Which statement best explains the function of hormone receptors?

Hormone receptors recognize a specific hormone and translate the hormonal signal into a cellular response.

The home care nurse is conducting client teaching with a client on corticosteroid therapy. To achieve consistency with the body's natural secretion of cortisol, when should the home care nurse instruct the client to take the corticosteroids?

In the morning between 7 AM and 8 AM

The nurse is caring for an elderly client with a suspected diagnosis of presbycusis. Select the most likely manifestation that the nurse would note.

Inability to understand words during a conversation

The nurse documents the presence of nystagmus when assessing a client. This can be interpreted as:

Involuntary eye movements that preserve eye fixation on stable objects in the visual field.

The nurse assesses a patient who has an obvious goiter. What type of deficiency does the nurse recognize is most likely the cause of this?

Iodine

The nurse is conducting a community education class on acute otitis media. Which statement by the participants indicates to the nurse that they are understanding the education?

It can be caused by a bacterial infection.

The nurse is conducting a community education class on acute otitis media. Which statement by the participants indicates to the nurse that they understand the education?

It can be caused by a bacterial infection. Explanation: The nurse determines that the participants understand the information when they state that acute otitis media can be caused by a bacterial infection. Breast-fed babies have a lower incidence of acute otitis media. The tympanic membrane appears cloudy in otitis media with effusions. Acute otitis media is not another name for swimmer's ear.

What is an advantage of assessing hormone levels through collection of a 24-hour urine?

It does not require serum collection for an accurate reading.

A patient develops hypocalcemia after thyroid surgery. Which of the following hormonal imbalances caused this complication?

Lack of parathyroid hormone Explanation: The parathyroid glands are located on the posterior side of the thyroid gland. If the parathyroid glands are removed at the same time as the thyroid the patient will experience a drop in serum calcium levels. Page 756

The nurse who works at a clinic for pediatric clients observes that a frequent reason for client visits is acute otitis media (AOM). The nurse understands which of the following structural characteristics of the ear in infants and children increase the potential for AOM?

Less efficient tensor muscle to open the eustachian tube

A nurse teaches a client with newly diagnosed hypothyroidism about the need for thyroid hormone replacement therapy to restore normal thyroid function. Which thyroid preparation is the agent of choice for thyroid hormone replacement therapy?

Levothyroxine (Synthroid)

Amblyopia, or lazy eye, occurs at a time when visual deprivation or abnormal binocular interactions occur in visual infancy. Whether amblyopia is reversible depends on what? a) The child has to be older than 5 years. b) Maturity of the visual system at time of onset c) The child has to have bilateral congenital cataracts. d) The child has to be able to wear contact lenses.

Maturity of the visual system at time of onset The reversibility of amblyopia depends on the maturity of the visual system at the time of onset and the duration of the abnormal experience. The other answers are incorrect

Causes of eyelid weakness include neurologic causes. There can be damage to the cranial nerves that innervate the eyelids, or there can be damage to the central nuclei of the cranial nerves. Where are the central nuclei of cranial nerve (CN) III (oculomotor nerve) and CN VII (facial nerve)?

Midbrain and caudal pons

Causes of eyelid weakness include neurologic causes. There can be damage to the cranial nerves (CN) that innervate the eyelids, or there can be damage to the central nuclei of the cranial nerves. Where are the central nuclei of cranial nerve III (oculomotor nerve) and CN VII (facial nerve)?

Midbrain and caudal pons Explanation: Neurologic causes of eyelid weakness include damage to the innervating cranial nerves or to the nerve's central nuclei in the midbrain and the caudal pons. The other answers are not related to the central nuclei of the oculomotor and the facial nerves.

A patient with a history of hypothyroidism is admitted to the intensive care unit unconscious and with a temperature of 95.2ºF. A family member informs the nurse that the patient has not taken thyroid medication in over 2 months. What does the nurse suspect that these findings indicate?

Myxedema coma

A client reports feeling like the room is spinning around him when standing still. Which diagnosis is this client likely experiencing?

Objective vertigo Explanation: Objective vertigo is the sensation of the person being stationary and the environment in motion. Subjective vertigo is a sensation of a person who may be in motion and the environment stationary. Nystagmus refers to the involuntary rhythmic and oscillatory eye movements that preserve eye fixation on stable objects in the visual field during angular and rotational movements of the head. Syncope is "fainting."

Stepping out of a mall and into the sunshine has caused a man's pupils to constrict. Place the following anatomical components of the man's pupillary reflex in the ascending chronological order that they responded to the light. Use all the options.

Oculomotor nuclei Retinal ganglionic cells Preganglionic neurons Pretectal nuclei

A patient has glaucoma. The nurse is taking a health history and knows that the most common form of glaucoma is which type?

Open angle.

A nursing instructor is teaching a group of students about the action of hormones. The instructor determined that teaching was effective when the students recognize the local action of hormones as:

Paracrine

A nursing instructor is teaching a group of students about the action of hormones. The instructor determined that teaching was effective when the students' recognize the local action of hormones as:

Paracrine

A nursing instructor is teaching a group of students about the action of hormones. The instructor determined that teaching was effective when the students' recognize the local action of hormones as: Paracrine Hormonal Pancreatic Autocrine

Paracrine

Hormones that cause the subsequent stimulation or release of another hormone, such as thyroid-stimulating hormone, typically belong to which structural classification?

Peptide proteins and glycoproteins

Which gland is often referred to as the master gland because it secretes many hormones?

Pituitary

What are the hallmark signs of diabetes mellitus? a) Polyuria, polyphagia, and polycythemia b) Polycythemia, polydipsia, and pheochromocytoma c) Polyuria, polydipsia, and polyphagia d) Polyuria, polydipsia, and pheochromocytoma

Polyuria, polydipsia, and polyphagia The most commonly identified signs and symptoms of diabetes are referred to as the three polys: (1) polyuria (i.e., excessive urination), (2) polydipsia (i.e., excessive thirst), and (3) polyphagia (i.e., excessive hunger). Pheochromocytoma and polycythemia are not hallmark signs of diabetes mellitus.

Select the most accurate statement regarding measurements of urinary hormone:

Provide a better measure of hormone levels during a designated period. Explanation: Measurements of urinary hormone or hormone metabolite excretion often are done on a 24-hour urine sample and provide a better measure of hormone levels during that period than hormones measured in an isolated blood sample. The advantages are relative ease of obtaining urine samples and blood sampling is not required. The disadvantages are that timed urine collections often are difficult to obtain and urine samples may be accidentally discarded or inaccurately preserved; drugs or disease states that alter hormone metabolism may interfere with the test results.

The spouse of a client with a history of chronic alcohol abuse tells the nurse that her spouse has no difficulty speaking and responds when spoken to but is having difficulty understanding what is being said. The spouse asks if the nurse believes the client needs a hearing aid. The nurse explains to the spouse that the client probably does not require a hearing aid but may be suffering from which of the following?

Receptive aphasia

Which diagnostic test results indicate a client has a vestibular disturbance?

Romberg test response shows increased swaying with arm drift to one side. Explanation: During a Romberg test the client closes the eyes while standing with feet together and arms extended. A vestibular disorder will produce increased swaying, and arm drift toward the affected side. A caloric stimulation test instills ice water into the ear to trigger the vestibular system. The eyes should have a slow nystagmus toward the irrigated side for 2 to 3 minutes followed by rapid movement away from the irrigated side. A client should not have nystagmus when focusing on a steady object. The Dix-Hallpike maneuver is commonly used to stimulate benign positional vertigo. The test is considered normal if the client does not experience nystagmus or vertigo.

A client diagnosed with ulcerative keratitis of the cornea asks the nurse what could have caused this type of keratitis. What are common causes of ulcerative keratitis? Select all that apply.

S. aureus, S. pneumoniae, C. trachomatis Exposure trauma Extended-wear contact lenses Explanation: Ulcerative keratitis is an inflammatory process in which parts of the epithelium, stroma, or both are destroyed. Causes of ulcerative keratitis include infectious agents such as those causing conjunctivitis (e.g., S. aureus, S. pneumoniae, C. trachomatis), exposure trauma, and use of extended-wear contact lenses (i.e., Pseudomonas keratitis and Acanthamoeba keratitis).

Following destruction of the pituitary gland, ACTH stimulation stops. Without ACTH to stimulate the adrenal glands, the adrenals' production of cortisol drops. This is an example of which type of endocrine disorder? a) Somatic b) Tertiary c) Secondary d) Primary

Secondary In secondary disorders of endocrine function, the target gland is essentially normal, but defective levels of stimulating hormones or releasing factors from the pituitary system alter its function.

A client with diabetes mellitus is reporting burning pain of the feet and some numbness. These symptoms are likely due to which cause?

Somatic neuropathy

Research has identified a cycle of insulin-induced posthypoglycemic episodes. What is this phenomenon called?

Somogyi effect

The nurse conducting a class on treatment of eye disorders may include that Botox injections may sometimes be used to treat which of the following eye disorders?

Stabismus

A 21 year-old female is suspected of having inadequate function of her hypothalamic-pituitary-thyroid system. Her care provider is planning to inject thyrotropin-releasing hormone (TRH) and then measure her levels of TSH. Which of the following diagnostic tests is being performed?

Stimulation test Explanation: A stimulation test involves the introduction of an element that stimulates the production of another factor or hormone followed by measurement of that hormone. This is not the case in a suppression test, RIA test of metabolite excretion test. Page 764

The physician suspects a client may be experiencing hypofunction of an endocrine organ. Select the most appropriate test to determine organ function.

Stimulation tests

A young child is flying on a plane for the first time. As the plane begins its decent for landing, his ears begin to hurt. The flight attendant sees that he is in pain and tells him to swallow until the pain goes away. Which structure is the child using to equalize the pressure and relieve the pain?

Swallowing pulls air through the eustachian tubes, allowing the air pressure to equalize in the inner ear.

A client with otosclerosis had a stapedectomy with reconstruction using a prosthesis. What is an expected outcome?

The client is able to hear quiet sounds from a distance. Explanation: Clients with otosclerosis tend to speak softly as the conduction of sound through bone remains intact while nerve conduction of sound decreases. Otosclerosis can place pressure on the vestibulocochlear nerve, creating tinnitus, vertigo, and sensorineural hearing loss. A positive outcome of the surgery would be improved hearing as a result of conduction of sound transmitted via the restored stapes to the tympanic membrane.

A client is managing his diabetes with exercise and diet. The health care provider reviews the client's most recent lab results: fasting blood sugar level at 80 mg/dL and a hemoglobin A1C of 5%. Select the response that best identifies the client. a) The client is at risk for developing hyperglycemia. b) The client is at risk for an insulin reaction. c) The client is achieving normal glycemic control. d) The client needs to modify his diet related to the low readings.

The client is achieving normal glycemic control. The reading for the fasting blood sugar is appropriate, and the hemoglobin A1C level estimates good control of glucose levels over a 6- to 12-week period. This client should continue his exercise and diet routine as planned.

The nurse is teaching a client who has been newly diagnosed with hypothyroidism about the function of the thyroid. Which of these does the nurse explain to the client is the role of the thyroid gland?

The thyroid gland is responsible for increasing the metabolic rate.

The nurse is teaching a client who has been newly diagnosed with hypothyroidism about the function of the thyroid. Which statement about the role of the thyroid gland is most accurate?

The thyroid gland is responsible for increasing the metabolic rate.

A client is returned to his room after a subtotal thyroidectomy. Which piece of equipment is most important for the nurse to keep at the client's bedside?

Tracheostomy set

The nurse is educating a patient diagnosed with Meniere's disease on lifestyle modifications to reduce the frequency and/or intensity of exacerbations. Which should the nurse include?

Decrease dietary intake of sodium.

Sometimes the measurement of hormones is done through a urine sample. What is an advantage of measuring hormone levels through a urine sample rather than a blood sample?

Urine samples are easily obtaine Explanation: The advantages of a urine test include the relative ease of obtaining urine samples and the fact that blood sampling is not required. The other answers are not true. Page 764

The nurse understands that movement of otoliths may result in which of the following?

Vertigo and nystagmus

A patient is having difficulty with balance. The nurse understands that the area of the ear that impacts balance is which one of the following? a) Cochlea b) Malleus c) Tympanic membrane d) Vestibular apparatus

Vestibular apparatus The vestibular system maintains and assists recover of stable body and head position and balance through control of postural reflexes. The vestibular system includes the three semicircular canals.

A patient is having difficulty with balance. The nurse understands that the area of the ear that impacts balance is which one of the following?

Vestibular apparatus.

To monitor for complications in a patient with type 2 diabetes, which tests will the nurse in the diabetic clinic schedule at least annually (select all that apply)? a. Chest x-ray b. Blood pressure c. Serum creatinine d. Urine for microalbuminuria e. Complete blood count (CBC) f. Monofilament testing of the foot

b, c, d, f

A 26-year-old patient with diabetes rides a bicycle to and from work every day. Which site should the nurse teach the patient to administer the morning insulin? a. thigh. b. buttock. c. abdomen. d. upper arm.

c. abdomen.

A 28-year-old male patient with type 1 diabetes reports how he manages his exercise and glucose control. Which behavior indicates that the nurse should implement additional teaching? a. The patient always carries hard candies when engaging in exercise. b. The patient goes for a vigorous walk when his glucose is 200 mg/dL. c. The patient has a peanut butter sandwich before going for a bicycle ride. d. The patient increases daily exercise when ketones are present in the urine.

d. The patient increases daily exercise when ketones are present in the urine.

A client, who travels frequently for work, reports intense ear pain during ascent from and descent into airports. The health care provider will recommend which category of medications to help alleviate this symptom?

decongestant nasal spary such as phenylephrine Explanation: Barotrauma most often occurs in people who travel while suffering from an upper respiratory tract infection. Decongestants, such as nose drops or nasal sprays, may be used 30 to 60 minutes prior to ascent or descent to reduce congestion and open the eustachian tubes. Steroids are not recommended for barotrauma. They are helpful for inflammation and nasal polyps. Antihistamines are helpful for tinnitus and vertigo.

A nurse is assessing a client with possible Cushing's syndrome. In a client with Cushing's syndrome, the nurse expects to find:

deposits of adipose tissue in the trunk and dorsocervical area.

51. prolapsed, downward displacement a. Acous/o k. kerat/o b. Blephar/o l. myring/o c. Chromat/o m. ophthalm/o d. Conjunctiv/o n. ot/o e. Choroid/o o. salping/o f. Core/o p. -metry g. Corne/o q. -opsia h. Dacry/o r. -ptosis i. dipl/o s. -rrhea j. irid/o t. -tropia

r. -ptosis

A client who experiences constant cloudiness of vision, sees floaters, and has an opaque lens is diagnosed with a cataract. The most appropriate treatment would be:

surgical lens replacement to correct vision. Explanation: Although strong bifocal lenses, magnification, appropriate lighting, and visual aids may help, surgery is the only treatment for cataract.

An 83-year-old client presents to the outpatient clinic with complaint of external ear pain. The ear is red, swollen, and tender to touch. What factors are likely contributors to this condition? Select all that apply.

• Hearing aid • Glasses

The three layers of the cornea are separated by two important basement membranes. What are characteristics of the Descemet membrane?

• Regenerates readily after injury • Has a felt-like appearance

A patient is having difficulty with balance. The nurse understands that the area of the ear that impacts balance is which one of the following?

Vestibular apparatus

After having a very stressful day in pathophysiology class, the student knows that which hormone (secreted by the adrenal cortex) will help decrease the effects of stress?

Cortisol, a glucocorticoid

After having a very stressful day in pathophysiology class, the student knows that which hormone (secreted by the adrenal cortex) will help decrease the effects of stress?

Cortisol, a glucocorticoid Explanation: Glucocorticoids, mainly cortisol affects metabolism of all nutrients; regulates blood glucose levels, affects growth, has antiinflammatory action, and decreases effects of stress. The other hormones do not affect stress levels. Page 756

As the eyes rotate upward, the upper eyelid reflexively retracts. Which cranial nerve is primarily responsible for this response?

Cranial nerve III.

The newborn-nursery nurse is obtaining a blood sample to determine if a newborn has congenital hypothyroidism. What long-term complication is the nurse aware can occur if this test is not performed and the infant has congenital hypothyroidism? a) Dehydration from diarrhea b) Accelerated growth c) Cretinism d) Irritability and restlessness

Cretinism Congenital hypothyroidism is a common cause of preventable mental retardation. It affects approximately 1 in 4000 infants. The manifestations of untreated congenital hypothyroidism are referred to as cretinism. The term does not apply to the normally developing infant in whom replacement thyroid hormones therapy was instituted shortly after birth.

The nurse understands that autonomic manifestations of vertigo may include which of the following? Select all that apply.

• Tachycardia • Hypotension

21. What is the contractile opening in the center of the iris through which light rays enter the eye? a. Choroid b. Pupil c. Sclera d. Conjunctiva e. Iris

B. Pupil

27. A person with dacryadenalgia has pain in the: a. Middle ear b. Tear gland c. Eardrum d. Retina e. Eyelid

B. Tear gland

The nurse is working at a first aid station in an amusement park. A 45-year-old client arrives reporting severe dizziness after a ride on the roller coaster. The nurse understands that a common cause of this sensation is which of the following?

Benign paroxysmal positional vertigo

The nurse is working at a first aid station in an amusement park. A 45-year-old client arrives reporting severe dizziness after a ride on the roller coaster. The nurse understands that a common cause of this sensation is:

Benign paroxysmal positional vertigo Explanation: Benign paroxysmal positional vertigo is the most common cause of pathologic vertigo and usually develops after the fourth decade of life. It commonly occurs when the person is getting in and out of bed, bending over and straightening up, or extending the head to look up. It also can be triggered by amusement rides that feature turns and twists. The other conditions are not manifested by severe sudden vertigo.

The nurse is working at a first aid station in an amusement park. A 45-year-old client arrives reporting severe dizziness after a ride on the roller coaster. The nurse understands that a common cause of this sensation is which of the following? a) Barotrauma b) Acute otitis media c) Benign paroxysmal positional vertigo d) Otosclerosis

Benign paroxysmal positional vertigo Benign paroxysmal positional vertigo is the most common cause of pathologic vertigo and usually develops after the fourth decade of life. It commonly occurs when the person is getting in and out of bed, bending over and straightening up, or extending the head to look up. It also can be triggered by amusement rides that feature turns and twists. The other conditions are not manifested by severe sudden vertigo.

To prevent the accumulation of hormones in our bodies, the hormones are constantly being metabolized and excreted. Where are adrenal and gonadal steroid hormones excreted? a) Bile and urine b) Bile and lungs c) Cell metabolites and lungs d) Feces and urine

Bile and urine Unbound adrenal and gonadal steroid hormones are conjugated in the liver, which renders them inactive, and then excreted in the bile or urine. Adrenal and gonadal steroid hormones are not excreted in the feces, cell metabolites, or the lungs.

To prevent the accumulation of hormones in our bodies, the hormones are constantly being metabolized and excreted. Where are adrenal and gonadal steroid hormones excreted?

Bile and urine Explanation: Unbound adrenal and gonadal steroid hormones are conjugated in the liver, which renders them inactive, and then excreted in the bile or urine. Adrenal and gonadal steroid hormones are not excreted in the feces, cell metabolites, or the lungs.

The nurse and nursing student are caring for a client undergoing a severe stressor with release of epinephrine into the bloodstream. Which of these effects on blood glucose levels does the nurse teach the student epinephrine will cause? a) Gluconeogenesis will occur. b) Blood glucose will elevate. c) Hypoglycemia will occur. d) An unusable form of glucose will be released.

Blood glucose will elevate. Epinephrine, a catecholamine, helps to maintain blood glucose levels during periods of stress. Epinephrine causes glycogenolysis in the liver, thus causing large quantities of glucose to be released into the blood.

A patient is suspected of having a pheochromocytoma and is having diagnostic tests done in the hospital. What symptoms does the nurse recognize as most significant for a patient with this disorder?

Blood pressure varying between 120/86 and 240/130 mm Hg

The diabetes nurse educator is teaching a community education class for new diabetics. Which of these does the nurse include in the discussion of signs and symptoms of hyperglycemia? Select all that apply.

Blurred vision Weight loss Thirst

When caring for the client with diabetic ketoacidosis, the nurse recognizes that fatty acids and ketones may be used for energy by most organs. Which of these organs does the nurse recognize is reliant on glucose as the major energy source? a) Lungs b) Spleen c) Brain d) Heart

Brain Although many tissues and organ systems are able to use other forms of fuel, such as fatty acids and ketones, the brain and nervous system rely almost exclusively on glucose as a fuel source. Because the brain can neither synthesize nor store more than a few minutes' supply of glucose, normal cerebral function requires a continuous supply from the circulation

What is the most common mechanism of hormone control? Negative feedback Positive feedback Hypothalamic-pituitary-target cell feedback Hypothalamic-pituitary-adrenal axis

Negative feedback

The hormone levels in the body need to be kept within an appropriate range. How is this accomplished for many of the hormones in the body?

Negative feedback loop

Which of the following hormones are derivatives of cholesterol? A) Epinephrine and norepinephrine B) Insulin and glucagon C) Aldosterone and testosterone D) Eicosanoids and retinoids

C

The nurse is reviewing the test results of a client who was given thyrotropin-releasing hormone (TRH) to evaluate the function of the pituitary gland. The nurse would recognize pituitary dysfunction as:

Decreased TSH levels

Which of the following are examples of amines? Select all that apply.

Epinephrine Norepinephrine

A nurse educator is conducting a course for newly diagnosed diabetes clients. Which statement by a participant should the nurse follow up first?

"I've had a little sore on the sole of my foot for a few days, but I'm sure it will eventually heal."

The nurse is describing to a new mother the health screening actions that will be performed on her infant. The client states, "I'm sure my baby's exhausted. I'd prefer to do this in a few weeks." What rationale for early hearing testing should the nurse describe?

"If there is a hearing problem, it can be addressed right away before your baby starts to adapt to it." Explanation: Early hearing testing occurs so that if there is a hearing problem, it can be addressed before the child begins to acclimate to its life. Stating that the testing is easy does not address the woman's concerns. Hearing deficits in newborns are not typically the result of fluid or of trauma during birth.

The mother of a 2-year-old newly diagnosed with type 1 diabetes asks why insulin has to be given by injection. The best response by the nurse is: a) "When your child gets old enough, you will not have to administer injections." b) "Your child is not old enough to swallow the pills needed to treat her diabetes." c) "Insulin is destroyed by the stomach contents and has to be administered by injection." d) "Insulin needs to go directly into the vein to work best."

"Insulin is destroyed by the stomach contents and has to be administered by injection." Insulin is destroyed by the gastrointestinal tract and needs to be administered via injection or inhalation. Type 1 diabetes is not treated with oral medications at this time. Insulin is administered subcutaneously rather than in the vein, and the final statement does not address the mother's concerns.

A 69-year-old client comes to the clinic for a routine checkup. Upon examination, the nurse practitioner informs the client that she has cataracts. The client then tells the nurse that she already knew that and her physician told her that she could use bifocals and that would take care of the problem. What would be the best response by the nurse practitioner?

"Surgery is the only effective treatment for cataracts." Explanation: There is no effective medical treatment for cataract. Strong bifocal lenses, magnification, appropriate lighting, and visual aids may be used as the cataract progresses, but surgery is the only treatment for correcting cataract-related vision loss. Telling the client that bifocals will cure cataracts is false as well as telling her that her doctor was correct. The other option is not a therapeutic response by the nurse.

A 69-year-old patient comes to the clinic for a routine checkup. Upon examination the nurse practitioner informs the patient that she has cataracts. The patient then tells the nurse that she already knew that and her physician told her that she could use bifocals and that would take care of the problem. What would be the best response by the nurse practitioner?

"Surgery is the only effective treatment for cataracts." Correct

A three year-old girl has just been diagnosed with type 1A diabetes. Her parents are currently receiving education from the diabetes education nurse. How can the nurse best explain to the parents the etiology (cause) of their daughter's diabetes? a) "The problem that underlies her diabetes is that her own body has destroyed the cells in her pancreas that produce insulin." b) "It's not known exactly why your daughter has completely stopped making insulin, and treatment will consist of your rigidly controlling her diet." c) "This tendency to produce insufficient amounts of insulin is likely something that she inherited." d) "Environmental and lifestyle factors are known to play a part in the fact that her pancreas secretes and withholds insulin at the wrong times."

"The problem that underlies her diabetes is that her own body has destroyed the cells in her pancreas that produce insulin." Type 1A, or immune-mediated, diabetes involves the autoimmune destruction of pancreatic beta cells and a consequent absolute lack of insulin. Exogenous insulin is required as dietary control alone is insufficient. The central problem is an absolute lack of insulin production rather than deranged release.

A client with a diagnosis of benign paroxysmal positional vertigo (BPPV) is receiving teaching from her physician about her diagnosis. The client is eager to avoid future episodes of vertigo and has asked the physician what she can do to prevent future episodes. How can the physician best respond?

"There are some exercises that I'll teach you to help reorient your inner ear and prevent vertigo." Explanation: Nondrug therapies for BPPV using habituation exercises and canalith repositioning are successful in many people. Canalith repositioning involves a series of maneuvers in which the head is moved to different positions in an effort to reposition the free-floating debris in the endolymph of the semicircular canals. Surgery is not a noted treatment option and even in the absence of hearing loss, treatment is warranted.

Which of the following hormones cross cell membranes to connect to nuclear receptors? Select all that apply.

-Glucocorticoids -Thyroid hormones -Vitamin D Explanation: Steroids, Vitamin D, and thyroid hormone are fat soluble and they can cross the cell membrane where they connect to intracellular receptors that move to the nucleus and bind with receptors on the target gene. Page 761

A nurse who works in the office of an endocrinologist is orienting a new staff member. Which of the following teaching points is the nurse justified in including in the orientation? Select all that apply. A) "A bodily process can be the result of the combined effect of several different hormones from different sources." B) "A single hormone can act not only on one process or organ but often on several different locations or processes." C) "It's common for production of hormones to be far removed from the tissue where they ultimately exert their effect." D) "Sometimes hormones act locally on the area where they were produced, like in the case of paracrine and autocrine actions." E) "The regulation in homeostasis requires that hormones be absent from the body when their effect is not needed."

A B C D

Which of the following individuals would be considered at high risk for developing cataracts? Select all that apply. A) An 88-year-old female with osteoporosis and congestive heart failure B) A 51-year-old female whose rheumatoid arthritis is controlled with oral corticosteroids C) A 50-year-old male who takes nebulized bronchodilators four times daily for the management of his emphysema D) A 39-year-old woman with a history of open-angle glaucoma and poorly controlled diabetes E) A 29-year-old artist who spends long hours in sunlight painting landscapes

A B D E

The mother of 6-year-old male and female fraternal twins has brought her son to see a pediatrician because he is nearly 4 inches shorter than his sister. Which of the following phenomena would the physician most likely suspect as contributing factor to the boy's short stature? a) Genetic short stature b) Lack of IGF receptors in epiphyseal long bones c) A shortage of hypothalamic GHRH production d) Excess insulin production resulting in chronically low blood glucose levels

A shortage of hypothalamic GHRH production Inadequate levels of hypothalamic GHRH will result in adequate production but inadequate release of GH by the pituitary. Genetic short stature is less likely given the disparity between his height and his twin's, and a shortage of IGF receptors is not a noted pathology. While poorly controlled diabetes can contribute to short stature, excess insulin production is not a likely factor.

35. The vascular layer of the eye that contains rods and cones is the: a. Retina b. Cornea c. Sclera d. Choroid e. Conjunctiva

A. Retina

The nurse is caring for a 42-year-old male client who is admitted for treatment of heart failure. He has abnormally large hands and feet and a broad face with a protruding jaw. Based on these signs and symptoms, the nurse identifies which of the following endocrine disturbances as the most likely cause for these physical changes? a) Hyperthyroidism b) Myxedema c) Acromegaly d) Cushing syndrome

Acromegaly Enlargement of the small bones of the hands and feet and of the membranous bones of the face and skull results in a pronounced enlargement of the hands and feet, a broad and bulbous nose, a protruding jaw, and a slanting forehead. Bone overgrowth often leads to arthralgias and degenerative arthritis of the spine, hips, and knees. Virtually every organ of the body is increased in size. Enlargement of the heart and accelerated atherosclerosis may lead to an early death. Hyperthyroidism results from excess thyroid hormone. Myxedema and Cushing syndrome are the result of adrenal abnormalities and do not cause these bone changes.

Cyclic adenosine monophosphate (cAMP) performs which role in the functioning of the endocrine system?

Acting as a second messenger to mediate hormone action on target cells

Paracrine action involves which characteristic?

Action locally on cells other than those that produce the hormone

The nurse is assessing a client with thyrotoxicosis and the nurse is explaining how the thyroid gland is stimulated to release thyroid hormones. The nurse should describe what process?

Action of releasing hormones from hypothalamus

Hormones exert their action by interacting with which type of receptors?

Affinity

Hormones are usually divided into categories according to their structure. The release of epinephrine would be classified as:

Amines and amino acids

A client has developed a tumor of the posterior pituitary gland. The client is at risk for problems with secretions of:

Antidiuretic hormone (ADH) and oxytocin Explanation: The posterior pituitary secretes ADH and oxytocin/vasopressin, while the anterior pituitary secretes the hormones listed in the other choices.

When hormones act locally rather than being secreted into the bloodstream, their actions are termed:

Autocrine and paracrine

12. Which disorder involves a reduction or dimness of vision with no apparent pathological condition? a. Heteropia b. Amblyopia c. Hyperopia d. Glaucoma e. Cataract disease

B. Amblyopia

46. Which condition is commonly called pinkeye? a. Coreitis b. Conjunctivitis c. Lacrimation d. Scleritis e. Anisocoria

B. Conjunctivitis

5. What is retinal damage marked by aneurismal dilation of blood vessels caused by? a. Strabismus b. Diabetic retinopathy c. Glaucoma d. Exothropia e. Astigmatism

B. Diabetic retinopathy

Which classification of medication does the nurse prepare to administer to the client with hyperthyroidism that will block the effects of the hyperthyroid state on sympathetic nervous system function? a) Beta-adrenergic blocking agent b) Angiotensin receptor blocking agent c) Calcium channel blocking agent d) Angiotensin converting enzyme inhibitor

Beta-adrenergic blocking agent The beta-adrenergic blocking drugs (propranolol, metoprolol, atenolol, and nadolol are preferred) are administered to block the effects of the hyperthyroid state on sympathetic nervous system function. They are given in conjunction with antithyroid drugs such as propylthiouracil and methimazole.

To prevent the accumulation of hormones in our bodies, the hormones are constantly being metabolized and excreted. Where are adrenal and gonadal steroid hormones excreted?

Bile and urine

Which pregnant female is at risk of having an infant born with congenital cataracts? A) A first-time mother over the age of 35 B) A pregnant mother with baby number 2 on the way who tested positive for human papillomavirus (HPV) C) A diabetic mother who regulates her blood glucose levels with insulin D) A mother who is 34 weeks' pregnant diagnosed with preeclampsia

C) A diabetic mother who regulates her blood glucose levels with insulin

In comparison to children with acute otitis media (AOM), those with otitis media with effusion (OME) have: A) Systemic infection B) Earache and fever C) Excess middle ear fluid D) Sensorineural hearing loss

C) Excess middle ear fluid

The nurse is assisting a client who is undergoing the Romberg test. The nurse understands that the client will be asked to do which of the following?

Close the eyes.

Which of the following criteria about insulin would prompt a diagnosis of type 1 diabetes? a) Insulin not efficiently used b) Small amounts of insulin secreted c) Large amounts of insulin secreted d) Complete failure of insulin secretion

Complete failure of insulin secretion In type 1 diabetes there is an absolute lack of insulin due to complete failure of the pancreas. In type 2 diabetes some insulin is produced but may not be properly used.

A patient's chart documents the finding of cholesteatoma. The nurse interprets this to mean that the patient has which of the following?

Cystlike mass in the middle ear

A patient's chart documents the finding of cholesteatoma. The nurse interprets this to mean that the patient has which of the following?

Cystlike mass in the middle ear.

The nurse is reviewing the test results of a client who was given thyrotropin-releasing hormone (TRH) to evaluate the function of the pituitary gland. The nurse would recognize pituitary dysfunction as:

Decreased TSH levels Explanation: Stimulation tests are used when hypofunction of an endocrine organ is suspected. Failure to increase TSH levels after a TRH stimulation test suggests an inadequate capacity to produce TSH by the pituitary (i.e., the pituitary is dysfunctional in some way). The other tests results would not increase. Page 756

A client experiences an increase in cortisol as a result of Cushing disease. Which hormonal responses demonstrate the negative feedback mechanism?

Decreased adrenocorticotropic hormone (ACTH)

A nurse is planning care for a client in acute addisonian crisis. Which nursing diagnosis should receive the highest priority?

Decreased cardiac output

The nurse is caring for a client who has been diagnosed with presbycusis. The nurse understands that with this disorder, the client experiences which of the following?

Difficulty hearing high-pitched frequencies Explanation: Presbycusis is used to describe degenerative hearing loss that occurs with advancing age. It is characterized by high-frequency hearing loss.

The nurse is caring for a client who has been diagnosed with presbycusis. The nurse understands that with this disorder, the client experiences which of the following? a) Difficulty hearing high-pitched frequencies b) Chronic ear drainage c) Severe pain in the mastoid area d) Fluid in the inner ear without infection

Difficulty hearing high-pitched frequencies Presbycusis is used to describe degenerative hearing loss that occurs with advancing age. It is characterized by high-frequency hearing loss.

23. Which term refers to the duct that collects and drains tears? a. Auricle b. Incus c. Malleus d. Choroid e. Lacrimal

E. Lacrimal

To diagnose a vestibular disorder, which test would be used that assesses the eye movements in response to vestibular, visual, and positional stimulation?

Electronystagmography Explanation: Of the options provided, the only test which records the eye movements in response to vestibular, visual, cervical, rotational and positional stimulation is the electronystagmography.

When caring for a client with anemia and a decrease in red blood cells (RBCs), the nurse recognizes which of these hormones will stimulate the bone marrow to produce additional RBCs? a) Somatostatin b) Erythropoietin c) Cortisol d) Atrial natriuretic peptide

Erythropoietin Erythropoietin is made in the kidney and stimulates erythropoiesis, production of RBCs in the bone marrow.

Which of the following is an example of a positive feedback system?

Estradiol and follicle-stimulating hormone (FSH)

Which nursing intervention is the highest priority intervention for a nurse caring for a 26-year-old client diagnosed with benign paroxysmal positional vertigo?

Fall precautions Explanation: The client with benign paroxysmal positional vertigo (BPPV) experiences brief periods of vertigo when getting in and out of bed, bending over and straightening up, and extending head to look up. This places the client at increased risk for falls and necessitates implementation of fall precautions. The Weber test and administration of an antibiotic are not indicated when caring for a client with BPPV. Skin assessment, while always a part of the nursing assessment, has a lower priority than implementation of safety measures for this client.

Which test can the nurse prepare the client for to determine the differentiation between a benign and a malignant thyroid disease? a) Ultrasonography b) Fine-needle aspiration biopsy c) MRI d) CT scan

Fine-needle aspiration biopsy Ultrasonography can be used to differentiate cystic from solid thyroid lesions, and CT and MRI scans are used to demonstrate tracheal compression or impingement on other neighboring structures. Fine-needle aspiration biopsy of a thyroid nodule has proved to be the best method for differentiation of benign from malignant thyroid disease. (less)

While discussing the regulation of hormone levels, the instructor gives an example of hormones regulated by feedback mechanisms. Which example of this regulation is best? In children, the body knows to release growth hormones while they are sleeping instead of when they are awake and running around. When a female is thinking about getting pregnant, the body knows to release female sex hormones in greater proportion than usual. When a person's body pH is decreasing, the stomach lining becomes more acidic to offset this. Following a meal that was high in carbohydrates, a person's blood glucose elevates, which stimulates the release of insulin from the pancreas.

Following a meal that was high in carbohydrates, a person's blood glucose elevates, which stimulates the release of insulin from the pancreas.

A clent has damage to the epithelial layer of the cornea. Which are clinical manifestations that can occur with damage to the epithelial layer of the cornea? Select all that apply.

Foreign body sensation Burning of the eyes Reflex lacrimation Explanation: Epithelial damage causes discomfort that ranges from a foreign body sensation and burning of the eyes to severe, incapacitating pain. Reflex lacrimation is common.

A 25-year-old female client exhibits exophthalmos (abnormal protrusion of the eyeballs) or eyeball of both eyes. The health care provider recognizes this as a manifestation of: a) Graves disease b) Myxedema c) Hashimoto thyroiditis d) Acquired hypothyroidism

Graves disease Graves disease is a state of hyperthyroidism in which opthalmopathies, such as exophthalmos, typically occur. The other conditions are states of hypothyroidism and are not associated with this abnormality.

A 62-year-old man who is overweight has just been diagnosed with type 2 diabetes. The nurse educator is instructing him in the ways his diabetes can be controlled. The nurse should initially prioritize which action?

Helping the client make meaningful changes to his diet and activity level.

An adult client is scheduled for testing of a suspected growth hormone (GH)-secreting tumor. Which result from the glucose suppression test would confirm the condition?

Increased GH secretion

A 51 year-old woman has been experiencing signs and symptoms of perimenopause and has sought help from her family physician. A deficiency in estrogen levels has been determined to be a contributing factor. Which of the following phenomena could potentially underlie the woman's health problem?

Insufficient estrogen production within the smooth endoplasmic reticulum of the relevant cells. Explanation: Steroids such as estrogen are produced in the smooth endoplasmic reticulum. Synthesis and release are not separate processes as in the case of peptide hormones and prohormones are associated with peptides, polypeptides and proteins hormones.

A client has been diagnosed with hemianopia. The client asks the nurse what this is describing concerning the eye?

Loss of half of the visual field in one eye Explanation: Blindness in one eye is called anopia. If half of the visual field for one eye is lost, the defect is called hemianopia; if a quarter of the field is lost, it is called quadrantanopia. Loss of the temporal or peripheral visual fields on both sides results in a narrow binocular field, commonly called tunnel vision.

A patient has been diagnosed with hemianopia. The patient asks the nurse what this is describing concerning the eye?

Loss of half of the visual field in one eye Blindness in one eye is called anopia. If half of the visual field for one eye is lost, the defect is called hemianopia; - if a quarter of the field is lost, it is called quadrantanopia. - Loss of the temporal or peripheral visual fields on both sides results in a narrow binocular field, commonly called tunnel vision.

Which intervention is the most critical for a client with myxedema coma?

Maintaining a patent airway

For a client with Graves' disease, which nursing intervention promotes comfort?

Maintaining room temperature in the low-normal range

A client with severe hypoglycemia is unconscious. Which method of providing glucose should be avoided? a) Orange juice orally b) Glucose gel in the buccal pocket c) IM glucagon injection d) Dextrose IV

Orange juice orally When clients are unconscious it is not safe to attempt to have them swallow liquids. Alternate routes that reduce the risk of choking such as buccal absorption, intramuscular or intravenous injections are preferred.

The nurse is assessing the hearing of a twenty-five-year-old client who reports chronic painless tinnitus and difficulty hearing. The nurse suspects the client suffers from which of the following?

Otosclerosis

A 38-year-old client who has begun to develop hearing loss is being evaluated by the nurse practitioner. The client tells the nurse that hearing improves when she is on the telephone and that chewing sounds very loud. What does the nurse practitioner suspect?

Otosclerosis Explanation: Otosclerosis involves the reabsorption of bone and replacement with new, sclerotic bone around the stapes and oval window, which slowly impedes hearing. The typical presentation is around 20 to 40 years of age. People with otosclerosis often experience an increase in the sound of chewing and can often hear on the telephone better than in person, due to the bone conduction of sound.

Conductive hearing loss can occur for a variety of reasons, including foreign bodies in the ear canal, damage to the ear drum, or disease. What disease is associated with conductive hearing loss?

Paget disease Explanation: More permanent causes of hearing loss are thickening or damage of the tympanic membrane or involvement of the bony structures (ossicles and oval window) of the middle ear due to otosclerosis or Paget disease. Huntington, Alzheimer, and Parkinson diseases are not associated with conductive hearing loss.

A nursing instructor is teaching a group of students about the action of hormones. The instructor determined that teaching was effective when the students' recognize the local action of hormones as:

Paracrine Explanation: When hormones act locally on cells other than those that produced the hormone, the action is called paracrine. Hormones also can exert an autocrine action on the cells in which they were produced. Pancreatic and hormonal are not actions. Page 754

A nursing instructor is teaching a group of students about the action of hormones. The instructor determined that teaching was effective when the students' recognize the local action of hormones as: a) Autocrine b) Hormonal c) Pancreatic d) Paracrine

Paracrine When hormones act locally on cells other than those that produced the hormone, the action is called paracrine. Hormones also can exert an autocrine action on the cells in which they were produced. Pancreatic and hormonal are not actions.

Which structure controls the functions of the greatest number of target glands and cells?

Pituitary gland

The nurse is caring for a client with a tentative diagnosis of Hashimoto thyroiditis. Which of these test results does the nurse anticipate will be present in autoimmune thyroiditis?

Positive anti-thyroid peroxidase antibodies Explanation: Blood tests that are routinely measured in endocrine disorders include various autoantibodies. For example, anti-thyroid peroxidase (anti-TPO) antibodies are measured during the initial diagnostic workup and subsequent followup of patients with Hashimoto thyroiditis. The other alterations may be present in all types of hypothyroidism.

The nurse assesses a patient who has been diagnosed with Addison's disease. Which of the following is a diagnostic sign of this disease?

Potassium of 6.0 mEq/L

A woman with poorly controlled type 1 diabetes has been admitted to a hospital unit for the treatment of ketoacidosis. Place the following events in the pathophysiology of ketoacidosis in the correct chronological order. Use all the options.

Production of fatty acids and glycerol Low serum insulin levels Ketone production by the liver Breakdown of triglycerides Decrease in pH

The nurse is caring for a client who has been diagnosed with Meniere disease. The nurse expects which of the following medications will be prescribed to manage the symptoms?

Promethazine (Phenergan)

What does the tinnitus that is caused by vascular disorders sound like?

Pulses Explanation: In some vascular disorders, sounds generated by turbulent blood flow (e.g., arterial bruits or venous hums) are conducted to the auditory system. Vascular disorders typically produce a pulsatile form of tinnitus.

Select the process that allows hormones to exert influence upon some cells and not others.

Receptor binding Explanation: Receptor binding is a process that allows target cells to respond to one hormone and not to others. For example, receptors in the thyroid are specific for thyroid-stimulating hormone, and receptors on the gonads respond to the gonadotropic hormones. Positive feedback control occurs when rising levels of a hormone cause another gland to release a hormone that is stimulating to the first. Protein binding describes the ability of proteins to form bonds with other substances. Pituitary-hypothalamic feedback describes negative feedback. Page 758

Select the process that allows hormones to exert influence upon some cells and not others.

Receptor binding Explanation: Receptor binding is a process that allows target cells to respond to one hormone and not to others. For example, receptors in the thyroid are specific for thyroid-stimulating hormone, and receptors on the gonads respond to the gonadotropic hormones. Positive feedback control occurs when rising levels of a hormone cause another gland to release a hormone that is stimulating to the first. Protein binding describes the ability of proteins to form bonds with other substances. Pituitary-hypothalamic feedback describes negative feedback.

How will cell receptors change in the absence of a particular hormone?

Receptors in each cell will increase.

The nurse is caring for a client who is receiving exogenous corticosteroids for rheumatoid arthritis. Recognizing that hormone levels are regulated by negative feedback, which laboratory test result does the nurse anticipate uncovering when reviewing the medical record?

Reduction in ACTH

Presbycusis is degenerative hearing loss associated with aging. What is the first symptom of this disorder?

Reduction in ability to understand speech.

The nurse takes a patient's blood pressure in the primary care provider's office. The last three times the patient has been in the office it has been high. What structure in the eye is this adverse effect of hypertension most likely to affect? a) Ciliary body b) Lens c) Retina d) Iris

Retina The retina is the structure in the eye that the adverse effect of hypertension most likely to affect. -The lens is affected with a cataract. -The iris and ciliary body are not impacted.

The nurse takes a patient's blood pressure in the primary care provider's office. The last three times the patient has been in the office it has been high. What structure in the eye is this adverse effect of hypertension most likely to affect?

Retina.

A college junior calls his mother, a nurse, complaining of, "not being able to see." When questioned further, he describes, "A gray curtain just went down my right visual field. I don't know what to do." The nurse should recognize this symptom as which of the following conditions and have their teenager go to the emergency department immediately.

Retinal detachment.

A 21 year-old female is suspected of having inadequate function of her hypothalamic-pituitary-thyroid system. Her care provider is planning to inject thyrotropin-releasing hormone (TRH) and then measure her levels of TSH. Which of the following diagnostic tests is being performed?

Stimulation test Explanation: A stimulation test involves the introduction of an element that stimulates the production of another factor or hormone followed by measurement of that hormone. This is not the case in a suppression test, RIA test of metabolite excretion test.

The physician suspects a client may be experiencing hypofunction of an endocrine organ. Select the most appropriate test to determine organ function. Stimulation tests Suppression tests Genetic testing Imaging studies

Stimulation tests

The physician suspects a client may be experiencing hypofunction of an endocrine organ. Select the most appropriate test to determine organ function.

Stimulation tests Explanation: Stimulation tests are used when hypofunction of an endocrine organ is suspected. Suppression tests are used when hyperfunction of an endocrine organ is suspected. Genetic testing is used for DNA analysis, and imaging may be used as a follow-up after the diagnosis. Page 764

The physician suspects a client may be experiencing hypofunction of an endocrine organ. Select the most appropriate test to determine organ function. a) Imaging studies b) Stimulation tests c) Suppression tests d) Genetic testing

Stimulation tests Stimulation tests are used when hypofunction of an endocrine organ is suspected. Suppression tests are used when hyperfunction of an endocrine organ is suspected. Genetic testing is used for DNA analysis, and imaging may be used as a follow-up after the diagnosis

The nurse is caring for a patient with hyperthyroidism who suddenly develops symptoms related to thyroid storm. What symptoms does the nurse recognize that are indicative of this emergency?

Temperature of 102ºF

A nurse is assessing a client after a thyroidectomy. The assessment reveals muscle twitching and tingling, along with numbness in the fingers, toes, and mouth area. The nurse should suspect which complication?

Tetany

A nurse working in a clinic performs screening tests for hearing loss. Which result indicates the client has hearing loss?

The client heard bone conduction longer than air conduction in the Rinne test. Explanation: A client is said to have normal hearing if they can detect sounds softer than 26 decibels. In addition to volume of sound, there is the ability to detect frequency. The human ear is most sensitive in detecting a range between 1000-3000 Hz. A Weber test compares ears. The sound of the tuning fork will lateralize to the weaker ear if there is conductive loss, and to the stronger ear if there is sensorineural loss. The Rinne test compares bone and air conduction. In conductive hearing loss, the bone conduction lasts longer than air conduction.

The nurse is caring for a client with decreased serum protein levels secondary to liver failure. When administering medications that are highly protein bound, the nurse anticipates the resulting drug level will respond in which of these ways?

The drug level will be elevated as lack of protein allows more free drug to circulate.

The nurse is preparing a pediatric client for insertion of a tympanostomy tube. The nurse will give which instruction to the parents postprocedure?

The ears must be kept out of water. Explanation: Tympanostomy tube insertion is a surgical procedure used to treat otitis media (OM). The ears of children with tubes must be kept out of water.

A client undergoing an evaluation of hormone levels asks, "What regulates the hormone levels?" Which response by the nurse would be considered most accurate?

The hypothalamic-pituitary-target cell system

A client who is referred to the endocrinologist's office for an evaluation of his hormone levels asks what regulates the hormone levels. The best response would be that hormone levels in the body are primarily regulated by:

The hypothalamic-pituitary-target cell system

A patient taking corticosteroids for exacerbation of Crohn's disease comes to the clinic and informs the nurse that he wants to stop taking them because of the increase in acne and moon face. What can the nurse educate the patient regarding these symptoms?

The moon face and acne will resolve when the medication is tapered off.

The nurse administers a glucocorticoid medication to a client with pneumonia. Which of these does the nurse teach the client is the purpose of the medication? a) To decrease appetite b) To regulate glucose metabolism c) To decrease airway inflammation d) To decrease stress of illness

To decrease airway inflammation Glucocorticoids affect metabolism of all nutrients and have anti-inflammatory effects, which can assist with airway inflammation.

The nurse administers a glucocorticoid medication to a client with pneumonia. Which of these does the nurse teach the client is the purpose of the medication?

To decrease airway inflammation Explanation: Glucocorticoids affect metabolism of all nutrients and have anti-inflammatory effects, which can assist with airway inflammation.

A 60-year-old client with a medical history of diabetes and hypertension is having a yearly physical exam, and the practitioner instructs the client to make an appointment with an ophthalmologist. The client states that there is nothing wrong with her vision. What is the best response from the practitioner to explain the purpose of a checkup by an opthamologist?

To rule out the presence of open-angle glaucoma, which often has no signs or symptoms

A client with a history of an endocrine disorder exhibits signs and symptoms of hormone deficiency. Which of the following processes would the client's care team most likely rule out first as a contributing factor?

Up-regulation has increased the sensitivity of the body to particular hormone levels.

A client with a history of an endocrine disorder exhibits signs and symptoms of hormone deficiency. Which process would the client's care team most likely rule out first as a contributing factor?

Up-regulation has increased the sensitivity of the body to particular hormone levels.

The nurse understands that movement of otoliths may result in which of the following?

Vertigo and nystagmus.

A 55-year-old female patient with type 2 diabetes has a nursing diagnosis of imbalanced nutrition: more than body requirements. Which goal is most important for this patient? a. The patient will reach a glycosylated hemoglobin level of less than 7%. b. The patient will follow a diet and exercise plan that results in weight loss. c. The patient will choose a diet that distributes calories throughout the day. d.The patient will state the reasons for eliminating simple sugars in the diet.

a. The patient will reach a glycosylated hemoglobin level of less than 7%.

A 38-year-old patient who has type 1 diabetes plans to swim laps daily at 1:00 PM. The clinic nurse will plan to teach the patient to a. check glucose level before, during, and after swimming. b. delay eating the noon meal until after the swimming class. c. increase the morning dose of neutral protamine Hagedorn (NPH) insulin. d. time the morning insulin injection so that the peak occurs while swimming.

a. check glucose level before, during, and after swimming

The nurse is teaching a client about the treatment of open-angle glaucoma. The most appropriate information for the nurse to give the client would be:

administration of topical beta-adrenergic antagonists to lower the pressure. Explanation: The elevation in intraocular pressure in persons with open-angle glaucoma is usually treated pharmacologically or, in cases where pharmacologic treatment fails, by increasing aqueous outflow through a surgically created pathway. Medication is generally administered topically.

A 42-year-old client has sought care because he reports increasing difficulty reading the labels on packages and other small fonts. He states that he has to continually hold it further and further away from his face in order to see the type clearly. The client is most likely experiencing:

an age-related change in accommodation.

While reviewing the major actions of follicle-stimulating hormone (FSH), the faculty points out that in males this hormone is responsible for the:

sperm production.

When a patient with type 2 diabetes is admitted for a cholecystectomy, which nursing action can the nurse delegate to a licensed practical/vocational nurse (LPN/LVN)? a. Communicate the blood glucose level and insulin dose to the circulating nurse in surgery. b. Discuss the reason for the use of insulin therapy during the immediate postoperative period. c. Administer the prescribed lispro (Humalog) insulin before transporting the patient to surgery. d. Plan strategies to minimize the risk for hypoglycemia or hyperglycemia during the postoperative period.

c. Administer the prescribed lispro (Humalog) insulin before transporting the patient to surgery.

Which finding indicates a need to contact the health care provider before the nurse administers metformin (Glucophage)? a. The patient's blood glucose level is 174 mg/dL. b. The patient has gained 2 lb (0.9 kg) since yesterday. c. The patient is scheduled for a chest x-ray in an hour. d. The patient's blood urea nitrogen (BUN) level is 52 mg/dL.

d. The patient's blood urea nitrogen (BUN) level is 52 mg/dL

85. test that measures hearing acuity at various sound frequencies a. achromatopsia k. macular degeneration b. anacusis l. meniere disease c. astigmatism m. myringoplasty d. audiometry n. myringotomy e. cataract o. otoscopy f. diabetic retinopathy p. photophobia g. exotropia q. rinne test h. glaucoma r. strabismus i. hordeolum s. tinnitus j. iridoplegia t. vertigo

d. audiometry

66. Conjunctiva a. Acous/o k. kerat/o b. Blephar/o l. myring/o c. Chromat/o m. ophthalm/o d. Conjunctiv/o n. ot/o e. Choroid/o o. salping/o f. Core/o p. -metry g. Corne/o q. -opsia h. Dacry/o r. -ptosis i. dipl/o s. -rrhea j. irid/o t. -tropia

d. conjunctiv/o

73. opacity (cloudiness) of the lens as a result of protein deposits on its surface that slowly build up until vision is lost a. achromatopsia k. macular degeneration b. anacusis l. meniere disease c. astigmatism m. myringoplasty d. audiometry n. myringotomy e. cataract o. otoscopy f. diabetic retinopathy p. photophobia g. exotropia q. rinne test h. glaucoma r. strabismus i. hordeolum s. tinnitus j. iridoplegia t. vertigo

e. cataract

70. choroid a. Acous/o k. kerat/o b. Blephar/o l. myring/o c. Chromat/o m. ophthalm/o d. Conjunctiv/o n. ot/o e. Choroid/o o. salping/o f. Core/o p. -metry g. Corne/o q. -opsia h. Dacry/o r. -ptosis i. dipl/o s. -rrhea j. irid/o t. -tropia

e. choroid/o

The nurse understands that vestibular rehabilitation includes:

exercises in balance retraining. Explanation: Vestibular rehabilitation has been shown to be helpful as a treatment for peripheral vestibular disorders. Physical therapists are usually involved in developing a program including habituation exercises, balance retraining exercises, and a general conditioning program for people to use at home.

The nursing student who is studying pathophysiology correctly identifies the condition that characteristically has an increase in the aqueous humor which fills the anterior and posterior chambers of the eye as which disease?

glaucoma

The nurse is examining a pediatric client who reports ear pain. As the nurse inserts the otoscope, the child begins to cough and gag. The nurse understands that this behavior is most likely related to:

stimulation of the vagus nerve. Explanation: Because of vagal innervation, the insertion of a speculum or an otoscope into the external ear canal can stimulate coughing or vomiting reflexes, particularly in young children.

A 21-year-old female is suspected of having inadequate function of her hypothalamic-pituitary-thyroid system. Her care provider is planning to inject thyrotropin-releasing hormone (TRH) and then measure her levels of TSH. Which diagnostic test is being performed?

stimulation test

A mother brings her toddler to the clinic and tells the nurse that she thinks something is wrong with the baby's eyes. Upon further assessment the nurse notices that the child has upper deviation in movement in only one eye.What should the nurse suspect is wrong with the child?

strabismus

A client with many nonspecific complaints has been ordered a positron emission tomography (PET) scanning for evaluation of:

tumors located on the endocrine glands.

In major athletic competition, athletes are required to submit to liquid chromatography testing looking for:

use of performance-enhancing agents to increase the chances of winning.

On the first obstetric clinic visit, which client is at risk for delivering a child with sensorineural hearing loss? Select all that apply.

woman working in an industrial factory where exposed to toluene, a solvent in paints, and styrene in manufacturing plastics woman with a history of renal disease who regularly takes furosemide, a loop diuretic to control blood pressure woman who is an athlete and regularly receives traumatic kicks to the torso and abdomen during soccer matches Explanation: Early in pregnancy, the embryo is particularly sensitive to toxic substances, including ototoxic drugs such as the aminoglycosides and loop diuretics. Trauma, particularly head trauma, may also cause sensorineural hearing loss. Toluene and styrene are the most widely and extensively used aromatic solvents in industry and can cause ototoxicity. Toluene is a major component of adhesives, paints, lacquers, varnishes, printing inks, degreasers, fuel additives, glues and thinners, whereas styrene is present when manufacturing plastics, rubber articles, and glass fibers. The businesswoman working long hours is at no greater risk than any pregnant female. Nurses are exposed to numerous infections, but standard precautions recommended by the CDC provides good protection.

When a client reports that he feels like he is spinning but the room is stable, the nurse should document the finding as:

Subjective vertigo Explanation: The description of feeling like one is spinning but the room is stable is a common description of subjective vertigo. The description of the person remaining fixed and the environment spinning around is a common description of objective vertigo. The other options are characterized by losing consciousness.

The nurse is caring for a client who has been diagnosed with Meniere disease. The nurse understands which of the following medications may be prescribed for diagnostic purposes? Select all that apply.

Furosemide (Lasix) Urea Glycerin

A client with hyperthyroidism is being treated with medication that blocks the activity of thyroid-stimulating hormone. Her care team has determined that she has been overproducing TSH. This client will have lost her ability to:

have negative feedback regulation.

A client tells the nurse that when she is in bed, changes position of her head, and rolls into a lateral position, she experiences brief periods of vertigo, usually lasting less than 1 minute. The client is most likely experiencing:

Benign paroxysmal positional vertigo.

A client with a diagnosis of benign paroxysmal positional vertigo (BPPV) is receiving teaching from her physician about her diagnosis. The client is eager to avoid future episodes of vertigo and has asked the physician what she can do to prevent future episodes. How can the physician best respond?

"There are some exercises that I'll teach you to help reorient your inner ear and prevent vertigo."

49. What is characteristic of the condition called tinnitus? a. Hearing loss due to old age b. Dizziness associated with nausea c. Ringing sound in the ears d. Dysphonia e. Discharge of fluid from the ears

C. Ringing sound in the ears

The most common cause of hypothyroidism is which of the following? -Autoimmune thyroiditis -Radioiodine therapy -Antithyroid medications -Thyroidectomy

-Autoimmune thyroiditis The most common cause of hypothyroidism is autoimmune thyroiditis (Hashimoto's disease), in which the immune system attacks the thyroid gland. Hypothyroidism can occur in patient with previous hyperthyroidism that has been treated with radioiodine, antithyroid medication treatment, or thyroidectomy.

Accidental removal of one or both parathyroid glands can occur during a thyroidectomy. Which of the following is used to treat tetany? -Calcium gluconate -Synthroid -Propylthiouracil (PTU) -Tapazole

-Calcium gluconate Sometimes in thyroid surgery, the parathyroid glands are removed, producing a disturbance in calcium metabolism. Tetany is usually treated with IV calcium gluconate. Synthroid is used in the treatment of hypothyroidism. PTU and Tapazole are used in the treatment of hyperthyroidism.

Surgical removal of the thyroid gland is the treatment of choice for thyroid cancer. During the immediate postoperative period, the nurse knows to evaluate serum levels of __________ to assess for a serious and primary postoperative complication of thyroidectomy.

Calcium

A client diagnosed with type 2 diabetes has been instructed about managing his condition with diet. Which statements by the client indicate a need for additional education? Select all that apply.

"I must avoid all candies and cookies, but can eat unlimited amounts of pasta and breads." "I need to carefully limit my protein consumption."

A patient is suspected of having a pheochromocytoma and is having diagnostic tests done in the hospital. What symptoms does the nurse recognize as most significant for a patient with this disorder? -Blood pressure varying between 120/86 and 240/130 mm Hg -Heart rate of 56-64 bpm -Shivering -Complaints of nausea

-Blood pressure varying between 120/86 and 240/130 mm Hg Hypertension associated with pheochromocytoma may be intermittent or persistent. Blood pressures exceeding 250/150 mm Hg have been recorded. Such blood pressure elevations are life threatening and can cause severe complications, such as cardiac dysrhythmias, dissecting aneurysm, stroke, and acute kidney failure.

A client with a pituitary adenoma has had a transsphenoidal removal, successfully. What statement made by the client after education by the nurse indicates the client understands pharmacological treatment? a) "I will have to take medication for the rest of my life." b) "They gave me all of the medication I needed in the hospital." c) "Since I have had surgery, there is nothing else I will need to take." d) "I will need to take my medication for about 6 to 12 months, or until my adrenal function returns."

"I will need to take my medication for about 6 to 12 months, or until my adrenal function returns." Transsphenoidal removal of a pituitary adenoma or a hemihypophysectomy is the preferred method of treatment for Cushing disease. This allows removal of only the tumor rather than the entire pituitary gland. After successful removal, the person must receive cortisol replacement therapy for 6 to 12 months or until adrenal function returns

The health care provider has prescribed Repaglinide 2 mg for a client diagnosed with type 2 diabetes. The most important information for the nurse to give the client would be: a) "Take the medication 15 to 30 minutes before each meal." b) "Use a different injection site for each injection." c) "Adjust the dosing based on recent blood glucose readings." d) "Keep the medication refrigerated until used."

"Take the medication 15 to 30 minutes before each meal." To avoid hypoglycemia, Repaglinide is taken 15 to 30 minutes before each meal. This is an oral medication, so there is not injection site and does not need to be refrigerated. The dosing is not based on glucose levels.

A client with hyperparathyroidism declines surgery and is to receive hormone replacement therapy with estrogen and progesterone. Which instruction is most important for the nurse to include in the client's teaching plan? -"Maintain a moderate exercise program." -"Rest as much as possible." -"Lose weight." -"Jog at least 2 miles per day."

-"Maintain a moderate exercise program." The nurse should instruct the client to maintain a moderate exercise program. Such a program helps strengthen bones and prevents the bone loss that occurs from excess parathyroid hormone. Walking or swimming provides the most beneficial exercise. Because of weakened bones, a rigorous exercise program such as jogging is contraindicated. Weight loss might be beneficial but it isn't as important as developing a moderate exercise program.

How long is the half-life of the hormone aldosterone, which is only 15% protein bound?

25 minutes

9. Following focal seizures that have damaged the dominant hemisphere of a patient's auditory association cortex, the nurse may observe the patient displaying A) receptive aphasia. B) facial drooping. C) auditory hallucinations. D) delusions of grandeur.

A Feedback: Damage to the auditory association cortex, especially if bilateral, results in deficiencies of sound recognition and memory (auditory agnosia). If the damage is in the dominant hemisphere, speech recognition can be affected (sensory or receptive aphasia). The others are not caused by focal seizures.

The nurse is reviewing assessment data and determines which client is at highest risk for developing type 2 diabetes?

A 45-year-old obese female with a sedentary lifestyle

Which of the following clients would be considered to be exhibiting manifestations of "prediabetes"? a) A middle-aged overweight adult with a fasting plasma glucose level of 122 with follow-up OGTT of 189 mg/dL. b) A school-aged child who had a blood glucose level of 115 following lunch. c) A retired female registered nurse with a fasting plasma glucose level of 92 mg/dL. d) An elderly client who got "light-headed" when he skipped his lunch. Blood glucose level was 60 mg/dL at this time.

A middle-aged overweight adult with a fasting plasma glucose level of 122 with follow-up OGTT of 189 mg/dL. Persons with IFG (impaired fasting plasma glucose [IFG] defined by an elevated FPG of 100 to 125 mg/dL) and/or IGT (impaired glucose tolerance [IGT] plasma glucose levels of 140 to 199 mg/dL with an OGTT) are often referred to as having prediabetes, meaning they are at relatively high risk for the future development of diabetes as well as cardiovascular disease.

A lung cancer client with small cell carcinoma may secrete an excess of which hormone causing an ectopic form of Cushing syndrome due to a nonpituitary tumor? a) ACTH b) DHEA c) GH d) TSH

ACTH The third form (of Cushing syndrome) is ectopic Cushing syndrome, caused by a nonpituitary ACTH-secreting tumor. Certain extra pituitary malignant tumors such as small cell carcinoma of the lung may secrete ACTH or, rarely, CRH and produce Cushing syndrome. The adrenal sex hormone dehydroepiandrosterone (DHEA) contributes to the pubertal growth of body hair, particularly pubic and axillary hair in women. Thyroid-stimulating hormone (TSH) levels are used to differentiate between primary and secondary thyroid disorders. Although secretion of growth hormone (GH) has diurnal variations over a 24-hour period, with nocturnal sleep bursts occurring 1 to 4 hours after onset of sleep, it is unrelated to ACTH and/or CRH secretion.

At times, it is necessary to give medications that suppress the adrenal glands on a long-term basis. When the suppression of the adrenals becomes chronic, the adrenal glands atrophy. What does the abrupt withdrawal of these suppressive drugs cause? a) Acute adrenal insufficiency b) Acute adrenal hyperplasia c) Acute adrenal hypoplasia d) Acute adrenal cortical hyperplasia

Acute adrenal insufficiency Chronic suppression causes atrophy of the adrenal gland, and the abrupt withdrawal of drugs can cause acute adrenal insufficiency. The other answers are incorrect.

A client reports sudden, acute left eye pain with blurred vision and a headache on the affected side. The client is most likely experiencing:

Acute angle-closure glaucoma

Stepping out of a mall and into the sunshine has caused a man's pupils to constrict. Place the following anatomical components of the man's pupillary reflex in the ascending chronological order that they responded to the light. Use all the options. A) Oculomotor nuclei B) Retinal ganglionic cells C) Preganglionic neurons D) Pretectal nuclei

B D C A

40. What structure leads from the middle ear to the nasopharynx? a. Tympanic membrane b. Eustachian tube c. Malleus d. Cochlea e. Auricle

B. Eustachian tube

The nurse is working at a first aid station in an amusement park. A 45-year-old client arrives reporting severe dizziness after a ride on the roller coaster. The nurse understands that a common cause of this sensation is which of the following?

Benign paroxysmal positional vertigo.

To prevent the accumulation of hormones in our bodies, the hormones are constantly being metabolized and excreted. Where are adrenal and gonadal steroid hormones excreted?

Bile and urine Explanation: Unbound adrenal and gonadal steroid hormones are conjugated in the liver, which renders them inactive, and then excreted in the bile or urine. Adrenal and gonadal steroid hormones are not excreted in the feces, cell metabolites, or the lungs. Page 757

Diabetic ketoacidosis (DKA) is a condition that mostly occurs in type 1 diabetics. What are the definitive diagnostic criteria for DKA?

Blood glucose level greater than 250 mg/dL (13.88 mmol/L); bicarbonate less than 15 mEq/L (15 mmol/L) and pH less than 7.3

A 64-year-old client comes to the clinic for a routine checkup and, while talking to the nurse, discusses some recent vision problems. Upon examination the nurse notices that the client's lenses are yellow. The client further states there have been problems with color discrimination as well as distance vision. What should the nurse suspect as the diagnosis for this client?

Cataracts Explanation: A cataract is a lens opacity that interferes with the transmission of light to the retina. Aging is the most common cause of cataracts. With further progression, there can be loss of color discrimination and also loss of vision, typically greater for distance that for near vision. This condition has nothing to do with the retina. Hyperopia is another term for farsightedness. Presbyopia refers to a decrease in accommodation that occurs because of aging.

The physician is assessing a client with a preliminary diagnosis of endocrine disorder. Further assessment findings identify abnormalities with emotion, pain, body temperature, and neural input. The physician determines the need to further assess the:

Hypothalamus

14. A stroke affecting which of the following areas of the brain would be most likely to leave an individual's vestibular system intact and posture and balance maintained? A) The brain stem B) The thalamus C) The temporal and parietal cortex D) The limbic system of the cerebrum

D Feedback: While the brain stem, thalamus, and temporal and parietal cortex are components of the vestibular apparatus, the limbic system is not a central component of the maintenance of posture and balance.

36. What does the combining form phac/o refer to? a. Iris b. Old age c. Hearing d. Lens e. vision

D. Lens

A patient experiences an increase in cortisol as a result of Cushing's disease. Which of the following hormonal responses demonstrates the negative feedback mechanism? a) Increased follicle-stimulating hormone b) Decreased adrenocorticotropic hormone (ACTH) c) Increased thyrotropin-releasing hormone d) Decreased thyroid-stimulating hormone

Decreased adrenocorticotropic hormone (ACTH) Negative feedback occurs when secretion of one hormone causes a reduction in the secretion of the hormone that stimulates production of the first hormone. In this case, ACTH manufactured by the anterior pituitary gland would normally stimulate the release of cortisol, but with the increase of cortisol produced by the secreting tumor, enough cortisol already floods the system that there should be a reduction in the ACTH level.

A client experiences an increase in thyroid hormone as a result of a thyroid tumor. Which hormonal response demonstrates the negative feedback mechanism?

Decreased thyroid-stimulating hormone (TSH)

A patient experiences an increase in thyroid hormone as a result of a thyroid tumor. Which of the following hormonal responses demonstrates the negative feedback mechanism?

Decreased thyroid-stimulating hormone (TSH)

When caring for a client who is being screened for osteoporosis, the nurse plans to evaluate the results of which of theses diagnostic tests?

Dual-energy x-ray absorptiometry (DEXA) Explanation: Dual-energy x-ray absorptiometry (DEXA) is used routinely for the diagnosis and monitoring of osteoporosis and metabolic bone diseases. Page 765

Congenital adrenal hyperplasia is a congenital disorder in which a deficiency exists in any of the enzymes necessary for the synthesis of cortisol. Infants of both sexes are affected, although boys are not diagnosed at birth unless of enlarged genitalia. Female infants often have ambiguous genitalia because of the oversecretion of adrenal androgens. What are the manifestations of the ambiguous genitalia caused by congenital adrenal hyperplasia? a) Enlarged clitoris, fused labia, and urogenital sinus b) Small clitoris, open labia, and urogenital sinus c) Small clitoris, fused labia, and urogenital sinus d) Enlarged clitoris, open labia, and urogenital sinus

Enlarged clitoris, fused labia, and urogenital sinus In female infants, an increase in androgens is responsible for creating the virilization syndrome of ambiguous genitalia with an enlarged clitoris, fused labia, and urogenital sinus. The other answers are incorrect

The endocrine system is closely linked with the nervous system. What neurotransmitter can also act as a hormone?

Epinephrine Explanation: Neurotransmitters such as epinephrine can act as neurotransmitters or as hormones. The other answers are not correct.

When caring for a client with anemia and a decrease in red blood cells (RBCs), the nurse recognizes which of these hormones will stimulate the bone marrow to produce additional RBCs?

Erythropoietin

The nurse is conducting a vision assessment on a 2-year-old child. The nurse should assess for which visual ability in this child?

Fixation on an object Explanation: The best vision assessment for infants and children under 3 years of age is accomplished by determining whether each eye can fixate on an object, maintain fixation, and follow the object. Recognizing letters and shapes and numbers is not appropriate for a 2-year-old.

Vitreous humor occupies the posterior portion of the eyeball. It is an amorphous biologic gel. When liquefaction of the gel occurs, as in aging, what can be seen during head movement?

Floaters Explanation: With the loss of gel structure, fine fibers, membranes, and cellular debris develop. When this occurs, floaters (images) can often be noticed because these substances move within the vitreous cavity during head movement. Blind spots, meshlike structures, and red spots are not seen during head movement with a loss of the gel structure of the vitreous humor.

Which hormone is secreted based on a cyclic rather than a diurnal manner?

Follicle-stimulating hormone (FSH)

What is the major cause of sensorineural hearing loss in children?

Genetic causes Explanation: Causes of sensorineural hearing impairment include genetic, infectious (most common infectious cause is cytomegalovirus (CMV), traumatic, and ototoxic factors). Genetic causes are probably responsible for as much as 50% of sensorineural hearing loss in children. Foreign bodies, including pieces of cotton and insects, may impair hearing and result in conductive hearing loss.

Hearing loss in children can be either conductive or sensorineural, as it is in adults. What is the major cause of sensorineural hearing loss in children?

Genetic causes.

A woman with a progressively enlarging neck comes into the clinic. She mentions that she has been in a foreign country for the previous 3 months and that she didn't eat much while she was there because she didn't like the food. She also mentions that she becomes dizzy when lifting her arms to do normal household chores or when dressing. What endocrine disorder should the nurse expect the physician to diagnose?

Goiter

Release and synthesis of anterior pituitary hormones are mainly regulated by which part of the body?

Hypothalamus

Which gland acts as a signal relaying bridge between multiple body systems and the pituitary gland?

Hypothalamus

The nurse is conducting a community education class on acute otitis media. Which statement by the participants indicates to the nurse that they are understanding the education? a) It is another name for swimmer's ear. b) It can be caused by a bacterial infection. c) The tympanic membrane appears cloudy. d) Breastfed babies have a higher incidence of this condition.

It can be caused by a bacterial infection. The nurse determines that the participants are understanding the information when they state that acute otitis media can be caused by a bacterial infection. -Breast fed babies have a lower incidence of acute otitis media. -The tympanic membrane appears cloudy in otitis media with effusions. -Acute otitis media is not another name for swimmer's ear

Which of the following organ systems is matched with the preferred type of imaging? a) Computed tomography (CT) scan of the thyroid gland b) Ultrasound of bone density c) Magnetic resonance imaging (MRI) of the pituitary gland d) Dual energy x-ray absorptiometry (DEXA) scan of the adrenal glands

Magnetic resonance imaging (MRI) of the pituitary gland Preferred types of scans for endocrine tissues include MRI of the pituitary gland and hypothalamus, CT scan of the adrenal glands, DEXA scan of bone density, and ultrasound of possible nodules of the thyroid gland.

The nurse screening for diabetes mellitus at a health fair does not have enough supplies for all of the clients attending. Which client should be given screening priority?

Male age 45, BMI 37

Amblyopia, or lazy eye, occurs at a time when visual deprivation or abnormal binocular interactions occur in visual infancy. Whether amblyopia is reversible depends on what?

Maturity of the visual system at time of onset

Neurotransmitters like catecholamines (ex. dopamine and epinephrine) have a reaction time of:

Milliseconds.

Which of the following hormones will elicit a more rapid response than the others?

Norepinephrine Explanation: Neurotransmitter hormones open ion channels and produce extremely rapid responses, whereas peptides and proteins change cell metabolism over a period of days. Page 754

Which of the following hormones will elicit a more rapid response than the others?

Norepinephrine Explanation: Neurotransmitter hormones open ion channels and produce extremely rapid responses, whereas peptides and proteins change cell metabolism over a period of days.

Which of the following inhibits growth hormone (GH)? a) Starvation b) Obesity c) Hypoglycemia d) Heavy exercise

Obesity GH is inhibited by increased glucose levels, free fatty acid release, cortisol, and obesity. It is stimulated by hypoglycemia, fasting starvation, increased blood levels of amino acids, and stress conditions such as trauma, excitement, emotional stress, and heavy exercise.

A client has glaucoma. The nurse is taking a health history and knows that the most common form of glaucoma is which type?

Open angle Explanation: Open-angle glaucoma is the most common form of glaucoma, and is usually asymptomatic and chronic, causing progressive damage to the optic nerve and visual field loss unless it is appropriately treated. Angle-closure glaucoma usually occurs as the result of an inherited anatomic defect that causes a shallow anterior chamber. There are several types of childhood glaucoma, including congenital glaucoma that is present at birth and infantile glaucoma that develops during the first 2 to 3 years of life.

When trying to explain hypothyroidism to a newly diagnosed client, the nurse stresses the fact that the thyroid hormone is transported in blood by specific:

PROTEINS

Which gland acts as a signal relaying bridge between multiple body systems and the pituitary gland? Hypothalamus Posterior pituitary Thyroid Parathyroid

Hypothalamus

Which type of receptor is mediated through vision in dim light? a) Mechanoreceptors b) Chemoreceptors c) Osmoreceptors d) Photoreceptors

Photoreceptors Vision in dim light is mediated through photoreceptors. - Hair cells are classified as mechanoreceptors. -The sensation of taste is recognized by ligand binding to chemoreceptors. -Osmoreceptors are not associated with vision

Which gland is often referred to as the master gland because it secretes many hormones? Pituitary Thyroid Pancreas Hypothalamus

Pituitary

Which gland is often referred to as the master gland because it secretes many hormones?

Pituitary Explanation: The pituitary gland has been called the master gland because its hormones control the functions of many target glands and cells. That is not a term used to refer to the other options. Page 761

Which gland is often referred to as the master gland because it secretes many hormones? a) Hypothalamus b) Pituitary c) Pancreas d) Thyroid

Pituitary The pituitary gland has been called the master gland because its hormones control the functions of many target glands and cells. That is not a term used to refer to the other options.

Which gland is often referred to as the master gland because it secretes many hormones?

Pituitary Explanation: The pituitary gland has been called the master gland because its hormones control the functions of many target glands and cells. That is not a term used to refer to the other options.

The nurse is caring for a client who has been diagnosed with Meniere disease. The nurse expects which of the following medications will be prescribed to manage the symptoms? a) Promethazine (Phenergan) b) Chlorpropamide (Diabenase) c) Clopidogrel (Plavix) d) Pantoprazole (Protonix)

Promethazine (Phenergan) Pharmacologic management includes suppressant drugs such as promethazine (Phenergan). -The other medications are not used to treat Meniere disease.

Select the most accurate statement regarding measurements of urinary hormone: a) Drugs will not alter testing results. b) Discarded samples will not alter results. c) Provide a better measure of hormone levels during a designated period. d) Requires blood and urine sampling.

Provide a better measure of hormone levels during a designated period. Measurements of urinary hormone or hormone metabolite excretion often are done on a 24-hour urine sample and provide a better measure of hormone levels during that period than hormones measured in an isolated blood sample. The advantages are relative ease of obtaining urine samples and blood sampling is not required. The disadvantages are that timed urine collections often are difficult to obtain and urine samples may be accidentally discarded or inaccurately preserved; drugs or disease states that alter hormone metabolism may interfere with the test results

Objective tinnitus is tinnitus that someone else can hear. What does the tinnitus that is caused by vascular disorders sound like?

Pulses.

The nurse is caring for a client who is receiving exogenous corticosteroids for rheumatoid arthritis. Recognizing that hormone levels are regulated by negative feedback, which of these laboratory test results does the nurse anticipate uncovering when reviewing the medical record? a) Reduction in TSH b) Reduction in ACTH c) Increase in ACTH d) Increase in TSH

Reduction in ACTH The function of the negative feedback system is similar to that of the thermostat in a heating system. When the sensors detect a decrease in hormone levels, they initiate changes that cause an increase in hormone production. When hormone levels rise above the set point of the system, the sensors cause hormone production and release to decrease, driving the stimulating hormone down.

Presbycusis is degenerative hearing loss associated with aging. What is the first symptom of this disorder? a) Inability to localize sounds b) Reduction in ability to identify sounds c) Reduction in ability to understand speech d) Inability to detect sound

Reduction in ability to understand speech The disorder first reduces the ability to understand speech and, later, the ability to detect, identify, and localize sounds.

A client is admitted for treatment of the syndrome of inappropriate antidiuretic hormone (SIADH). Which nursing intervention is appropriate?

Restricting fluids

The nurse takes a patient's blood pressure in the primary care provider's office. The last three times the patient has been in the office it has been high. What structure in the eye is this adverse effect of hypertension most likely to affect?

Retina

The nurse is assessing a steel mill worker who suffers from hearing loss related to exposure to loud, occupational noise. The nurse understands that this type of hearing loss is classified as which of the following?

Sensorineural hearing loss

Research has identified a cycle of insulin-induced posthypoglycemic episodes. What is this phenomenon called? a) Dawn phenomenon b) Joslin phenomenon c) Somogyi effect d) Sunset effect

Somogyi effect The Somogyi effect describes a cycle of insulin-induced posthypoglycemic episodes. In 1924, Joslin and associates noticed that hypoglycemia was associated with alternate episodes of hyperglycemia. The other answers are not correct. (

Otosclerosis is a condition where spongy, pathologic bone grows around the stapes and oval window.Which surgical treatment may be performed for otosclerosis?

Stapedectomy Explanation: Because much of the conductive hearing loss associated with otosclerosis is caused by stapedial fixation, surgical treatment involves stapedectomy with stapedial reconstruction using the client's own stapes or stapedial prosthesis.

The kidney produces 1,25-dihydroxyvitamin D. This form of vitamin D is responsible for which action in the body?

Stimulates calcium absorption from the intestine

The kidney produces 1,25-dihydroxyvitamin D. This form of vitamin D is responsible for which action in the body?

Stimulates calcium absorption from the intestine Explanation: 1,25-dihydroxyvitamin D from the kidney stimulates calcium absorption from the intestine. Somatostatin from the pancreas delays interestingal absorption of glucose. Thyroxine (T4) can increase metabolic rate. Growth hormone (GH) stimulates growth of bone and muscle. Page 756

A 21-year-old female is suspected of having inadequate function of her hypothalamic-pituitary-thyroid system. Her care provider is planning to inject thyrotropin-releasing hormone (TRH) and then measure her levels of TSH. Which diagnostic test is being performed?

Stimulation test

Which gland acts as a signal-relaying bridge between multiple body systems and the pituitary gland?

Hypothalamus

A patient suffers from seasonal allergic rhinoconjunctivitis. What is the best treatment for prolonged allergic conjunctivitis?

Systemic antihistamines.

A woman with a diagnosis of type 2 diabetes has been ordered a hemoglobin A1C test for the first time by her primary care provider. The woman states, "I don't see why you want to test my blood cells when it's sugar that's the problem." What aspect of physiology will underlie the care provider's response to the client?

The amount of glucose attached to A1C cells reflects the average blood glucose levels over the life of the cell.

A woman with a diagnosis of type 2 diabetes has been ordered a hemoglobin A1C test for the first time by her primary care provider. The woman states, "I don't see why you want to test my blood cells when its sugar that's the problem." What aspect of physiology will underlie the care provider's response to the client? a) Hemoglobin synthesis by the bone marrow is inversely proportionate to blood glucose levels, with low A1C indicating hyperglycemia. b) The high metabolic needs of red cells and their affinity for free glucose indicate the amount of glucose that has been available over 6 to 12 weeks. c) Insulin is a glucose receptor agonist on the hemoglobin molecule and high glucose suggests low insulin levels. d) The amount of glucose attached to A1C cells reflects the average blood glucose levels over the life of the cell.

The amount of glucose attached to A1C cells reflects the average blood glucose levels over the life of the cell. Glucose entry into red blood cells is not insulin dependent, and the rate at which glucose becomes attached to the hemoglobin molecule depends on blood glucose levels. A1C levels thus indirectly indicate glucose levels. Hemoglobin synthesis, the metabolic needs of hemoglobin and an agonist role of insulin do not underlie the A1C test.

The nurse is educating a newly diagnosed client with Hashimoto thyroiditis who is to be discharged from the acute care facility. What should the nurse be sure to include in the education to prevent complications? a) The client should be informed that if he observes an increase in fatigue, an extra dose of his thyroid medication should be taken. b) The client should skip a dose of the thyroid supplement if he has symptoms of fever or restlessness. c) The client should be informed about the signs and symptoms of severe hypothyroidism and the need for early intervention. d) The client should not participate in any exercise activity that could utilize more thyroid hormone.

The client should be informed about the signs and symptoms of severe hypothyroidism and the need for early intervention. Prevention is preferable to treatment and entails special attention to high-risk populations, such as women with a history of Hashimoto thyroiditis. These persons should be informed about the signs and symptoms of severe hypothyroidism and the need for early medical treatment

A client has been diagnosed with sensorineural hearing loss. What should the care team assess to determine a plausible cause?

The client's recent medication use Explanation: Ototoxic responses to medications can result in sensorineural hearing loss. Cerumen accumulation can cause conductive hearing loss. The client's immunization status and the patency of Eustachian tubes are not related to the most common causes of sensorineural hearing loss.

Which physiologic process best exemplifies a positive feedback mechanism?

The increase in prolactin secretion that occurs with more frequent breast-feeding

The nurse is teaching a client who has been newly diagnosed with hypothyroidism about the function of the thyroid. Which of these does the nurse explain to the client is the role of the thyroid gland?

The thyroid gland is responsible for increasing the metabolic rate. Explanation: The thyroid gland produces thyroid hormones, T3 and T4. These hormones increase the metabolic rate; increase protein and bone turnover; increase responsiveness to catecholamines; are necessary for fetal and infant growth and development. The parathyroid gland regulates calcium metabolism. The adrenal glands regulate flight or fight and the testes or ovaries regulate development of secondary sex characteristics.

Which of the following describes how water-soluble peptides such as parathyroid hormone, or glucagon, exert their effect on cells?

They bind to receptors.

Which statement describes how water-soluble peptides, such as parathyroid hormone or glucagon, exert their effect on cells?

They bind to receptors.

The nurse observes nystagmus in a client sitting quietly without head movement. The nurse interprets this finding in which of the following way?

This is a pathological finding.

The nurse observes nystagmus in a client sitting quietly without head movement. The nurse interprets this finding in which way?

This is a pathological finding. Explanation: Spontaneous nystagmus that occurs without head movement or visual stimuli is always pathologic and requires investigation, but is not an emergency situation.

Binocular vision depends on the coordination of how many pairs of extraocular nerves that provide for the conjugate eye movements?

Three

A client with a history of an endocrine disorder exhibits signs and symptoms of hormone deficiency. Which of the following processes would the client's care team most likely rule out first as a contributing factor?

Up-regulation has increased the sensitivity of the body to particular hormone levels. Explanation: Up-regulation is a response to low hormone levels in which the number of receptors increases. As such, it would not likely result in signs and symptoms of deficiency but is rather a compensatory mechanism that counters a deficiency. Insufficient numbers of receptors, low affinity and down-regulation could all contribute to signs and symptoms of a hormone deficiency. Page 758

Sometimes the measurement of hormones is done through a urine sample. What is an advantage of measuring hormone levels through a urine sample rather than a blood sample? a) Urine samples are easily obtaine b) Blood sampling has more pure hormone than urine does. c) There are more hormone metabolites in urine than in blood. d) Urine has more accurate measurements of hormones.

Urine samples are easily obtaine The advantages of a urine test include the relative ease of obtaining urine samples and the fact that blood sampling is not required. The other answers are not true.

While on a cruise to the Caribbean, a person develops "motion sickness" with associated malaise, and nausea/vomiting. The nurse notes the client's BP is 88/52; pulse is 110; and skin moist with perspiration. The client diagnosis related to the clinical manifestations would most likely be:

Vertigo

A client with a new diagnosis of an endocrine disorder is unclear how the body can control the levels of different hormones over time. Which statement most accurately underlies the dominant regulation process of hormone levels in the body?

With input from various sensors, hormone production and release are adjusted based on existing hormone levels.

The obstetrical nurse is caring for a client who has been treated for gestational diabetes. When teaching the client about the causes of gestational diabetes, the nurse should include which risk factor in the teaching?

Woman with a family history of diabetes

A patient has been diagnosed with bacterial conjunctivitis. This type of infection is usually characterized by large amounts of what color drainage?

Yellow-green

A client develops fever, headache, and burning/itching in the periorbital area. After a few days, a vesicular rash appears around the eyelid margins. The health care provider will likely prescribe:

antiviral medication for herpes zoster ophthalmicus. Explanation: Herpes zoster ophthalmicus usually presents with malaise, fever, headache, and burning and itching of the periorbital area. These symptoms commonly precede the ocular eruption by a day or two. The rash, which is initially vesicular, becomes pustular and then develops crusts. Treatment includes the use of oral and intravenous antiviral drugs. Initiation of treatment within the first 72 hours after the appearance of the rash reduces the incidence of ocular complications but not the postherpetic neuralgia. Chlamydial conjunctivitis is commonly spread by contact with genital secretions. It is treated with antimicrobial medications. Causes of ulcerative keratitis include infectious agents, exposure to trauma, and use of extended-wear contact lenses. The first manifestations of recurrent herpes keratitis are irritation, photophobia, and tearing. A history of fever blisters or other herpetic infection is often noted. Allergic conjunctivitis encompasses a spectrum of conjunctival conditions usually characterized by itching. Allergic conjunctivitis also has been successfully treated with topical mast cell stabilizers, histamine type 1 (H1) receptor antagonists, and topical nonsteroidal anti-inflammatory drugs.

When a patient who takes metformin (Glucophage) to manage type 2 diabetes develops an allergic rash from an unknown cause, the health care provider prescribes prednisone (Deltasone). The nurse will anticipate that the patient may a. need a diet higher in calories while receiving prednisone. b. develop acute hypoglycemia while taking the prednisone. c. require administration of insulin while taking prednisone. d. have rashes caused by metformin-prednisone interactions.

c. require administration of insulin while taking prednisone.

A patient with type 2 diabetes is scheduled for a follow-up visit in the clinic several months from now. Which test will the nurse schedule to evaluate the effectiveness of treatment for the patient? a. Urine dipstick for glucose b. Oral glucose tolerance test c. Fasting blood glucose level d. Glycosylated hemoglobin level

d. Glycosylated hemoglobin level

Which action by a patient indicates that the home health nurse's teaching about glargine and regular insulin has been successful? a. The patient administers the glargine 30 minutes before each meal. b. The patient's family prefills the syringes with the mix of insulins weekly. c. The patient draws up the regular insulin and then the glargine in the same syringe. d. The patient disposes of the open vials of glargine and regular insulin after 4 weeks.

d. The patient disposes of the open vials of glargine and regular insulin after 4 weeks.

Which information is most important for the nurse to report to the health care provider before a patient with type 2 diabetes is prepared for a coronary angiogram? a. The patient's most recent HbA1C was 6.5%. b. The patient's admission blood glucose is 128 mg/dL. c. The patient took the prescribed metformin (Glucophage) today. d. The patient took the prescribed captopril (Capoten) this morning.

d. The patient took the prescribed captopril (Capoten) this morning.

89. strabismus in which there is deviation of the visual axis of one eye away from that of the other eye; also called wall-eye a. achromatopsia k. macular degeneration b. anacusis l. meniere disease c. astigmatism m. myringoplasty d. audiometry n. myringotomy e. cataract o. otoscopy f. diabetic retinopathy p. photophobia g. exotropia q. rinne test h. glaucoma r. strabismus i. hordeolum s. tinnitus j. iridoplegia t. vertigo

g. exotropia

The nursing student who is studying pathophysiology correctly identifies the condition that characteristically has an increase in the aqueous humor which fills the anterior and posterior chambers of the eye as which disease? a) cataract b) glaucoma c) retinopathy d) amblyopia

glaucoma Glaucoma is a chronic degenerative optic neuropathy characterized by optic disk cupping and visual field loss. It is usually associated with an elevation in intraocular pressure. Amblyopia is lazy eye while retinopathy is a disorder of the retinal vessels which interrupt blood flow to the visual receptors leading to visual impairment. A cataracts is a lens opacity that interferes with the transmission light to the retina

78. unusual intolerance and sensitivity to light that occurs in such diseases as meningitis, inflammation of the eyes, measles, and rubella a. achromatopsia k. macular degeneration b. anacusis l. meniere disease c. astigmatism m. myringoplasty d. audiometry n. myringotomy e. cataract o. otoscopy f. diabetic retinopathy p. photophobia g. exotropia q. rinne test h. glaucoma r. strabismus i. hordeolum s. tinnitus j. iridoplegia t. vertigo

p. photophobia

During a routine 2-month checkup at the pediatric clinic, a mother expresses concern that her son looks "cross-eyed." She asks if she needs to put patches over his good eye. Assessment reveals full eye movement, and the child uses each eye independently. The health care provider explains that the best treatment for the infant's eye problem is:

to prepare for some surgery to correct this problem early in life in order to correct the eye muscle disorder. Explanation: The disorder may be nonaccommodative, accommodative, or a combination of the two. Infantile esotropia is the most common cause of nonaccommodative strabismus. It occurs in the first 6 months of life, with large-angle deviations, in otherwise developmentally and neurologically normal infants. Eye movements are full, and the child often uses each eye independently to alter fixation (cross-fixation). Infantile esotropia is usually treated surgically by weakening the medial rectus muscle on each eye while the infant is under general anesthesia. Recurrences are common with infantile esotropia, and multiple surgeries are often required.

A client presents with varicella predominately occuring from the top of the nose to the eyelid margins. Which further assessments correlate to the development of varicella-zoster ophthalmicus? Select all that apply.

uveitis with elevated intraocular pressure of 32 mm Hg tearing of the eye and photophobia eye pain and vesicular rash Explanation: Herpes zoster ophthalmicus usually presents with malaise, fever, headache, and burning and itching of the periorbital area. These symptoms commonly precede the ocular eruption by a day or so. The rash is initally vesicular, becomes pustular and then crusts. Involvement of the tip of the nose and lid margins indicates a high likelihood of ocular involvement. Ocular signs include conjunctivitis, keratitis (irritation tearing and photophobia), and anterior uveitis, often with elevated intraoccular pressure. Bruising is not common with this diagosis. Bulging of the eye orbit (exophthalmos) is usually associated with thyroid problems (hyperthyroidism).

The nurse understands that movement of otoliths may result in:

vertigo and nystagmus. Explanation: Movement of the otoliths or free-floating debris causes the vestibular system to become more sensitive, such that any movement of the head in the plane parallel to the posterior duct may cause vertigo and nystagmus.

Loss of pituitary function can result in deficiencies/loss of which of the following hormones' secretions? Select all that apply. a) Growth hormone b) Luteinizing hormone c) Prolactin d) Corticotropin-releasing hormone e) Follicle stimulating hormone

• Growth hormone • Luteinizing hormone • Follicle stimulating hormone • Prolactin Anterior pituitary hormone loss is usually gradual, especially with progressive loss of pituitary reserve due to tumors or previous pituitary radiation therapy (which may take 10 to 20 years to produce hypopituitarism). The loss of pituitary function tends to follow a classic course beginning with the loss of GH, LH, and FSH secretion followed by deficiencies in TSH, then ACTH, and finally prolactin.

A student nurse is taking a test on the endocrine system. From the following list of clinical manifestations, she needs to select the ones she would see in hypothyroidism. Which answers should she select? Select all that apply. a) Coarse brittle hair b) Heat intolerance c) Puffy face with swollen eyelids d) Weight gain despite loss of appetite e) Nervousness with fine muscle tremors

• Weight gain despite loss of appetite • Coarse brittle hair • Puffy face with swollen eyelids The hypometabolic state associated with hypothyroidism is characterized by a gradual onset of weakness and fatigue, a tendency to gain weight despite a loss of appetite, and cold intolerance. As the condition progresses, the skin becomes dry and rough and the hair becomes coarse and brittle. Reduced conversion of carotene to vitamin A and increased blood levels of carotene may give the skin a yellowish color. The face becomes puffy with edematous eyelids, and there is thinning of the outer third of the eyebrows. Nervousness with fine muscle tremors and heat intolerance are signs of hyperthyroidism.

A mother brings her baby in with a possible diagnosis of congential or infantile glaucoma. What are the clinical manifestations that the mother would tell the nurse about the baby? Select all that apply.

Fussy, Poor eating habits, and Rubs his eyes frequently.

During accommodation, pupillary dilation partially compensates for the reduced size of the retinal image by: A) Thickening the lens B) Contracting the ciliary muscle C) Increasing light entering the pupil D) Narrowing the palpebral opening

C) Increasing light entering the pupil

A client diagnosed with type 2 diabetes has been instructed about managing his condition with diet. The nurse determines further teaching is necessary when the client states: a) "I need to avoid adding salt to my foods." b) "I can drink 8 to 10 glasses of water daily without concern for calories." c) "I must avoid all candies and cookies, but can eat unlimited amounts of pasta and breads." d) "I need to limit the amount of foods that I eat that contain trans fats."

"I must avoid all candies and cookies, but can eat unlimited amounts of pasta and breads." The client needs more education regarding carbohydrate sources. All the other statements are correctly stated and demonstrate good understanding of dietary needs for the type 2 diabetic.

9. What is the medical term for farsightedness? a. Diplopia b. Hyperopia c. Myopia d. Erythropia e. Xanthopia

B. Hyperopia

A client with diabetes mellitus has sudden onset of slurred speech, incoordination, and cool, clammy skin. What will the nurse do first?

Check blood glucose

What is the most common mechanism of hormone control?

Negative feedback

67. act of measuring a. Acous/o k. kerat/o b. Blephar/o l. myring/o c. Chromat/o m. ophthalm/o d. Conjunctiv/o n. ot/o e. Choroid/o o. salping/o f. Core/o p. -metry g. Corne/o q. -opsia h. Dacry/o r. -ptosis i. dipl/o s. -rrhea j. Irid/o t. -tropia

p. -metry

A male patient has been diagnosed with a low sperm count. Which of the following endocrine imbalances could contribute to this condition? Select all that apply. a) Increased growth hormone b) Excess thyroid hormone c) Lack of follicle-stimulating hormone (FSH) d) Insufficient androgens e) Excess calcitonin

• Lack of follicle-stimulating hormone (FSH) • Insufficient androgens Sperm production is affected by a number of hormones. FSH from the anterior pituitary stimulates production as do androgens from the testes. Androgens from the adrenal cortex are converted to testosterone in the male. Thyroid hormone plays an indirect role in sperm production. Insufficient thyroid could lead to a low sperm count.

The nurse is conducting hearing evaluation of a group of pre-school children. The nurse understands that failure to recognize hearing loss in children can result in impairment of which of the following? Select all that apply.

• Language development • Emotional development • Cognitive development

A client is brought to the emergency department with a suspected diagnosis of DKA (diabetic ketoacidosis). Select the assessment/diagnostic data to confirm the diagnosis. Select all that apply. a) Hypoglycemia b) Positive urine ketones c) Low serum bicarbonate d) Negative serum ketones e) High arterial pH

• Low serum bicarbonate • Positive urine ketones The definitive diagnosis of DKA consists of hyperglycemia (blood glucose levels >250 mg/dL [13.8 mmol/L]), low serum bicarbonate, low arterial pH, and positive urine and serum ketones

Select the most common symptoms of diabetes. Select all that apply. a) Polycythemia b) Polydipsia c) Polyuria d) Polyphagia e) Polyhydramnios

• Polydipsia • Polyuria • Polyphagia The most commonly identified signs and symptoms of diabetes are often referred to as the three polys: (1) polyuria (i.e., excessive urination), (2) polydipsia (i.e., excessive thirst), and (3) polyphagia (i.e., excessive hunger). Polyhydramnios is a medical condition describing an excess of amniotic fluid in the amniotic sac. Polycythemia is a condition of increased red blood cells.

A client presents to the emergency room with fatigue, weakness, dehydration and thirst. What additional symptoms would correlate with a diagnosis of type 1 diabetes mellitus? Select all that apply. a) Polyuria b) Blurred vision c) Joint pain d) Insomnia e) Recent weight loss

• Polyuria • Blurred vision • Recent weight loss The cardinal manifestations of diabetes mellitus are the three "polys": polyuria, polydipsia, and polyphagia, and weight loss. Additional symptoms include fatigue and weakness, blurred vision, and skin infections.

A 69-year-old patient comes to the clinic for a routine checkup. Upon examination the nurse practitioner informs the patient that she has cataracts. The patient then tells the nurse that she already knew that and her physician told her that she could use bifocals and that would take care of the problem. What would be the best response by the nurse practitioner? a) "Strong bifocal lenses can often cure cataracts." b) "Surgery is the only effective treatment for cataracts." c) "The doctor was correct and you are doing everything you can to help with this condition." d) "You are wrong and should not listen to your doctor."

"Surgery is the only effective treatment for cataracts." There is no effective medical treatment for cataract. -Strong bifocal lenses, magnification, appropriate lighting, and visual aids may be used as the cataract progresses, but surgery is the only treatment for correcting cataract-related vision loss. -Telling the patient that bifocals will cure cataracts is false as well as telling her that her doctor was correct. -The other option is not a therapeutic response by the nurse.

A child has been removed from a home in which she has experienced severe neglect and emotional abuse, and has been placed in foster care. The child has psychosocial dwarfism and the foster parents ask the nurse what this means for the future of the child. What is the best response by the nurse? a) "This situation will not improve at all and is not reversible." b) "The child must receive injections of growth hormone for the duration of her life." c) "The prognosis of the child depends on an improvement in behavior and catch-up growth." d) "The child will have a thin build and delayed skeletal and sexual maturation."

"The prognosis of the child depends on an improvement in behavior and catch-up growth." Psychosocial dwarfism involves a functional hypopituitarism and is seen in some emotionally deprived children. These children usually present with poor growth, potbelly, and poor eating and drinking habits. Typically, there is a history of disturbed family relationships in which the child has been severely neglected or disciplined. Often, the neglect is confined to one child in the family. GH function usually returns to normal after the child is removed from the constraining environment. The prognosis is dependent on improvement in behavior and catch-up growth.

The father of a third grade girl has brought his daughter to a walk-in clinic because he believes the girl has pink eye, which has been going around the students in her class. The nurse at the clinic concurs with the father's suspicion of conjunctivitis. Which follow-up explanation by the nurse is most accurate? a) "An antibiotic ointment will likely resolve her infection, but pain control will be necessary in the mean time." b) "The surfaces of her eyes have bacteria or a virus established, and it's important to maintain good hand hygiene until it goes away." c) "It's important to aggressively treat this in children, since damage to her sight can result if it's not treated." d) "The insides of her eyelids have become infected. This often produces severe discomfort."

"The surfaces of her eyes have bacteria or a virus established, and it's important to maintain good hand hygiene until it goes away." Conjunctivitis often spontaneously resolves. The pain associated with conjunctivitis usually produces only mild discomfort compared with severe discomfort associated with corneal lesions or deep and severe pain associated with acute glaucoma. Conjunctivitis may spread to other family members. - The corneal surface is not primarily involved and pain that is severe suggests corneal involvement rather than conjunctivitis. Sight damage is not likely to result.

A 70 year-old woman with a diagnosis of benign paroxysmal positional vertigo (BPPV) is receiving teaching from her physician about her diagnosis. The client is eager to avoid future episodes of vertigo and has asked the physician what she can do to prevent future episodes. How can the physician best respond? a) "Unfortunately there aren't any proven treatments for your condition." b) "There are some exercises that I'll teach you to help reorient your inner ear and prevent vertigo." c) "We usually don't actively treat BPPV unless it starts to affect your hearing." d) "Although they involve some risks, there are some options for ear surgery that can prevent future vertigo."

"There are some exercises that I'll teach you to help reorient your inner ear and prevent vertigo." Nondrug therapies for BPPV using habituation exercises and canalith repositioning are successful in many people. Canalith repositioning involves a series of maneuvers in which the head is moved to different positions in an effort to reposition the free-floating debris in the endolymph of the semicircular canals. Surgery is not a noted treatment option and even in the absence of hearing loss, treatment is warranted.

A client with Graves' disease has opthalmopathy and asks the nurse if the eyes will stay like this forever. What is the best response by the nurse? a) "The protrusion of the eyes will get worse before they get better." b) "With treatment of the hyperthyroid state, the opthalmopathy usually tends to stabilize." c) "Once we treat your Graves' disease, your eyes will go back to their normal state." d) "Your eyes will be like this but there are things we can do to reduce visual loss."

"With treatment of the hyperthyroid state, the opthalmopathy usually tends to stabilize." The ophlalmopathy of Graves' disease can cause severe eye problems, including tethering of the extraocular muscles resulting in diplopia; involvement of the optic nerve, with some visual loss; and corneal ulceration because the lids do not close over the protruding eyeball. The opthalmopathy usually tends to stabilize after treatment of the hyperthyroidism.

A nurse is assessing a client with Cushing's syndrome. Which observation should the nurse report to the physician immediately? -Pitting edema of the legs -An irregular apical pulse -Dry mucous membranes -Frequent urination

-An irregular apical pulse Because Cushing's syndrome causes aldosterone overproduction, which increases urinary potassium loss, the disorder may lead to hypokalemia. Therefore, the nurse should immediately report signs and symptoms of hypokalemia, such as an irregular apical pulse, to the physician. Edema is an expected finding because aldosterone overproduction causes sodium and fluid retention. Dry mucous membranes and frequent urination signal dehydration, which isn't associated with Cushing's syndrome.

A nurse is assigned to care for a patient with increased parathormone secretion. Which of the following serum levels should the nurse monitor for this patient? -Glucose -Sodium -Calcium -Potassium

-Calcium Increased secretion of parathormone results in bone resorption. Calcium is released into the blood, increasing serum levels.

A client with severe hypoparathyroidism is experiencing tetany. What medication, prescribed by the physician for emergency use, will the nurse administer to correct the deficit? -Sodium bicarbonate -Fludrocortisone -Calcium gluconate -Methylprednisolone

-Calcium gluconate Tetany and severe hypoparathyroidismare treated immediately by the administration of an IV calcium salt, such as calcium gluconate. The other medications are not effective for the treatment of calcium deficit.

Which outcome indicates that treatment of a client with diabetes insipidus has been effective? -Fluid intake is less than 2,500 ml/day. -Urine output measures more than 200 ml/hour. -Blood pressure is 90/50 mm Hg. -Heart rate is 126 beats/minute.

-Fluid intake is less than 2,500 ml/day. Diabetes insipidus is characterized by polyuria (up to 8 L/day), constant thirst, and an unusually high oral intake of fluids. Treatment with the appropriate drug should decrease both oral fluid intake and urine output. A urine output of 200 ml/hour indicates continuing polyuria. A blood pressure of 90/50 mm Hg and a heart rate of 126 beats/minute indicate compensation for the continued fluid deficit, suggesting that treatment hasn't been effective.

Trousseau's sign is elicited by which of the following? -Carpopedal spasm is induced by occluding the blood flow to the arm for 3 minutes with the use of a blood pressure cuff. -A sharp tapping over the facial nerve just in front of the parotid gland and anterior to the ear causes spasm or twitching of the mouth, nose, and eye. -After making a clenched fist, the palm remains blanched when pressure is placed over the radial artery. -The patient complains of pain in the calf when his foot is dorsiflexed.

-Carpopedal spasm is induced by occluding the blood flow to the arm for 3 minutes with the use of a blood pressure cuff. A positive Trousseau's sign is suggestive of latent tetany. A positive Chvostek's sign is demonstrated when a sharp tapping over the facial nerve just in front of the parotid gland and anterior to the ear causes spasm or twitching of the mouth, nose, and eye. A positive Allen's test is demonstrated by the palm remaining blanched with the radial artery occluded. The radial artery should not be used for an arterial puncture. A positive Homans' sign is demonstrated when the patient complains of pain in the calf when his foot is dorsiflexed.

Which disorder is characterized by a group of symptoms produced by an excess of free circulating cortisol from the adrenal cortex? -Cushing syndrome -Addison disease -Graves disease -Hashimoto disease

-Cushing syndrome The client with Cushing syndrome demonstrates truncal obesity, moon face, acne, abdominal striae, and hypertension. Regardless of the cause, the normal feedback mechanisms that control the function of the adrenal cortex become ineffective, and the usual diurnal pattern of cortisol is lost. The signs and symptoms of Cushing syndrome are primarily a result of the oversecretion of glucocorticoids and androgens, although mineralocorticoid secretion also may be affected.

A nurse is planning care for a client in acute addisonian crisis. Which nursing diagnosis should receive the highest priority? -Risk for infection -Decreased cardiac output -Impaired physical mobility -Imbalanced nutrition: Less than body requirements

-Decreased cardiac output An acute addisonian crisis is a life-threatening event, caused by deficiencies of cortisol and aldosterone. Glucocorticoid insufficiency causes a decrease in cardiac output and vascular tone, leading to hypovolemia. The client becomes tachycardic and hypotensive and may develop shock and circulatory collapse. The client with Addison's disease is at risk for infection; however, reducing infection isn't a priority during an addisonian crisis. Impaired physical mobility and Imbalanced nutrition: Less than body requirements are appropriate nursing diagnoses for the client with Addison's disease, but they aren't priorities in a crisis.

A client with a traumatic brain injury is producing an abnormally large volume of dilute urine. Which alteration to a hormone secreted by the posterior pituitary would the nurse expect to find? -Deficient production of vasopressin -Increased antidiuretic hormone -Increased oxytocin -A deficient amount of somatostatin

-Deficient production of vasopressin The most common disorder related to posterior lobe dysfunction is diabetes insipidus, a condition in which abnormally large volumes of dilute urine are excreted as a result of deficient production of vasopressin. Diabetes insipidus may occur following surgical treatment of a brain tumor, secondary to nonsurgical brain tumors, and traumatic brain injury.

During physical examination of a client with a suspected endocrine disorder, the nurse assesses the body structures. The nurse gathers this data based on the understanding that it is an important aid in which of the following? -Detecting evidence of hormone hypersecretion. -Detecting information about possible tumor growth. -Determining the presence or absence of testosterone levels. -Determining the size of the organs and location.

-Detecting evidence of hormone hypersecretion. The evaluation of body structures helps the nurse detect evidence of hypersecretion or hyposecretion of hormones. This helps in the assessment of findings that are unique to specific endocrine glands. Radiographs of the chest or abdomen are taken to detect tumors. Radiographs also determine the size of the organ and its location. Antidiuretic hormone (ADH) levels determine the presence or absence of ADH and testosterone levels.

A client sustained a head injury when falling off of a ladder. While in the hospital, the client begins voiding large amounts of clear urine and states he is very thirsty. The client states that he feels weak, and he has had an 8-lb weight loss since admission. What should the client be tested for? -Diabetes insipidus (DI) -Syndrome of inappropriate antidiuretic hormone secretion (SIADH) -Pituitary tumor -Hypothyroidism

-Diabetes insipidus (DI) Urine output may be as high as 20 L/24 hours. Urine is dilute, with a specific gravity of 1.002 or less. Limiting fluid intake does not control urine exertion. Thirst is excessive and constant. Activities are limited by the frequent need to drink and void. Weakness, dehydration, and weight loss develop. SIADH will have the opposite clinical manifestations. The client's symptoms are related to the trauma and not a pituitary tumor. The thyroid gland does not exhibit these symptoms.

The nurse obtains a complete family history of a client with a suspected endocrine disorder based on which rationale? -An allergy to iodine is inherited. -Endocrine disorders can be inherited. -It helps determine the client's general status. -Diet and drug histories are related to the family history.

-Endocrine disorders can be inherited. Some endocrine disorders are inherited or have a tendency to run in families. Therefore, it is essential to take a complete family history. A complete blood count and chemistry profile are performed to determine the client's general status and to rule out disorders. Obtaining information about an allergy to iodine is important because diagnostic testing may involve the use of contrast dyes. However, an allergy to iodine is not related to endocrine disorders. Diet and drug histories, although important information, are not associated with the family history.

A health care provider suspects that a thyroid nodule may be malignant. The nurse knows to prepare information for the patient based on the usual test that will be ordered to establish a diagnosis. What is that test? -Serum immunoassay for TSH -Fine-needle biopsy of the thyroid gland -Free T4 analysis -Ultrasound of the thyroid gland

-Fine-needle biopsy of the thyroid gland Fine needle biopsy of the thyroid gland is often used to establish the diagnosis of thyroid cancer. The purpose of the biopsy is to differentiate cancerous thyroid nodules from noncancerous nodules and to stage the cancer if detected. The procedure is safe and usually requires only a local anesthetic.

The nurse is reviewing a client's history which reveals that the client has had an over secretion of growth hormone (GH) that occurred before puberty. The nurse interprets this as which of the following? -Gigantism -Dwarfism -Acromegaly -Simmonds' disease

-Gigantism When over secretion of GH occurs before puberty, gigantism results. Dwarfism occurs when secretion of GH is insufficient during childhood. Oversecretion of GH during adulthood results in acromegaly. An absence of pituitary hormonal activity causes Simmonds' disease.

The nurse is reviewing a client's history which reveals that the client has had an oversecretion of growth hormone (GH) that occurred before puberty. The nurse interprets this as which of the following? -Gigantism -Dwarfism -Acromegaly -Simmonds' disease

-Gigantism When oversecretion of GH occurs before puberty, gigantism results. Dwarfism occurs when secretion of GH is insufficient during childhood. Oversecretion of GH during adulthood results in acromegaly. An absence of pituitary hormonal activity causes Simmonds' disease.

Which feature(s) indicates a carpopedal spasm in a client with hypoparathyroidism? -Hand flexing inward -Cardiac dysrhythmia -Moon face and buffalo hump -Bulging forehead

-Hand flexing inward Carpopedal spasm is evidenced by the hand flexing inward. Cardiac dysrhythmia is a symptom of hyperparathyroidism. Moon face and buffalo hump are the symptoms of Cushing syndrome. A bulging forehead is a symptom of acromegaly.

A client with acromegaly has been given the option of a surgical approach or a medical approach. The client decides to have a surgical procedure to remove the pituitary gland. What does the nurse understand this surgical procedure is called? -Hypophysectomy -Hysteroscopy -Thyroidectomy -Ablation

-Hypophysectomy The treatment of choice is surgical removal of the pituitary gland (transsphenoidal hypophysectomy) through a nasal approach. The surgeon may substitute an endoscopic technique using microsurgical instruments to reduce surgical trauma. A hysteroscopy is a gynecologic procedure. The thyroid gland is not involved for a surgical procedure. Ablation is not a removal of the pituitary gland.

A client has been diagnosed with myxedema from long-standing hypothyroidism. What clinical manifestations of this disorder does the nurse recognize are progressing to myxedema coma? Select all that apply. -Hypothermia -Hypertension -Hypotension -Hypoventilation -Hyperventilation

-Hypothermia -Hypotension -Hypoventilation Severe hypothyroidism is called myxedema. Advanced, untreated myxedema can progress to myxedemic coma. Signs of this life-threatening event are hypothermia, hypotension, and hypoventilation. Hypertension and hyperventilation indicate increased metabolic responses, which are the opposite of what the client would be experiencing.

A client is suspected to have a pituitary tumor due to signs of diabetes insipidus. What initial test does the nurse help to prepare the client for? -Magnetic resonance imaging (MRI) -Radioactive iodine uptake test -Radioimmunoassay -A nuclear scan

-Magnetic resonance imaging (MRI) A computed tomography (CT) or magnetic resonance imaging (MRI) scan is performed to detect a suspected pituitary tumor or to identify calcifications or tumors of the parathyroid glands. A radioactive iodine uptake test would be useful for a thyroid tumor. Radioimmunoassay determines the concentration of a substance in plasma.

For a client with Graves' disease, which nursing intervention promotes comfort? -Restricting intake of oral fluids -Placing extra blankets on the client's bed -Limiting intake of high-carbohydrate foods -Maintaining room temperature in the low-normal range

-Maintaining room temperature in the low-normal range Graves' disease causes signs and symptoms of hypermetabolism, such as heat intolerance, diaphoresis, excessive thirst and appetite, and weight loss. To reduce heat intolerance and diaphoresis, the nurse should keep the client's room temperature in the low-normal range. To replace fluids lost via diaphoresis, the nurse should encourage, not restrict, intake of oral fluids. Placing extra blankets on the bed of a client with heat intolerance would cause discomfort. To provide needed energy and calories, the nurse should encourage the client to eat high-carbohydrate foods.

A client receiving thyroid replacement therapy develops influenza and forgets to take her thyroid replacement medicine. The nurse understands that skipping this medication puts the client at risk for developing which life-threatening complication? -Exophthalmos -Thyroid storm -Myxedema coma -Tibial myxedema

-Myxedema coma Myxedema coma, severe hypothyroidism, is a life-threatening condition that may develop if thyroid replacement medication isn't taken. Exophthalmos (protrusion of the eyeballs) is seen with hyperthyroidism. Although thyroid storm is life-threatening, it's caused by severe hyperthyroidism. Tibial myxedema (peripheral mucinous edema involving the lower leg) is associated with hypothyroidism but isn't life-threatening.

A nurse is caring for a client who had a thyroidectomy and is at risk for hypocalcemia. What should the nurse do? -Monitor laboratory values daily for elevated thyroid-stimulating hormone. -Observe for swelling of the neck, tracheal deviation, and severe pain. -Evaluate the quality of the client's voice postoperatively, noting any drastic changes. -Observe for muscle twitching and numbness or tingling of the lips, fingers, and toes.

-Observe for muscle twitching and numbness or tingling of the lips, fingers, and toes. Muscle twitching and numbness or tingling of the lips, fingers, and toes are signs of hyperirritability of the nervous system due to hypocalcemia. The other options describe complications for which the nurse should also be observing; however, tetany and neurologic alterations are primary indications of hypocalcemia.

Which assessment would a nurse perform on a client with Cushing's syndrome who is at high risk of developing a peptic ulcer? -Observe stool color. -Monitor bowel patterns. -Monitor vital signs every 4 hours. -Observe urine output.

-Observe stool color. The nurse should observe the color of each stool and test the stool for occult blood.

An instructor is preparing a teaching plan for a class on the various pituitary hormones. Which hormone would the instructor include as being released by the posterior pituitary gland? -Somatotropin -Prolactin -Oxytocin -Adrenocorticotropic hormone

-Oxytocin The posterior pituitary gland released oxytocin and antidiuretic hormone. Somatotropin, prolactin, and adrenocorticotropic hormone are released by the anterior pituitary gland.

A client visits the clinic to seek treatment for disturbed sleep cycles and depressed mood. Which glands and hormones help to regulate sleep cycles and mood? -Thymus gland, thymosin -Parathyroid glands, parathormone -Pineal gland, melatonin -Adrenal cortex, corticosteroids

-Pineal gland, melatonin The pineal gland secretes melatonin, which aids in regulating sleep cycles and mood. Melatonin plays a vital role in hypothalamicpituitary interaction. The thymus gland secretes thymosin and thymopoietin, which aid in developing T lymphocytes. The parathyroid glands secrete parathormone, which increases the levels of calcium and phosphorus in the blood. The adrenal cortex secretes corticosteroids hormones, which influence many organs and structures of the body.

The nurse assesses a patient who has been diagnosed with Addison's disease. Which of the following is a diagnostic sign of this disease? -Potassium of 6.0 mEq/L -Sodium of 140 mEq/L -Glucose of 100 mg/dL -A blood pressure reading of 135/90 mm Hg

-Potassium of 6.0 mEq/L Addison's disease is characterized by hypotension, low blood glucose, low serum sodium, and high serum potassium levels. The normal serum potassium level is 3.5 to 5 mEq/L.

A nurse is caring for a client with Cushing's syndrome. Which would the nurse not include in this client's plan of care? -Provide a high-sodium diet. -Examine extremities for pitting edema. -Report systolic BP that exceeds 139 mm Hg or diastolic BP that exceeds 89 mm Hg. -Administer prescribed diuretics.

-Provide a high-sodium diet. Limiting sodium reduces the potential for fluid retention. Fluid retention is manifested by swelling in dependent areas, pitting when pressure is applied to the skin over a bone by tight-fitting shoes or rings, the appearance of lines in the skin from stockings and seams in the shoes or areas where they lace. Hypertension is defined as a consistently elevated BP above 139/89 mm Hg. One factor that contributes to hypertension is excess circulatory volume. Diuretics promote the excretion of sodium and water.

A client with a history of chronic hyperparathyroidism admits to being noncompliant. Based on initial assessment findings, the nurse formulates the nursing diagnosis of Risk for injury. To complete the nursing diagnosis statement for this client, which "related-to" phrase should the nurse add? -Related to bone demineralization resulting in pathologic fractures -Related to exhaustion secondary to an accelerated metabolic rate -Related to edema and dry skin secondary to fluid infiltration into the interstitial spaces -Related to tetany secondary to a decreased serum calcium level

-Related to bone demineralization resulting in pathologic fractures Poorly controlled hyperparathyroidism may cause an elevated serum calcium level. This increase, in turn, may diminish calcium stores in the bone, causing bone demineralization and setting the stage for pathologic fractures and a risk for injury. Hyperparathyroidism doesn't accelerate the metabolic rate. A decreased thyroid hormone level, not an increased parathyroid hormone level, may cause edema and dry skin secondary to fluid infiltration into the interstitial spaces. Hyperparathyroidism causes hypercalcemia, not hypocalcemia; therefore, it isn't associated with tetany.

Patients with hyperthyroidism are characteristically: -Apathetic and anorexic -Calm -Emotionally stable -Sensitive to heat

-Sensitive to heat Those with hyperthyroidism tolerate heat poorly and may perspire unusually freely. Their condition is characterized by symptoms of nervousness, hyperexcitability, irritability, and apprehension.

A nurse is caring for a client in acute addisonian crisis. Which test result does the nurse expect to see? -Serum potassium level of 6.8 mEq/L -Blood urea nitrogen (BUN) level of 2.3 mg/dl -Serum sodium level of 156 mEq/L -Serum glucose level of 236 mg/dl

-Serum potassium level of 6.8 mEq/L A serum potassium level of 6.8 mEq/L indicates hyperkalemia, which can occur in adrenal insufficiency as a result of reduced aldosterone secretion. A BUN level of 2.3 mg/dl is lower than normal. A client in addisonian crisis is likely to have an increased BUN level because the glomerular filtration rate is reduced. A serum sodium level of 156 mEq/L indicates hypernatremia. Hyponatremia is more likely in this client because of reduced aldosterone secretion. A serum glucose level of 236 mg/dl indicates hyperglycemia. This client is likely to have hypoglycemia caused by reduced cortisol secretion, which impairs glyconeogenesis.

A client has a decreased level of thyroid hormone being excreted. What will the feedback loop do to maintain the level of thyroid hormone required to maintain homeostatic stability? -Stimulate more hormones using the negative feedback system -Stimulate more hormones using the positive feedback system -Produce a new hormone to try and regulate the thyroid function -The feedback loop will be unable to perform in response to low levels of thyroid hormone.

-Stimulate more hormones using the negative feedback system Feedback can be either negative or positive. Most hormones are secreted in response to negative feedback; a decrease in levels stimulates the releasing gland.

The nurse is caring for a patient with hyperthyroidism who suddenly develops symptoms related to thyroid storm. What symptoms does the nurse recognize that are indicative of this emergency? -Heart rate of 62 -Blood pressure 90/58 mm Hg -Oxygen saturation of 96% -Temperature of 102ºF

-Temperature of 102ºF Thyroid storm is characterized by the following: 1) high fever (hyperpyrexia), >38.5°C (>101.3°F); 2) extreme tachycardia (>130 bpm); 3) exaggerated symptoms of hyperthyroidism with disturbances of a major system—for example, gastrointestinal (weight loss, diarrhea, abdominal pain) or cardiovascular (edema, chest pain, dyspnea, palpitations); and 4) altered neurologic or mental state, which frequently appears as delirium psychosis, somnolence, or coma.

A nurse is assessing a client after a thyroidectomy. The assessment reveals muscle twitching and tingling, along with numbness in the fingers, toes, and mouth area. The nurse should suspect which complication? -Tetany -Hemorrhage -Thyroid storm -Laryngeal nerve damage

-Tetany Tetany may result if the parathyroid glands are excised or damaged during thyroid surgery. Hemorrhage is a potential complication after thyroid surgery but is characterized by tachycardia, hypotension, frequent swallowing, feelings of fullness at the incision site, choking, and bleeding. Thyroid storm is another term for severe hyperthyroidism — not a complication of thyroidectomy. Laryngeal nerve damage may occur postoperatively, but its signs include a hoarse voice and, possibly, acute airway obstruction.

A member of the military is preparing for a mission that requires scotopic vision. How long does the client have to be in total or near total darkness to increase rod sensitivity to the optimum level?

4 hours Explanation: Dark adaptation is the process by which rod sensitivity increases to the optimum level. This requires approximately 4 hours in total or near-total darkness and is referred to as scotopic vision (night vision).

A client is scheduled for a diagnostic test to measure blood hormone levels. The nurse expects that this test will determine which of the following? -The concentration of a substance in plasma -Details about the size of the organ and its location -The functioning of endocrine glands -The client's blood sugar level

-The functioning of endocrine glands Measuring blood hormone levels helps determine the functioning of endocrine glands. A radioimmunoassay determines the concentration of a substance in plasma. The measurement of blood hormone levels will not reveal a client's blood sugar level. Radiographs of the chest or abdomen determine the size of the organ and its location.

When describing the difference between endocrine and exocrine glands, which of the following would the instructor include as characteristic of endocrine glands? -The secretions are released directly into the blood stream. -The glands contain ducts that produce the hormones. -The secreted hormones act like target cells. -The glands play a minor role in maintaining homeostasis.

-The secretions are released directly into the blood stream. The endocrine glands secrete hormones, chemicals that accelerate or slow physiologic processes, directly into the bloodstream. This characteristic distinguishes endocrine glands from exocrine glands, which release secretions into a duct. Hormones circulate in the blood until they reach receptors in target cells or other endocrine glands. They play a vital role in regulating homeostatic processes.

Which hormone is secreted by the posterior pituitary? -Vasopressin -Calcitonin -Corticosteroids -Somatostatin

-Vasopressin Vasopressin causes smooth muscle, particularly blood vessels, to contract. Calcitonin is secreted by the parafollicular cells of the thyroid gland. Corticosteroids are secreted by the adrenal cortex. Somatostatin is released by the anterior lobe of the pituitary.

The nurse practitioner who assesses a patient with hyperthyroidism would expect the patient to report which of the following conditions? -Fatigue -Dyspnea -Weight loss -Hair loss

-Weight loss Weight loss is consistent with a diagnosis of hyperthyroidism. The other conditions are found in hypothyroidism.

A 35-year-old female client who complains of weight gain, facial hair, absent menstruation, frequent bruising, and acne is diagnosed with Cushing's syndrome. Cushing's syndrome is most likely caused by: -an ectopic corticotropin-secreting tumor. -adrenal carcinoma. -a corticotropin-secreting pituitary adenoma. -an inborn error of metabolism.

-a corticotropin-secreting pituitary adenoma. A corticotropin-secreting pituitary adenoma is the most common cause of Cushing's syndrome in women ages 20 to 40. Ectopic corticotropin-secreting tumors are more common in older men and are commonly associated with weight loss. Adrenal carcinoma isn't usually accompanied by hirsutism. A female with an inborn error of metabolism wouldn't be menstruating.

A nurse is reviewing the laboratory order for a client suspected of having an endocrine disorder. The lab slip includes obtaining cortisol levels. What is being tested? -adrenal function -thyroid function -thymus function -parathyroid function

-adrenal function The adrenal cortex manufactures and secretes glucocorticoids, such as cortisol, which affect body metabolism, suppress inflammation, and help the body withstand stress.

Trousseau sign is elicited -by occluding the blood flow to the arm for 3 minutes with the use of a blood pressure cuff. -by tapping sharply over the facial nerve just in front of the parotid gland and anterior to the ear, causing spasm or twitching of the mouth, nose, and eye. -after making a clenched fist and opening the hand; the palm remains blanched when pressure is placed over the radial artery. -when the foot is dorsiflexed and there is pain in the calf.

-by occluding the blood flow to the arm for 3 minutes with the use of a blood pressure cuff. A positive Trousseau sign is suggestive of latent tetany. A positive Chvostek sign is demonstrated when a sharp tapping over the facial nerve just in front of the parotid gland and anterior to the ear causes the mouth, nose, and eye to spasm or twitch. The palm remaining blanched when the radial artery is occluded demonstrates a positive Allen test. The radial artery should not be used for an arterial puncture. A positive Homans sign is demonstrated when the client reports pain in the calf when the foot is dorsiflexed.

Which diagnostic test is done to determine suspected pituitary tumor? -computed tomography scan -measurement of blood hormone levels -radioimmunoassay -radiographs of the abdomen

-computed tomography scan A computed tomography or magnetic resonance imaging scan is done to detect a suspected pituitary tumor. Radiographs of the chest or abdomen are taken to detect tumors. Radiographs also determine the size of the organ and their location. Measuring blood hormone levels helps determine the functioning of endocrine glands. A radioimmunoassay determines the concentration of a substance in plasma.

A client is being evaluated for hypothyroidism. During assessment, the nurse should stay alert for: -exophthalmos and conjunctival redness. -flushed, warm, moist skin. -systolic murmur at the left sternal border. -decreased body temperature and cold intolerance.

-decreased body temperature and cold intolerance. Hypothyroidism markedly decreases the metabolic rate, causing a reduced body temperature and cold intolerance. Other signs and symptoms include dyspnea, hypoventilation, bradycardia, hypotension, anorexia, constipation, decreased intellectual function, and depression. Exophthalmos; conjunctival redness; flushed, warm, moist skin; and a systolic murmur at the left sternal border are typical findings in a client with hyperthyroidism.

When educating a client with possible glucocorticoid dysfunction, the nurse will explain that the CRH controls the release of ACTH. The best time to perform the blood test to measure peak ACTH levels would be: a) 04:00 to 6:00 PM b) 09:00 to 11:00 PM c) 06:00 to 08:00 AM d) 10:00 to 12:00 AM

06:00 to 08:00 AM Levels of cortisol increase as ACTH levels rise and decrease as ACTH levels fall. There is considerable diurnal variation in ACTH levels, which reach their peak in the early morning (around 6 to 8 AM) and decline as the day progresses.

The nurse is caring for a client whose hearing is impaired due to impacted earwax. The nurse understands that the associated deafness is due to which of the following? a) Conduction disorder b) Sensorineural disorder c) Mixed sensorineural and conduction disorder d) Infection

Conduction disorder Impacted earwax is one cause for conductive hearing loss. It is not a cause of the other answer choices.

How long is the half-life of the hormone aldosterone, which is only 15% protein bound?

25 minutes Explanation: Aldosterone, which is only 15% protein-bound, has a half-life of only 25 minutes. The higher the percentage of protein binding, the longer the half-life will be. Page 757

A client has shingles. What is the best time frame for the initiation of oral and intravenous antiviral drugs after the appearance of a rash to reduce the incidence of ocular complication?

3 days Explanation: Treatment includes the use of oral and intravenous antiviral drugs. Initiation of treatment within the first 72 hours after the appearance of the rash reduces the incidence of ocular complications but not the postherpetic neuralgia. Reference:

A patient has shingles. What is the best time frame for the initiation of oral and intravenous antiviral drugs after the appearance of a rash to reduce the incidence of ocular complication?

3 days.

While working at the triage desk in the local emergency department, which of the following patients is likely having a medical emergency and needs to be seen first?

55 year old truck-driver complaining of sudden onset of ocular pain and blurred vision.

A client has been diagnosed with a cataract and wants to know what percentage of individuals get cataracts. The client is 78 years old. What percentage of clients over the age of 75 have cataracts?

70% Explanation: Cataracts are the most common cause of blindness in the world, accounting for nearly 48 percent of all blindness. Aging is the most common cause of cataracts, with an estimated prevalence of 50 percent in persons between the ages of 65 and 74 years, and increasing to 70 percent in those over 75 years of age.

The nurse educates a client newly diagnosed with diabetes regarding diet and energy requirements. The nurse knows that 1 gram of fat will provide _____ kcal of energy.

9

The nurse educates a patient newly diagnosed with diabetes regarding diet and energy requirements. The nurse knows that one gram of fat will provide _____ kcal of energy.

9 Fat, with 9 kcal/g, has the greatest per gram amount of available energy. Carbohydrates and proteins provide 4 kcal/g.

A 31-year-old female has been recently diagnosed with type 2 diabetes mellitus and is attending a diabetes education class. Which of the following statements by the woman demonstrates an accurate understanding of her health problem? A) "I'll have to control my blood sugars, my blood pressure, and my cholesterol in order to make sure I don't develop sight problems." B) "I'm grieving the fact that I won't be able to get pregnant without causing permanent damage to my vision." C) "It's surprising that sugar in my blood can accumulate on the lens of my eye and cause a loss of sight." D) "I want to avoid going through the treatments for sight restoration that I would need if my diabetes causes damage to my vision."

A

A 48-year-old woman has been found to have nodules on her thyroid that must be biopsied to determine whether or not they are malignant. Which of the following imaging techniques will be most helpful to the surgeon in visualization of the nodes for fine needle aspiration? A) Ultrasound B) Magnetic resonance imaging C) Radioactive scanning using radioiodine D) Radioactive scanning using sestamibi

A

A public health nurse is teaching a health promotion class to a group of older adults at a seniors' center. A woman attending states that, "My husband has got dry macular degeneration, and I don't know what we're going to do when he goes blind from it." How can the nurse best respond to the woman's statement? A) "Vitamins C and E as well as zinc and beta carotene may have some value in slowing the progression of his disease." B) "You should talk to your doctor about the surgical options that might help preserve his sight." C) "If your husband can lower his blood pressure and bad cholesterol, it can possibly slow the progression of his disease." D) "Cataract surgery is showing promise as a way of replacing the part of the eye associated with macular degeneration."

A

Neurotransmitters like catecholamines (e.g., dopamine and epinephrine) have a reaction time of A) milliseconds. B) less than 10 minutes. C) 24 to 36 hours. D) 4 to 7 days.

A

Which of the following best describes the half-life of a highly protein-bound drug such as thyroxine (99% protein bound)? The half-life would be A) much longer to reduce the concentration of the hormone by one half. B) shorter because only a little of the hormone has to be used up to reduce the concentration. C) dependent on which drugs were in the blood system holding on to the hormone. D) dependent on the liver to carry the hormone to its designated target organ

A

Which of the following statements best captures an aspect of the role of hormones in the body? A) Some chemical substances can function as hormones or be integrated with the central and peripheral nervous systems. B) Hormones directly initiate many of the processes that contribute to homeostasis. C) Control of body processes is ensured by the fact that a single hormone can only exert one effect on one specific system or tissue. D) Each hormone that exists in the body is produced by only one specific endocrine gland.

A

Which of the following statements best captures the essence of a second messenger in the mechanisms of the endocrine system? A) Second messengers act as the intracellular signal that responds to the presence of a hormone. B) Endocrine-producing cells must release both a hormone and a second messenger in order to exert a distant effect. C) Second messengers act to supplement hormone effects on cell receptors when the desired hormonal effect must be either increased. D) Second messengers provide an alternative pathway for endocrine effects on a cell that bypass the normal receptor pathways.

A

For which type of hearing loss should the nurse assess in a child who has had recurrent otitis media since birth?

Conductive loss Explanation: Hearing loss that is associated with recurrent otitis media is conductive loss.

A client presents to the clinic complaining, "I have something in my eye." When questioned, the client admits to a scratching and burning sensation and light sensitivity. The health care provider suspects the client has developed: A. Conjunctivitis B. Retinal detachment C. Acute glaucoma D. Corneal edema

A Conjunctivitis causes bilateral tearing, itching, burning, foreign body sensation, and morning eyelash crusting and eye redness. The primary symptom of retinal detachment is painless changes in vision. Commonly, flashing lights or sparks, followed by small floaters or spots in the field of vision, are early symptoms. Attacks of glaucoma (increased intraocular pressure) are manifested by ocular pain, excruciating headache, blurred or iridescent vision, and corneal edema with hazy cornea, dilated (mydriasis), and fixed pupil; with repeated or prolonged attacks, the eye becomes reddened. With corneal edema, the cornea appears dull, uneven, and hazy; visual acuity decreases; and iridescent vision (i.e., rainbows around lights) occurs.

Which of the following signs and symptoms is most indicative of Ménière disease? A. Rotary vertigo and tinnitus B. Nausea and vomiting C. Progressive hearing loss and frequent falls D. Otalgia and recurrent otitis media

A Ménière disease is characterized by fluctuating episodes of tinnitus, feelings of ear fullness, and violent rotary vertigo (room spinning) that often renders the person unable to sit or walk. Nausea and vomiting, hearing loss, and falls may accompany the disease, but these signs and symptoms are less definitive than frequent episodes of rotary vertigo accompanied by tinnitus.

A client presents to the emergency department complaining of loss of part of his vision. An MRI with contrast reveals multiple aneurysms of the circle of Willis. The client is diagnosed with "bitemporal heteronymous anopia." For this client, what effect will this have on his vision? A. With both eyes open, the client has full binocular visual fields B. Bilateral loss of peripheral vision on both sides with a narrow binocular field C. Total, irreversible blindness D. Loss of a quarter of the visual field in both eyes

A The loss of the temporal fields (nasal retina) of both eyes is called bitemporal heteronymous anopia. With both eyes open, the person with bilateral defects still has the full binocular visual field. The loss of the same half-fields in the two eyes is called a homonymous loss, and the abnormality is called homonymous hemianopia. Blindness in one eye is called anopia. If half of the visual field for one eye is lost, the defect is called hemianopia; if a quarter of the field is lost, it is called quadrantanopia. Loss of the temporal or peripheral visual fields on both sides results in a narrow binocular field, commonly called tunnel vision.

10. The physician suspects that a patient under a lot of stress (stimulation of the sympathetic nervous system) is experiencing stress-induced release of vasopressin. Knowing the effects of an activated renin-angiotensin-aldosterone system (RAAS), the nurse should be assessing the patient for which primary nursing diagnosis? A) Fluid volume excess related to retention of water in the body B) Decreased cardiac output related to positional low blood pressure C) Electrolyte imbalance related to retention of potassium D) Nutritional imbalance related to vomiting and diarrhea

A Feedback: Activation of the RAAS results in an increase in vascular tone (elevation of BP) and renal retention of sodium and water. These changes contribute to the physiologic changes that occur with the stress response.

13. An electrician who has been working 14 to 16 hour/day for several weeks in order to ensure the financial survival of his business presents to his family physician with complaints of persistent headaches and insomnia. His family physician attributes the physical complaints to the ongoing stress that the man has been undergoing. Which of the following factors has most likely contributed the most to the man's response to this stressful period of his life? A) The fact that he has previously had difficulty coping with stress B) The fact that he is not physically active on a regular basis C) The fact that his company is heavily in debt D) The fact that he takes diuretic medication for his hypertension

A Feedback: An individual's subjective response to stress is closely linked to previous experiences in dealing with stress and the presence or absence of coping mechanisms. Physical activity, subjective financial status, and diuretic medications would be less likely to have as great an effect.

2. A 61-year-old woman with a 40-pack-a year history of cigarette smoking and a diagnosis of chronic obstructive pulmonary disease (COPD) is experiencing an increase in arterial levels of CO2. This change has been detected in the client's medulla, aortic bodies, and carotid bodies, stimulating the inspiratory center in the medulla oblongata, which has in turn caused the diaphragm to contract more forcefully and increase the respiratory rate. Which of the following terms best describes the role of the inspiratory center? A) Integrator/comparator B) Sensor C) Effector D) Feedback system

A Feedback: An integrator/comparator sums and compares incoming data with a set point. In this case, the inspiratory center has determined the need for increased respiration and stimulated the effector (diaphragm) to respond. The sensor role is in the client's medulla, aortic bodies, and carotid bodies. This overall process is an example of a feedback system, but this does not characterize the particular role of the inspiratory center.

11. Which of the following questions is most likely to be clinically useful in the differential diagnosis of sensorineural versus conductive hearing loss? A) "What medications do you currently take?" B) "What effect is this hearing loss having on your quality of life?" C) "Has your hearing loss developed quickly or more slowly?" D) "Do you ever hear a persistent ringing in your ears?"

A Feedback: Conductive hearing loss occurs when auditory stimuli are not adequately transmitted through the auditory canal, tympanic membrane, middle ear, or ossicle chain to the inner ear. It can be a temporary loss from impacted cerumen. Sensorineural hearing loss occurs with disorders that affect the inner ear, auditory nerve, or auditory pathways to the brain. Numerous drugs have ototoxic potential, a consequence of which is sensorineural hearing loss. The onset and course of hearing loss and the presence or absence of tinnitus do not necessarily help to differentiate between conductive and sensorineural hearing loss. The subjective effect of the client's hearing loss, while a valid concern, does not help with the differential diagnosis.

A client is diagnosed with hyperthyroidism and is exhibiting weight loss, diarrhea, and tachycardia. What does the nurse understand that these clinical manifestations are related to? a) A decrease in the level of glucose b) A decrease in oxygen consumption c) A decrease in sympathetic nervous system activity d) A hypermetabolic state

A hypermetabolic state Many of the manifestations of hyperthyroidism are related to the increase in oxygen consumption and use of metabolic fuels associated with the hypermetabolic state, as well as to the increase in sympathetic nervous system activity that occurs.

18. A counseling psychologist is working with a 30-year-old female client who is experiencing the symptoms of posttraumatic stress disorder (PTSD) following a house fire several months prior. Which of the client's following diagnostic results could most likely be interpreted as a manifestation of PTSD? A) The client has decreased levels of cortisol. B) The client has decreased levels of norepinephrine. C) The client has decreased levels of angiotensin II. D) The client has increased levels of growth hormones.

A Feedback: Decreased cortisol levels are a marker of PTSD and a differentiation from other subtypes of stress. Low levels of norepinephrine and angiotensin II would not accompany any common variant of stress response, and growth hormones are suppressed in response to stress.

16. Which of the following patients, when faced with acute stressful situations, would be considered at highest risk for becoming noncompliant with their medication regimen? A) End-stage renal failure patient experiencing electrolyte imbalances related to having trouble sticking to his prescribed diet B) Teenager whose period is late by 2 weeks and afraid to tell her parents C) Paraplegic patient who forgot his pressure control device and has a small decubitus on his coccyx D) New mother who is breast-feeding every 2 to 3 hours throughout the day and night.

A Feedback: For people with health problems (like ESRD patients), acute stress can interrupt compliance with medication regimens and exercise programs.

7. A 29-year-old woman has been diagnosed with otosclerosis after several years of progressive hearing loss. What pathophysiological process has characterized her diagnosis? A) New spongy bone has been formed around her stapes and oval window. B) Her incus, malleus, and stapes have become disconnected from her normal neural pathways. C) Her temporal bone is experiencing unusually rapid resorption. D) Her tympanic cavity is becoming filled with bone due to inappropriate osteogenesis.

A Feedback: Otosclerosis begins with resorption of bone in one or more foci. During active bone resorption, the bone structure appears spongy and softer than normal (i.e., osteospongiosis). The resorbed bone is replaced by an overgrowth of new, hard, sclerotic bone. Distortion of neural pathways, resorption of the temporal bone, and filling of the tympanic cavity do not occur with otosclerosis.

12. Otitis media (OM), which can occur in any age group, is the most common diagnosis made by health care providers who care for children. Which bacterial pathogen causes the largest proportion of cases that result in sensorineural hearing loss? A) Streptococcus pneumoniae B) Acoustic neuromas C) Haemophilus influenzae D) Parainfluenza

A Feedback: S. pneumoniae is the most common cause of bacterial meningitis that results in sensorineural hearing loss after the neonatal period. Acoustic neuromas are cancers that cause impaired hearing. Parainfluenza and influenza viruses are common viral pathogens in OM.

The nurse is reviewing assessment data on four clients. Select the client at highest risk for developing type 2 diabetes. a) A 40-year-old male who has an active lifestyle b) A 45-year-old obese female with a sedentary lifestyle c) A 60-year-old female with a history of gestational diabetes d) A 10-year-old male whose grandmother has type 2 diabetes

A 45-year-old obese female with a sedentary lifestyle The person most at risk for developing type 2 diabetes is the 45-year-old obese female with a sedentary lifestyle. Other risk factors include family history, over age 40, and history of gestational diabetes. The 60-year-old would have developed it before age 60, if there were additional risk factors.

Which of the following patients would be considered high risk for developing papilledema? Select all that apply. A) A 2-year-old patient who has a shunt placed following delivery where he was diagnosed with hydrocephalus B) A 55-year-old male with substernal chest pain radiating down both arms and experiencing nausea C) A 43-year-old male with diabetes, renal insufficiency, and BP 200/107 D) A 25-year-old motorcyclist who was in an accident and has a potential subdural hematoma E) An 18-year-old female complaining of severe cramps with her menstrual bleeding

A C D

In an adult with acromegaly, a growth hormone (GH)-secreting tumor is suspected. What diagnostic test would be used for this client?

A GH suppression test Explanation: A GH suppression test may be useful to confirm this situation. The other answers are incorrect. Page 764

A patient is ordered desmopressin (DDAVP) for the treatment of diabetes insipidus. What therapeutic response does the nurse anticipate the patient will experience?

A decrease in urine output

A client comes to a scheduled appointment in the endocrine clinic. The primary care physician referred the client, suspecting acromegaly. Knowing the usual testing involved, the nurse should educate the client about which lab/diagnostic procedure?

A suppression test Explanation: Suppression tests are used when hyperfunction of an endocrine organ is suspected. When an organ or tissue is functioning autonomously (i.e., is not responding to the normal negative feedback control mechanisms and continues to secrete excessive amounts of hormone), a suppression test may be useful to confirm the situation. For example, when a GH-secreting tumor is suspected, the GH response to a glucose load is measured as part of the diagnostic workup. Normally, a glucose load would suppress GH levels. However, in adults with GH-secreting tumors (a condition known as acromegaly), GH levels are not suppressed. Stimulation tests are used when hypofunction of an endocrine organ is suspected. The diagnosis of genetic diseases using DNA analysis is rapidly becoming a routine part of endocrine practice. Completion of the human genome sequence has revealed the presence of about 30,000 genes. The considerable interest in the field of genomics (i.e., examination of the DNA) and transcriptomics (i.e., examination of the mRNA) has been complemented by advances in proteomics. Brain scans may be advised after initial suppression testing if a tumor is suspected. Page 764

Which of the following individuals is most likely to be diagnosed with a central vestibular disorder? a) A woman who has ongoing difficulty balancing herself when walking. b) A man who states that he feels car sick whenever he rides in the back seat of a vehicle. c) A man who got up quickly from his bed and sustained an injury after he "blacked out". d) A woman who suffered a loss of consciousness after being struck on the head during a soccer game.

A woman who has ongoing difficulty balancing herself when walking. Central vestibular disorders are marked by a sensation of motion that interferes with balance, but that is mild and constant and chronic in duration. It should be differentiated from postural hypotension, loss of balance from a head injury or motion sickness.

A client exhibiting problems with her thyroid has been scheduled for a radioactive scan. From the following list of clients, which would the nurse question as to whether this would be a safe procedure for this client?

A young female client who has been trying to get pregnant

A patient exhibiting problems with their thyroid has been scheduled for a radioactive scan. From the following list of patients, which would the nurse question as to whether this would be a safe procedure for this patient? An adult patient having an episode of wheezing from allergies. A young female patient who has been trying to get pregnant. A middle-aged male patient with uncontrolled type 2 diabetes mellitus. An elderly patient who has a history of aortic stenosis.

A young female patient who has been trying to get pregnant.

A patient exhibiting problems with their thyroid has been scheduled for a radioactive scan. From the following list of patients, which would the nurse question as to whether this would be a safe procedure for this patient?

A young female patient who has been trying to get pregnant. Explanation: Radioactive iodine therapy is contraindicated in pregnant women because 131I crosses the placenta and can adversely affect the fetal thyroid gland. The other clients would have no contraindication to the substance. Page 765

A patient exhibiting problems with their thyroid has been scheduled for a radioactive scan. From the following list of patients, which would the nurse question as to whether this would be a safe procedure for this patient? a) A middle-aged male patient with uncontrolled type 2 diabetes mellitus. b) An adult patient having an episode of wheezing from allergies. c) A young female patient who has been trying to get pregnant. d) An elderly patient who has a history of aortic stenosis.

A young female patient who has been trying to get pregnant. Radioactive iodine therapy is contraindicated in pregnant women because 131 crosses the placenta and can adversely affect the fetal thyroid gland. The other clients would have no contraindication to the substance.

A patient exhibiting problems with their thyroid has been scheduled for a radioactive scan. From the following list of patients, which would the nurse question as to whether this would be a safe procedure for this patient?

A young female patient who has been trying to get pregnant. Explanation: Radioactive iodine therapy is contraindicated in pregnant women because 131I crosses the placenta and can adversely affect the fetal thyroid gland. The other clients would have no contraindication to the substance.

A new mother brings her infant to the clinic reporting that the child is not sleeping or eating much. Upon assessment, the health care provider notes that the infant's ear canal is reddened with a bulging tympanic membrane. Which other data collected would lead to the diagnosis of acute otitis media (AOM)? Select all that apply. A) "Yes, he has been pulling at his ear." B) "We like to throw him up in the air hoping any water in his ear will drain." C) "He's been very irritable and fussy the past couple of days." D) "When I dropped a pan on the floor, he jumped." E) "He jabbers all the time usually."

A) "Yes, he has been pulling at his ear." C) "He's been very irritable and fussy the past couple of days."

Which of the following clients are at risk for developing and acute episode of angle-closure glaucoma? Select all that apply. A) A 60-year-old female emotionally devastated after divorcing her husband of 35 years B) A soldier ordered to stay in a pitch-black cave to observe enemy militants for an extended period of time C) A person with low dietary intake of lutein, omega-3 fatty acids, and zinc D) A person with a high grade of myopia E) A surgical client who has received many doses of IV atropine to keep heart rate above 50

A) A 60-year-old female emotionally devastated after divorcing her husband of 35 years B) A soldier ordered to stay in a pitch-black cave to observe enemy militants for an extended period of time E) A surgical client who has received many doses of IV atropine to keep heart rate above 50

Which of the following people are at high risk for developing nonulcerative inflammation of the cornea? Select all that apply. A) A welder with inadequate eye protection B) A contact lens wearer who is noncompliant with cleaning and sterilizing the lens C) A female who uses a tanning bed four to five times/week D) A person who touched the fever blister and then rubbed the eye E) A ski patrol who works 12 hours/day in bright, sunny mountain tops

A) A welder with inadequate eye protection C) A female who uses a tanning bed four to five times/week E) A ski patrol who works 12 hours/day in bright, sunny mountain tops

A client presents to the clinic complaining, "I have something in my eye." When questioned, the client admits to a scratching and burning sensation and light sensitivity. The health care provider suspects the client has developed: A) Conjunctivitis B) Retinal detachment C) Acute glaucoma D) Corneal edema

A) Conjunctivitis

As part of the community health department, a nurse is educating a group of diabetic clients about prevention of blindness. Which of the following topics should be covered during this class? Select all that apply. A) Importance of yearly eye exams B) Need to have liver enzymes checked annually C) Tight control of blood glucose levels D) Keep BP below 130.85 (Am. Heart Assoc. Guidelines) E) Never eat dessert when you have eaten pasta for the meal

A) Importance of yearly eye exams C) Tight control of blood glucose levels D) Keep BP below 130.85 (Am. Heart Assoc. Guidelines)

A client presents to the ENT clinic with some vague signs/symptoms. Which complaints would lead the health care provider to suspect the client has otosclerosis? Select all that apply. A) Inability to hear whispering. B) Inability to hear when talking on the telephone. C) When chewing food, sounds are much intensified. D) In a noisy environment has a hard time hearing unless the health care provider speaks directly into the client's ear canal. E) The person's own voice sounds unusually loud.

A) Inability to hear whispering. C) When chewing food, sounds are much intensified. E) The person's own voice sounds unusually loud.

Which of the following signs and symptoms is most indicative of Ménière disease? A) Rotary vertigo and tinnitus B) Nausea and vomiting C) Progressive hearing loss and frequent falls D) Otalgia and recurrent otitis media

A) Rotary vertigo and tinnitus

Parents with a profoundly deaf child ask, "How can you test such a young infant for hearing loss?" The health care provider will likely explain which of the following testing procedures? Select all that apply. A) Tuning fork B) Audioscope C) EEG with auditory brain stem-evoked responses (ABRs) D) Playing music and slowly increasing the sound until response is elicited E) PET scanning

A) Tuning fork B) Audioscope C) EEG with auditory brain stem-evoked responses (ABRs)

A client presents to the emergency department complaining of loss of part of his vision. An MRI with contrast reveals multiple aneurysms of the circle of Willis. The client is diagnosed with "bitemporal heteronymous anopia." For this client, what effect will this have on his vision? A) With both eyes open, the client has full binocular visual fields B) Bilateral loss of peripheral vision on both sides with a narrow binocular field C) Total, irreversible blindness D) Loss of a quarter of the visual field in both eyes

A) With both eyes open, the client has full binocular visual fields

Parents with a profoundly deaf child ask, "How can you test such a young infant for hearing loss?" The health care provider will likely explain which of the following testing procedures? Select all that apply. A. Tuning fork B. Audioscope C. EEG with auditory brain stem-evoked responses (ABRs) D. Playing music and slowly increasing the sound until response is elicited E. PET scanning

A, B, C Tuning forks are used to differentiate conductive and sensorineural hearing loss. Audioscopes can be used to assess a person's ability to hear pure tones at 1000 to 2000 Hz (usual speech frequencies). The ABR uses electroencephalographic (EEG) electrodes and high-gain amplifiers to produce a record of brain wave activity elicited during repeated acoustic stimulations of either or both ears. It involves subjecting the ear to loud clicks and using a computer to analyze nerve impulses as they are processed in the midbrain. Imaging studies such as computed tomography (CT) scans and magnetic resonance imaging (MRI) can be done to determine the site of a lesion (if tumor is suspected) and the extent of damage.

16. While a travel during a vacation via car, the mother notes that her 14-year-old child is getting sick. The mother suspects motion sickness. Which of the following clinical manifestations would confirm this diagnosis? Select all that apply. A) Rapid breathing B) Feeling faint C) Red, flushed face D) Rapid pulse rate E) Severe balance problems

A, B, D Feedback: Autonomic signs (of motion sickness) including lowered BP, tachycardia, and excessive sweating may occur. Hyperventilation produces changes in blood volume and pooling of blood in the lower extremities, leading to postural hypotension and sometimes syncope. Red, flushed face is usually associated with elevated temperature. Severe balance problems are usually associated with irritation or damage of the vestibular end organs.

Which of the following clients are at risk for developing and acute episode of angle-closure glaucoma? Select all that apply. A.A 60-year-old female emotionally devastated after divorcing her husband of 35 years B. A soldier ordered to stay in a pitch-black cave to observe enemy militants for an extended period of time C.A person with low dietary intake of lutein, omega-3 fatty acids, and zinc D. A person with a high grade of myopia E. A surgical client who has received many doses of IV atropine to keep heart rate above 50

A, B, E Manifestations of acute angle-closure glaucoma are related to sudden, intermittent increases in intraocular pressure. These occur after prolonged periods in the dark, emotional upset, and other conditions that cause extensive and prolonged dilation of the pupil. Administration of pharmacologic agents, such as atropine, that cause pupillary dilation (mydriasis) also can precipitate an acute episode. Age-related macular degeneration (AMD) is the most common cause of reduced vision in the elderly. In addition to older age, identifiable risk factors include cigarette smoking, obesity, and low dietary intake of lutein, omega 3 fatty acids, zinc, and vitamins A, C, and E. Persons with high grades of myopia or nearsightedness may have abnormalities in the peripheral retina that predispose to sudden detachment.

19. When assessing a patient diagnosed with brain stem ischemia complaining of vertigo, the nurse will likely observe which of the following clinical manifestations? Select all that apply. A) Inability to coordinate voluntary muscular movements B) Difficulty in articulating words C) Feelings of ear fullness D) Deafness E) Facial weakness

A, B, E Feedback: Inability to coordinate voluntary muscular movements (ataxia), difficulty in articulating words (dysarthria), and facial weakness are usually associated with brain stem ischemia. Fullness in the ear is often a sign of Ménière disease. Deafness is usually not associated with brain stem ischemia.

A new mother brings her infant to the clinic reporting that the child is not sleeping or eating much. Upon assessment, the health care provider notes that the infant's ear canal is reddened with a bulging tympanic membrane. Which other data collected would lead to the diagnosis of acute otitis media (AOM)? Select all that apply. A. "Yes, he has been pulling at his ear." B. "We like to throw him up in the air hoping any water in his ear will drain." C. "He's been very irritable and fussy the past couple of days." D. "When I dropped a pan on the floor, he jumped." E. "He jabbers all the time usually."

A, C AOM is characterized by an acute onset of otalgia (ear pain), fever, and hearing loss. Younger children often have nonspecific signs and symptoms that manifest as ear tugging, irritability, nighttime awakening, and poor feeding. Key diagnostic criteria include ear pain that interferes with activity or sleep, tympanic membrane erythema (redness), and middle ear effusion. A child with otitis media with effusion (OME) may develop delayed speech and language skills.

As part of the community health department, a nurse is educating a group of diabetic clients about prevention of blindness. Which of the following topics should be covered during this class? Select all that apply. A. Importance of yearly eye exams B. Need to have liver enzymes checked annually C. Tight control of blood glucose levels D. Keep BP below 130.85 (Am. Heart Assoc. Guidelines) E. Never eat dessert when you have eaten pasta for the meal

A, C, D Current guidelines recommend that persons with diabetes have yearly eye examinations. Preventing diabetic retinopathy from developing or progressing is considered the best approach to preserving vision. Growing evidence suggests that careful control of blood glucose levels in persons with diabetes mellitus may retard the onset and progression of retinopathy. There also is a need for intensive management of hypertension and hyperlipidemia, both of which have been shown to increase the risk of diabetic retinopathy in persons with diabetes.

A client presents to the ENT clinic with some vague signs/symptoms. Which complaints would lead the health care provider to suspect the client has otosclerosis? Select all that apply. A. Inability to hear whispering. B. Inability to hear when talking on the telephone. C. When chewing food, sounds are much intensified. D. In a noisy environment has a hard time hearing unless the health care provider speaks directly into the client's ear canal. E. The person's own voice sounds unusually loud.

A, C, E The symptoms of otosclerosis involve an insidious hearing loss. Initially, the affected person is unable to hear a whisper or someone speaking at a distance. In the earliest stages, the person's own voice sounds unusually loud, and the sound of chewing becomes intensified. Because of bone conduction, most of these persons can hear fairly well on the telephone, which provides an amplified signal. Many are able to hear better in a noisy environment. The pressure of otosclerotic bone on inner ear structures or the vestibulocochlear nerve (cranial nerve VIII) may contribute to the development of tinnitus, sensorineural hearing loss, and vertigo.

A client with a history of brain tumors that resulted in partial removal of the pituitary gland years ago expresses concern to the health care provider about whether she will be able to breast-feed her infant. Which physiologic function of the pituitary gland facilitates breast milk production?

Prolactin

Which of the following people are at high risk for developing nonulcerative inflammation of the cornea? Select all that apply. A. A welder with inadequate eye protection B. A contact lens wearer who is noncompliant with cleaning and sterilizing the lens C. A female who uses a tanning bed four to five times/week D. A person who touched the fever blister and then rubbed the eye E. A ski patrol who works 12 hours/day in bright, sunny mountain tops

A, C, E In nonulcerative keratitis, all the layers of the epithelium may be affected, but the epithelium remains intact. There are a number of causes of epithelial keratitis, including epidemic keratoconjunctivitis caused by adenoviruses 8 and 19 and ultraviolet (UV) light exposure keratitis. Most cases of UV keratitis occur in welders with inadequate eye protection, but may also occur with tanning booth and other UV lamp exposure, and from sun reflecting off snow. Most cases of herpes keratitis are caused by HSV type 1 (labial [lip] herpes) and is ulcerative in nature. Acanthamoeba keratitis is a rare but sight-threatening complication that typically occurs in people who wear soft contact lens, particularly overnight or without proper disinfection.

4. A 4-year-old boy has had otitis media with effusion (OME) for several weeks, and his condition has recently progressed to acute otitis media (AOM). Which of the following factors could have contributed to his AOM? Select all that apply. A) Reflux of fluid from the boy's nose into his middle ear B) A deficiency in immunoglobulin M C) Accumulation of cerumen in the external acoustic meatus D) Sensorineural deficits in the auditory control apparatus E) Exposure to respiratory virus

A, E Feedback: Reflux via the eustachian tubes, IgG deficiency, and exposure to RSV have all been implicated in the development of AOM. Cerumen accumulation in the outer ear, deficient IgM, and sensorineural deficits are unlikely to contribute to AOM.

37. Which procedure involves an incision of the eardrum to relieve pressure and drain fluid? a. Myringotomy b. Tympanostomy c. Myringorrhexis d. Labyrinthotomy e. Tympanoplasty

A. Myringotomy

39. Which test evaluates bone conduction versus air conduction of sound in one ear at a time? a. Rinne b. PERRLA c. Diopter d. LASIK e. Weber

A. Rinne

16. What is the term for the white of the eye? a. Sclera b. Retina c. Choroid d. Cornea e. iris

A. Sclera

43. What test does the optometrist use to diagnose glaucoma? a. Tonometry b. Ophthalmoscopy c. Fluorescein angiography d. Visual acuity examingation e. Snellen chart examination

A. Tonometry

A client reports sudden, acute left eye pain with blurred vision and a headache on the affected side. The client is most likely experiencing: a) Primary open-angle glaucoma b) Acute-onset wide-angle glaucoma c) Acute angle-closure glaucoma d) Subacute angle-closure glaucoma

Acute angle-closure glaucoma The sudden onset of eye pain, blurred vision, and a headache on the affected side indicate acute angle-closure glaucoma, which is an ophthalmic emergency. -Subacute angle-closure glaucoma manifests as recurrent short episodes of unilateral pain, conjunctival redness, and blurring of vision associated with halos around lights. -Open-angle glaucoma is usually asymptomatic and chronic.

The nurse is caring for a pediatric client whose parent reports that the child has been pulling at her ears, has a fever, and has been crying more than usual. The nurse suspects the client may be experiencing which of the following?

Acute otitis media Explanation: Acute otitis media is characterized by the following key criteria: acute onset of otalgia (or pulling of the ears in an infant), fever, hearing loss, evidence of middle ear inflammation, and middle ear effusion.

The nurse is caring for a pediatric client whose parent reports that the child has been pulling at her ears, has a fever, and has been crying more than usual. The nurse suspects the client may be experiencing which of the following?

Acute otitis media Explanation: Acute otitis media is characterized by the following key criteria: acute onset of otalgia (or pulling of the ears in an infant), fever, hearing loss, evidence of middle ear inflammation, and middle ear effusion.

The nurse understands that an antimicrobial drug is most likely to be prescribed for which condition?

Acute otitis media Explanation: Otitis media with effusion does not usually require treatment with antimicrobial agents; however, acute otitis media is treated with antimicrobials. Barotrauma and otosclerosis are not treated with antimicrobials.

A client tells the health care provider that he developed an acute onset (hours) of vertigo, nausea, and vomiting lasting several days, but he has not had auditory or other neurologic manifestations. The health care provider interprets this as:

Acute vestibular neuronitis

Which of the following vision deficits is a clinician justified in attributing to the normal aging process? A. Conjunctivitis B. Presbyopia C. Strabismus D. Angle-closure glaucoma

B The term presbyopia refers to a decrease in accommodation that occurs because of aging. Conjunctivitis, strabismus, and angle-closure glaucoma are considered abnormal and pathologic in clients of all ages.

A 60 year-old man has long managed his type 1 diabetes effectively with a combination of vigilant blood sugar monitoring, subcutaneous insulin administration, and conscientious eating habits. This morning, however, his wife has noted that he appears pale and clammy and appears to be in a stupor, though he is responsive. She suspects that he has made an error in his insulin administration and that he is experiencing a hypoglycemic episode. Which action should be the wife's first choice?

Administration of 15 to 20 g of glucose in a concentrated carbohydrate source.

The critical care nurse has just admitted a client with diabetic ketoacidosis (DKA) whose blood glucose level is 877 mg/dL. The client's breath has a fruity odor and the client is confused. Which of these does the nurse set as the priority at this time? a) Monitoring for fever b) Orienting the client to the events surrounding his admission c) Administration of intravenous fluids d) Education related to prevention of DKA

Administration of intravenous fluids Goals of care for clients with DKA include administration of insulin and intravenous fluid and electrolyte replacement solutions. A common reason for development of DKA is an infection; monitoring for a fever should not take priority over administration of insulin and fluid replacement.

The eustachian tube connects the nasopharynx and the middle ear. In infants and children with abnormally patent tubes, what enters the eustachian tube when the infant or child cries or blows their nose?

Air and secretions

The eustachian tube connects the nasopharynx and the middle ear. In infants and children with abnormally patent tubes, what enters the eustachian tube when the infant or child cries or blows their nose?

Air and secretions.

Which of the following hormones are derivatives of cholesterol? a) Insulin and glucagon b) Aldosterone and testosterone c) Epinephrine and norepinephrine d) Eicosanoids and retinoids

Aldosterone and testosterone Steroids such as aldosterone and testosterone are a classification of hormones that are derived from cholesterol. Epinephrine and norepinephrine are both amino acids while insulin and glucagon and classified among peptides, polypeptides, proteins, and glycoproteins. Eicosanoids and retinoids consist of fatty acid compounds

What should the nurse teach a client on corticosteroid therapy in order to reduce the client's risk of adrenal insufficiency?

Always have enough medication on hand to avoid running out.

Select the category of hormones that include norepinephrine and epinephrine.

Amines and amino acids

Select the category of hormones that include norepinephrine and epinephrine.

Amines and amino acids Explanation: Hormones can be divided into three categories: (1) amines and amino acids; (2) peptides, polypeptides, proteins, and glycoproteins; and (3) steroids. The amines include norepinephrine and epinephrine. The second category, the peptides, includes polypeptides, proteins, and glycoproteins. The third category consists of the steroid hormones, which are derivatives of cholesterol. Page 757

Select the category of hormones that include norepinephrine and epinephrine. a) Glycoproteins b) Amines and amino acids c) Peptides d) Steroids

Amines and amino acids Hormones can be divided into three categories: (1) amines and amino acids; (2) peptides, polypeptides, proteins, and glycoproteins; and (3) steroids. The amines include norepinephrine and epinephrine. The second category, the peptides, includes polypeptides, proteins, and glycoproteins. The third category consists of the steroid hormones, which are derivatives of cholesterol.

While reviewing the role of glucagon regarding the regulation of blood glucose, the nurse knows that which situation could lead to an inhibition of glucagon release?

An increase in glucose levels

While reviewing the role of glucagon in regards to regulation of blood glucose, the nurse knows which of the following situations could lead to an inhibition of glucagon release? a) An increase in glucose levels. b) Recent intake of large amounts of protein-rich food. c) A sharp decrease in blood glucose concentration. d) Recent strenuous physical activity

An increase in glucose levels. Low blood sugar, intake of protein and strenuous physical activity are associated with glucagon release. Lowered cellular metabolic needs and/or increased glucose levels would inhibit glucagon release

Hormones can be synthesized by both vesicle-mediated pathways and non-vesicle-mediated pathways. What hormones are synthesized by non-vesicle-mediated pathways?

Androgens and estrogens

Hormones can be synthesized by both vesicle-mediated pathways and non-vesicle-mediated pathways. What hormones are synthesized by non-vesicle-mediated pathways?

Androgens and estrogens Explanation: Hormones that are synthesized by non-vesicle-mediated pathways include the glucocorticoids, androgens, estrogens, and mineralocorticoids—all steroids derived from cholesterol. The other answers are incorrect. Page 755

The students are reviewing the anatomy of the eye and where the eye structures are located. Where is the anterior chamber located?

Anterior segment of the globe

The students are reviewing the anatomy of the eye and where the eye structures are located. Where is the anterior chamber located?

Anterior segment of the globe Explanation: The anterior chamber is in the anterior segment of the globe, not the posterior segment of the globe, choroid, or retina.

The students are reviewing the anatomy of the eye and where the eye structures are located. Where is the anterior chamber located? a) Anterior segment of the globe b) Posterior segment of the globe c) Retina d) Choroid

Anterior segment of the globe The anterior chamber is in the anterior segment of the globe, not the posterior segment of the globe, choroid or retina.

The students are reviewing the anatomy of the eye and where the eye structures are located. Where is the anterior chamber located?

Anterior segment of the globe.

22. A physician who specializes in treatment of eye disorders is a(n): a. Optometrist b. Ophthalmologist c. Physiologist d. Psychologist e. Neurologist

B. Ophthalmologist

The number of hormone receptors on a cell may be altered for any of several reasons. The most appropriate response would be:

Antibodies may destroy or block the receptor proteins. Explanation: The number of hormone receptors on a cell may be altered for any of several reasons. Antibodies may destroy or block the receptor proteins. Increased or decreased hormone levels often induce changes in the activity of the genes that regulate receptor synthesis. Decreased hormone levels often produce an increase in receptor numbers by means of a process called up-regulation, increasing the sensitivity of the body to existing hormone levels. Sustained levels of excess hormone often bring about a decrease in receptor numbers by down-regulation, producing a decrease in hormone sensitivity. Page 758

The number of hormone receptors on a cell may be altered for any of several reasons. The most appropriate response would be: a) Altered hormone levels will not induce changes in the activity of the genes that regulate receptor synthesis. b) Decreased hormone levels often produce an increase in receptor numbers by means of a process called the down-regulation. c) Sustained levels of excess hormone often bring about a decrease in receptor numbers by up-regulation. d) Antibodies may destroy or block the receptor proteins.

Antibodies may destroy or block the receptor proteins. The number of hormone receptors on a cell may be altered for any of several reasons. Antibodies may destroy or block the receptor proteins. Increased or decreased hormone levels often induce changes in the activity of the genes that regulate receptor synthesis. Decreased hormone levels often produce an increase in receptor numbers by means of a process called up-regulation, increasing the sensitivity of the body to existing hormone levels. Sustained levels of excess hormone often bring about a decrease in receptor numbers by down-regulation, producing a decrease in hormone sensitivity.

The nurse is explaining to the parent of a 5-year-old that the child has otitis media with effusion (OME), noted by otoscopic exam, following an upper respiratory infection. Unlike acute otitis media (AOM), OME does not require treatment with antibiotics because it is usually which of the following? a) Asymptomatic and requires the administration of antivirals b) Very symptomatic and requires immediate intervention c) Symptomatic and the tympanic membrane is blue d) Asymptomatic and often self-limiting

Asymptomatic and often self-limiting It is very important to distinguish appropriately AOM from OME to ensure the proper treatment and to avoid the unnecessary use of antimicrobials. AOM often presents with sudden onset of ear pain, fever, hearing loss, and middle ear infection, whereas OME often presents as an asymptomatic effusion of the middle ear.

The release of insulin from the pancreatic beta cells can inhibit further release of insulin from the same cells. This is an example of which type of hormone action?

Autocrine

When hormones act locally rather than being secreted into the bloodstream, their actions are termed what?

Autocrine and paracrine Explanation: When hormones act locally on cells other than those that produced the hormone, the action is called paracrine. Hormones also can exert an autocrine action on the cells from which they were produced. The other terms are incorrect. Page 765

When hormones act locally rather than being secreted into the bloodstream, their actions are termed what? a) Autocratic and paracratic b) Autocrine and paracrine c) Localized and influential d) Preventers and inhibitors

Autocrine and paracrine When hormones act locally on cells other than those that produced the hormone, the action is called paracrine. Hormones also can exert an autocrine action on the cells from which they were produced. The other terms are incorrect

A 30-year-old woman has sought care because of her recurrent photophobia, tearing, and eye irritation. During assessment, her care provider asks about any history of cold sores or genital herpes. What is the rationale for the care provider's line of questioning? A) Herpes simplex virus (HSV) conjunctivitis indicated a need for antiviral rather than antibacterial treatment. B) HSV infection of the cornea is a common cause of corneal ulceration and blindness. C) Chronic viral infection of the eyes can result in HSV autoinoculation of the mouth and labia. D) A history of HSV with eye irritation is suggestive of glaucoma

B

7. What is the medical term that means impairment of hearing due to the aging process? a. Tinnitus b. Presbycusis c. Meniere disease d. Otosclerosis e. Myringopathy

B. Presbycusis

A 37-year-old male has survived a logging accident in which the severing of his femoral artery and consequent blood loss resulted in cardiogenic shock. On recovery, one of the deficits that he finds most frustrating is a significant loss of visual acuity. Which is the most likely rationale for his vision damage? A) Decreased cerebral perfusion results in progressive damage to the optic nerve. B) Circulatory collapse causes rapid death of retinal neurons. C) Lack of oxygen results in a distortion of the fovea. D) The visual cortex is susceptible to hypoxic necrosis.

B

A care aide at a long-term care facility has informed a resident physician that an 80-year-old woman's eyes appear to be inflamed and that her eyelids are caked with sticky secretions. The woman has been subsequently diagnosed with posterior blepharitis. Which of the following treatments is the physician likely to initiate? A) Surgical repair of the woman's blocked meibomian glands B) Warm compresses to be applied regularly to her eyes in addition to oral antibiotics C) Regularly scheduled cleansing of the woman's eyes with normal saline D) Intravenous steroids coupled with topical antibiotic ointment

B

A client with a new diagnosis of an endocrine disorder is unclear how the body can control the levels of different hormones over time. Which of the following statements most accurately underlies the dominant regulation process of hormone levels in the body? A) A positive feedback cycle ensures that stable levels of hormones exist in the body over time. B) With input from various sensors, hormone production and release are adjusted based on existing hormone levels. C) The hypothalamus ensures that hormone levels correspond accurately to the diurnal cycle. D) The pituitary gland is genetically programmed to stimulate and inhibit hormone production and/or release based on the needs at different points in the life cycle.

B

A middle-aged woman has acromegaly as a result of a pituitary adenoma that was found and removed when she was a teenager. The physician is suspecting that the tumor has returned and has ordered a diagnostic work-up. A glucose load is ordered. If the tumor has returned, the nurse would expect which of the following results? A) The glucoses load will suppress GH level. B) The growth hormone level will not be suppressed following glucose load. C) The glucose load will raise her serum glucose level to the point of requiring insulin. D) There will be no change in the serum growth hormone level following the glucose load.

B

A patient exhibiting problems with his or her thyroid has been scheduled for a radioactive scan. From the following list of patients, what would the nurse question as to whether this would be a safe procedure for this patient? A) An adult patient having an episode of wheezing from allergies B) A young female patient who has been trying to get pregnant C) A middle-aged male patient with uncontrolled type 2 diabetes mellitus D) An elderly patient who has a history of aortic stenosis

B

As the eyes rotate upward, the upper eyelid reflexively retracts. Which cranial nerve is primarily responsible for this response? A) Cranial nerve I B) Cranial nerve III C) Cranial nerve VI D) Cranial nerve IV

B

During the follicular stage of menstruation, increased estradiol production causes an increase in FSH production. This increase in FSH production by the anterior pituitary gland will have what effect on the follicle? A) The follicle will continue to grow until it can no longer stay in its membrane. B) The follicle will die, which results in a fall of FSH. C) The follicle will continue to grow and produce estradiol. D) The follicle will secrete additional hormones to attract swimming sperm.

B

Following a meal, a woman's blood glucose level has increased. In addition, her pancreas has increased the amount of insulin produced and released. Which of the following phenomena has occurred? A) Increased hormone level according to a negative feedback mechanism B) Adjustment according to the level of the substance a hormone regulates C) Hormone production and release via the positive feedback cycle D) Hypothalamic-pituitary control of hormone levels

B

If the anterior-posterior dimension of the eyeball is too long, the focus point for an infinitely distant target is anterior to the retina. This patient would be diagnosed as having A) hyperopia. B) myopia. C) cycloplegia. D) presbyopia

B

The father of a third grade girl has brought his daughter to a walk-in clinic because he believes the girl has pink eye, which has been going around the students in her class. The nurse at the clinic concurs with the father's suspicion of conjunctivitis. Which follow-up explanation by the nurse is most accurate? A) "The insides of her eyelids have become infected. This often produces severe discomfort." B) "The surfaces of her eyes have bacteria or a virus established, and it's important to maintain good hand hygiene until it goes away." C) "An antibiotic ointment will likely resolve her infection, but pain control will be necessary in the mean time." D) "It's important to aggressively treat this in children, since damage to her sight can result if it's not treated."

B

While working at the triage desk in the local emergency department, which of the following patients is likely having a medical emergency and needs to be seen first? A) A 17-year-old high school student who has a red, itchy eye B) A 55-year-old truck driver complaining of sudden onset of ocular pain and blurred vision C) A 45-year-old school teacher complaining of a red eye that is draining yellow secretions D) An infant with red eyes who is irritable and refusing to eat

B

A client seeks medical care when he wrecks his car because of poor eyesight. At the time of admission, his blood glucose level was 390 mg/dL. The client is diagnosed with diabetes (type 2). The ophthalmologist must perform an urgent intravitreal injection. The nurse explains this to the client by stating the doctor will: A. Just put a couple of drops in each of your eyes B. Put a needle with syringe into your eyeball and inject some medication to decrease active bleeding. C. Remove some of the vitreous from your eye by withdrawing it with a needle/syringe and then strip some of the membranes off your inner eye. D. Use a laser to try to seal off any bleeding vessels in your eyeball.

B People with type 2 diabetes may have retinopathy as a presenting symptom at the time of diagnosis. Intravitreal injections of anti-VEGF agents are also being used to reduce active neovascularization and vitreous hemorrhage. Other treatment strategies for diabetic retinopathy include laser photocoagulation applied directly to leaking microaneurysms and grid photocoagulation with a checkerboard pattern of laser burns applied to diffuse areas of leakage and thickening. Because laser photocoagulation destroys the proliferating vessels and the ischemic retina, it reduces the stimulus for further neovascularization. Vitrectomy may be used for removing vitreous hemorrhage and severing vitreoretinal membranes that develop.

A client has been diagnosed with open-angle glaucoma during routine eye exam. The client has been prescribed a topical b-adrenergic antagonist. Client teaching about how this drug works should include which of the following statements? β-adrenergic antagonists: A. Cause an early decrease in production of aqueous humor by constricting the vessels supplying the ciliary body. B. Lower intraocular pressure by decreasing aqueous humor production C. Reduce the secretion of aqueous humor D. Exert their effects by increasing the effects of acetylcholine, thereby increase aqueous outflow through contraction of the ciliary muscle and pupillary constriction

B Topical β-adrenergic antagonists, which are thought to lower intraocular pressure by decreasing aqueous humor production, are usually the drugs of first choice. The α-adrenergic agonists cause an early decrease in production of aqueous humor by constricting the vessels supplying the ciliary body. Carbonic anhydrase inhibitors reduce the secretion of aqueous humor. Cholinergic drugs exert their effects by increasing the effects of acetylcholine (a postganglionic neurotransmitter in the parasympathetic nervous system) and increase aqueous outflow through contraction of the ciliary muscle and pupillary constriction.

While on a cruise to the Caribbean, a person develops "motion sickness" with associated malaise, and nausea/vomiting. The nurse notes the client's BP is 88/52; pulse is 110; and skin moist with perspiration. The client diagnosis related to the clinical manifestations would most likely be: A. Light-headedness B. Vertigo C. Syncope D. Dizzy

B Vertigo can result from peripheral or central vestibular disorders (proprioception) unrelated to hearing loss. Vertigo is a vestibular disorder in which a unique illusion of motion occurs. Persons with vertigo frequently describe it as a sensation of spinning or tumbling, a "to-and-fro" motion, or falling forward or backward. Light-headedness, faintness, and unsteadiness are different in that the person perceives weakness yet still has a sense of balance. Syncope (loss of consciousness) is not directly associated with the sensation of vertigo. An inability to maintain normal gait may be described as dizziness despite the absence of objective vertigo.

15. Which of the following individuals is most likely to be diagnosed with a central vestibular disorder? A) A man who got up quickly from his bed and sustained an injury after he "blacked out" B) A woman who has ongoing difficulty in balancing herself when walking C) A woman who suffered a loss of consciousness after being struck on the head during a soccer game D) A man who states that he feels car sick whenever he rides in the back seat of a vehicle

B Feedback: Central vestibular disorders are marked by a sensation of motion that interferes with balance but that is mild and constant and chronic in duration. It should be differentiated from postural hypotension, loss of balance from a head injury, or motion sickness.

10. Following a serious bout of bacterial meningitis, the parents of a 14-month-old has noted the child is not responding to verbal commands. The nurse will explain the pathophysiologic principle behind this by educating the patients by which of the following statements? A) "This could be caused by the same organism that caused the meningitis, infecting the child's tympanic membrane." B) "This may be due to a loss of hair cells and damage to the auditory nerve." C) "The ear and the lining of the brain that was infected are all connected together." D) "It is common for meningitis to use up all the natural killer cells and therefore increase the risk of having brain tumors develop."

B Feedback: Deafness or some degree of hearing impairment is the most common serious complication of bacterial meningitis in infants and children. The mechanism causing hearing impairment seems to be suppurative labyrinthitis or neuritis resulting in the loss of hair cells and damage to the auditory nerve. There is no direct connection between the meninges of the brain and the tympanic membrane. Bacterial meningitis is not associated with an increased risk of developing a brain tumor.

17. Based on the assessment results, which of the following hospital patients is most likely to be experiencing the effects of chronic stress? A) A 32-year-old intravenous drug user with a diagnosis of endocarditis B) A 45-year-old paraplegic who is experiencing delayed wound healing of an ischial ulcer C) A 79-year-old woman who has a diagnosis of acute and chronic renal failure D) A 63-year-old man with a diagnosis of chronic obstructive pulmonary disease (COPD) and heart failure (HF)

B Feedback: Delayed wound healing has been associated with chronic stress. Endocarditis, renal failure, COPD, and CHF are not noted as common effects of chronic stress.

19. During a period of stress, the nurse asks the patient to close his eyes and think of a calm, relaxing place where he can feel the wind blowing on his cheek and smell the salty air from the ocean. This is an example of utilizing which type of treatment for stress reduction? A) Hypnosis B) Guided imagery C) Yoga D) Biofeedback

B Feedback: Guided imagery is another technique that is used to achieve relaxation. One method is scene visualization, in which the person is asked to sit back, close the eyes, and concentrate on a scene narrated by the therapist. Whenever possible, all five senses are involved.

11. A patient who has been awaiting the results of a bone marrow biopsy for several days is experiencing stress as a result of uncertainty and the possibility that abnormal cell growth may be detected. A physical examination and blood work would most likely yield which of the following results? A) Increased respiratory rate; increased levels of testosterone B) Increased blood pressure and heart rate; increased antidiuretic hormone (ADH) C) Pupil dilation; increased somatomedin C D) Increased gastrointestinal motility; decreased thyroid-stimulating hormone

B Feedback: Increases in ADH, blood pressure, and heart rate are all associated with the stress response. Testosterone and somatomedin C are decreased in prolonged stress exposure. Gastrointestinal motility is decreased, not increased, during stress.

17. A 70-year-old woman with a diagnosis of benign paroxysmal positional vertigo (BPPV) is receiving teaching from her physician about her diagnosis. The client is eager to avoid future episodes of vertigo and has asked the physician what she can do to prevent future episodes. How can the physician best respond? A) "Unfortunately there aren't any proven treatments for your condition." B) "There are some exercises that I'll teach you to help reorient your inner ear and prevent vertigo." C) "Although they involve some risks, there are some options for ear surgery that can prevent future vertigo." D) "We usually don't actively treat BPPV unless it starts to affect your hearing."

B Feedback: Nondrug therapies for BPPV using habituation exercises and canalith repositioning are successful in many people. Canalith repositioning involves a series of maneuvers in which the head is moved to different positions in an effort to reposition the free-floating debris in the endolymph of the semicircular canals. Surgery is not a noted treatment option, and even in the absence of hearing loss, treatment is warranted.

7. When explaining to a patient admitted for stress-induced supraventricular tachycardia, the nurse will incorporate which of the following statements about what happens in the body as a result of excess stress? A) The primary role of the parasympathetic nervous system is to stimulate the vagus nerve. B) The corticotropin-releasing factor stimulates the release of norepinephrine, which is responsible for "fight-or-flight" reaction to stress. C) Endorphins are released from the brain every time we experience stress. D) The pituitary gland is ultimately responsible for growth, metabolism, and maturation, all of which are important when the body is stressed.

B Feedback: Norepinephrine stimulates the secretion of CRF, and CRF stimulates the release of norepinephrine.

5. An 80-year-old woman is slated for total hip replacement the following day and is experiencing a large amount of stress around her potential surgical outcomes. Which of the following is most likely to be uninvolved in the physiological response to her stress? A) Hypothalamus B) Parathyroid C) Adrenal cortex D) Pituitary gland

B Feedback: The physiology of stress is associated with the hypothalamic-pituitary-adrenal axis. The parathyroid is not noted to be centrally involved in stress response.

A 60-year-old client's long history of poorly controlled hypertension has culminated in a diagnosis of retinal detachment. What type of retinal detachment is this client most likely to have experienced? A. Rhegmatogenous detachment B. Exudative retinal detachment C. Posterior vitreous detachment D. Traction retinal detachment

B Exudative (or serous) retinal detachment results from the accumulation of serous or hemorrhagic fluid in the subretinal space due to severe hypertension, inflammation, or neoplastic effusions. Rhegmatogenous detachment is a full-thickness break ("rhegma") in the sensory retina, with the passage of liquefied vitreous through the break into the subretinal space. Persons with high grades of myopia or nearsightedness may have abnormalities in the peripheral retina that predispose to sudden detachment. In moderate to severe myopia, the anteroposterior length of the eye is increased, and the retina tends to be thinner and more prone to formation of a hole or tear. As a result, there is greater vitreoretinal traction, and posterior vitreous detachment may occur at a younger age than in persons without myopia. Traction retinal detachment occurs with mechanical forces on the retina, usually mediated by fibrotic tissue, resulting from previous hemorrhage (e.g., from diabetic retinopathy), injury, infection, or inflammation. Intraocular surgery such as cataract extraction may produce traction on the peripheral retina that causes eventual detachment months or even years after surgery.

A female client with rheumatoid arthritis has taken high doses of aspirin for several years to control inflammatory pain. Which of the following statements leads the health care provider to suspect the client has developed ototoxicity? A. "I can't go to the movies anymore. It's so noisy, I miss half the words." B. "I've been getting dizzy and light-headed. I seem to have a constant ringing in my ear." C. "I almost got hit by a garbage truck. I didn't hear its backup beeper." D. "When my grandchildren whisper, I can't hear a word they are saying."

B Ototoxicity results in sensorineural hearing loss. Vestibular symptoms of ototoxicity include light-headedness, giddiness, and dizziness; if toxicity is severe, cochlear symptoms consisting of tinnitus or hearing loss occur. The symptoms of drug-induced hearing loss may be transient, as often is the case with salicylates and diuretics, or they may be permanent. Hearing loss in the elderly is further characterized by reduced hearing sensitivity and speech understanding in noisy environments, slowed central processing of acoustic information, and impaired localization of sound sources. High-frequency warning sounds, such as beepers, turn signals, and escaping steam, are not heard and localized, with potentially dangerous results. Clinical measures for hearing loss such as whispered voice tests and finger friction tests are reportedly imprecise and are not reliable methods for screening.

A 60-year-old client's long history of poorly controlled hypertension has culminated in a diagnosis of retinal detachment. What type of retinal detachment is this client most likely to have experienced? A) Rhegmatogenous detachment B) Exudative retinal detachment C) Posterior vitreous detachment D) Traction retinal detachment

B) Exudative retinal detachment

A client has been diagnosed with open-angle glaucoma during routine eye exam. The client has been prescribed a topical b-adrenergic antagonist. Client teaching about how this drug works should include which of the following statements? β-adrenergic antagonists: A) Cause an early decrease in production of aqueous humor by constricting the vessels supplying the ciliary body. B) Lower intraocular pressure by decreasing aqueous humor production C) Reduce the secretion of aqueous humor D) Exert their effects by increasing the effects of acetylcholine, thereby increase aqueous outflow through contraction of the ciliary muscle and pupillary constriction

B) Lower intraocular pressure by decreasing aqueous humor production

14. A widow, who lost her husband a few weeks ago, is having trouble with insomnia and maintaining her normal sleep pattern. When visiting with her health care provider, he suggests a prescription to help regain her normal circadian pattern. This is based on the fact that interruption of sleep-wake cycles can cause which of the following problems? Select all that apply. A) Hallucinations that may result in harm to the individual B) Alterations in the immune function that can result in an infection C) Excess of non-rapid eye movement (NREM) sleep that affects the creativity process D) An increased risk in accidents when sleep deprived similar to those under the influence of alcohol E) Restless leg syndrome due to inability to find a comfortable sleeping position

B, D Feedback: Sleep disorders and alterations in the sleep-wake cycle have been shown to alter immune function, normal hormone secretion, and physical and psychological functioning. However, hallucinations are not associated with sleep deprivations. With sleep disorders, the REM sleep cycle decreases, affecting creativity and forming associations. Restless leg syndrome is not associated with insomnia.

15. Which disorder involves excessive intraocular pressure, caused by a failure of the aqueous humor to drain properly? a. Astigmatism b. Glaucoma c. Esotropia d. Cataract e. Amblyopia

B. Glaucoma

1. Where are the anatomical structures of the labyrinth located? a. Choroid b. Inner ear c. External ear d. Middle ear e. Eustachian tube

B. Inner ear

42. Myopia can be described as: a. Farsightedness b. Nearsightedness c. Astigmatism d. Strabismus e. Esotropia

B. Nearsightedness

50. A visual examination of the eye is called: a. Audiometry b. Ophthalmography c. Tympanometry d. Tuning fork test e. Rinne test

B. Ophthalmography

An adult client has presented to the emergency department reporting symptoms of vertigo. Further assessment reveals that the client is ataxic and has facial weakness. The care team should consider which possible cause of this client's health problem?

Brain stem ischemia Explanation: Central causes of vertigo include brain stem ischemia. When brain stem ischemia is the cause of vertigo, it usually is associated with other brain stem signs such as diplopia, ataxia, dysarthria, or facial weakness. Cerumen impaction and auditory tumors can affect hearing, but not balance. Otitis media does not cause ataxia or facial weakness.

The nurse suggests which action to a mother who is taking her 2-month-old infant on an airplane ride?

Breast-feed or bottle feed the infant during the airplane's ascent or descent. Explanation: Barotrauma can occur when the airplane changes altitudes and pressure in the middle ear is unequal with ambient air pressure. Attempts to equalize air pressure in the middle ear may include swallowing.

Acromegaly is a disorder that is caused by the production of excessive GH in the adult. Because the person cannot grow taller, the soft tissues continue to grow, presenting a very distinctive appearance. What is it that is distinctive in a person with acromegaly? a) Small hands and feet compared to length of arms and legs b) Slanting forehead and a receding lower jaw c) Broad, bulbous nose and a protruding lower jaw d) Protruding lower jaw and forehead

Broad, bulbous nose and a protruding lower jaw When the production of excessive GH occurs after the epiphyses of the long bones have closed, as in the adult, the person cannot grow taller, but the soft tissues continue to grow. Enlargement of the small bones of the hands and feet and of the membranous bones of the face and skull results in a pronounced enlargement of the hands and feet, a broad and bulbous nose, a protruding lower jaw, and a slanting forehead. The other answers are incorrect.

A client with acromegaly comes to the clinic and informs the nurse that she is having a productive cough and a low grade fever. This is the client's fourth visit in one year for the same problem. What condition does the nurse understand results from this client's enlarged cartilaginous structures? a) Rhinitis b) Tuberculosis c) Influenza d) Bronchitis

Bronchitis The cartilaginous structures in the larynx and respiratory tract become enlarged, resulting in a deepening of the voice and tendency to develop bronchitis.

A 21-year-old female is suspected of having inadequate function of her hypothalamic-pituitary-thyroid system. Her care provider is planning to inject thyrotropin-releasing hormone (TRH) and then measure her levels of TSH. Which of the following diagnostic tests is being performed? A) Suppression test B) Radioimmunoassay (RIA) test C) Stimulation test D) Metabolite excretion test

C

A 3-year-old girl has been diagnosed with amblyopia. Which of the following pathophysiological processes is most likely to underlie her health problem? A) The child may have a congenital deficit of rods and/or cones. B) The girl may have chronic bacterial conjunctivitis. C) She may have been born with infantile cataracts. D) The child may have a neural pathway disorder.

C

A 38-year-old woman takes clomiphene, an infertility drug that works by competing with, and thereby blocking, cellular receptors for estrogen. Which of the following statements is most likely to be true of this client? A) Receptors for all other steroid hormones will also be blocked. B) Up-regulation will increase the number of estrogen receptors on each target cell. C) Estrogen will continue to pass freely through the cellular membranes. D) Laboratory tests will reveal an increase in cyclic adenosine monophosphate (cAMP) levels.

C

A 51-year-old woman has been experiencing signs and symptoms of perimenopause and has sought help from her family physician. A deficiency in estrogen levels has been determined to be a contributing factor. Which of the following phenomena could potentially underlie the woman's health problem? A) Sufficient synthesis of estrogen but inadequate vesicle-mediated release B) Inadequate synthesis in the rough endoplasmic reticulum of her ovarian cells C) Insufficient estrogen production within the smooth endoplasmic reticulum of the relevant cells D) A lack of prohormone precursors needed for estrogen synthesis and release

C

A college junior calls his mother, a nurse, complaining of "not being able to see." When questioned further, he describes, "A gray curtain just went down my right visual field. I don't know what to do." The nurse should recognize this symptom as which of the following conditions and have her teenager go to the emergency department immediately. A) Glaucoma B) Strabismus C) Retinal detachment D) Macular degeneration

C

A patient has sought medical attention because of a loss of different half-fields in the two eyes. Knowing the potential causes of this complaint, the nurse anticipates that the physician will order tests looking for A) metal fragments in the eyes. B) hemorrhages in the capillaries of both eyes. C) an enlarging pituitary tumor. D) subarachnoid hemorrhage.

C

An example of a single hormone that can exert effects in different tissues, erythropoietin, made in the kidney stimulates the bone marrow to produce A) platelets. B) natural killer cells. C) red blood cells. D) mast cells.

C

Since steroid hormones are bound to protein carriers for transport, this means A) they are water soluble and circulate freely in the blood. B) they are degraded by enzymes in the blood. C) they are inactive in the bound state. D) they will be converted into a useable form by enzymes in the blood.

C

Which of the following preoperative teaching points related to corneal transplantation is most justified? A) "You should know that there is a significant risk that your body will reject the transplant." B) "The cornea is highly vascular, and therefore you will be at risk for hemorrhage." C) "Your new cornea would come from someone who has recently died." D) "You run a risk of developing a major inflammatory response post-op and will need frequent follow-up appointments."

C

During accommodation, pupillary dilation partially compensates for the reduced size of the retinal image by: A. Thickening the lens B. Contracting the ciliary muscle C. Increasing light entering the pupil D. Narrowing the palpebral opening

C Accommodation is the process whereby a clear image is maintained as gaze is shifted from far to near objects. During accommodation, pupillary dilation partially compensates for the reduced size of the retinal image by increasing the light entering the pupil. Accommodation requires convergence of the eyes, pupillary constriction, and thickening of the lens through contraction of the ciliary muscle, which is controlled mainly by the parasympathetic fibers of the oculomotor cranial nerve (CN III). A third component of accommodation involves reflex narrowing of the palpebral opening during near vision and widening during far vision.

Which pregnant female is at risk of having an infant born with congenital cataracts? A. A first-time mother over the age of 35 B. A pregnant mother with baby number 2 on the way who tested positive for human papillomavirus (HPV) C. A diabetic mother who regulates her blood glucose levels with insulin D. A mother who is 34 weeks' pregnant diagnosed with preeclampsia

C Acquired congenital and infantile cataracts and other developmental defects of the ocular apparatus depend on the total dose of the agent and the embryonic stage at the time of exposure. During the last trimester of fetal life, genetically or environmentally influenced malformation of the superficial lens fibers can occur. Congenital lens opacities may occur in children of diabetic mothers. Mothers' age, HPV status, or preeclampsia does not place them at risk for giving birth to an infant with cataracts.

A 78-year-old female client has been scheduled for outpatient cataract surgery. While taking a presurgery history, which statement by the client correlates to the surgical procedure? A. "One of my eyes has redness and purulent drainage." B. "I had intense eye pain coupled with photosensitivity." C. "I have blurred vision in both my eyes and my visual is distorted." D. "I feel like I have a buildup of pressure in my eyeball."

C Age-related cataracts, which are the most common type, are characterized by increasingly blurred vision and visual distortion. Symptoms of conjunctivitis include a foreign body sensation, a scratching or burning sensation, itching, and photophobia or light sensitivity. Severe pain suggests corneal rather than conjunctival disease. A discharge, or exudate, may be present. It is usually watery, when the conjunctivitis is caused by allergy, a foreign body, or viral infection, and mucopurulent (mucus mixed with pus) in the presence of bacterial or fungal infection. Trauma that causes abrasions of the cornea can be extremely painful. Glaucoma is a chronic, pressure-induced degenerative neuropathy that produces changes in the optic nerve and visual field loss.

In comparison to children with acute otitis media (AOM), those with otitis media with effusion (OME) have: A. Systemic infection B. Earache and fever C. Excess middle ear fluid D. Sensorineural hearing loss

C Otitis media with effusion (OME) is a condition in which the tympanic membrane is intact and there is an accumulation of fluid in the middle ear without fever or other signs or symptoms of infection. Acute otitis media (AOM) is characterized by otalgia (earache), fever, temporary conductive hearing loss, and excess middle ear fluid in combination with signs and symptoms of an acute or systemic infection.

6. When educating the parents of a child who has just had tympanostomy tube insertion, the nurse should provide further teaching if the parents say which of the following statements? A) "I will call the physician if I see that the tube has come out." B) "I will be looking for any drainage coming from the ears." C) "I'm so glad that we can take the child swimming next week when we go on vacation." D) "I'm so glad we had the child tested for allergies prior to having these tubes placed."

C Feedback: After tube insertion, the ears of children with tubes must be kept out of water. All of the other distracters are normal post-op teaching for this procedure. Anytime a device comes out after surgery, the physician should be notified. Routine post-op education includes looking for infection, which in this case could be fever, increase in drainage from the ear, or restlessness. Prior to surgery, most children with recurrent otitis media have allergy testing performed.

2. A 60-year-old man has presented to his family physician following an earache that has become progressively more painful in recent days. Following a history and examination with an otoscope, the man has been diagnosed with otitis externa. Which of the physician's following statements to the man is most accurate? A) "You'll need to avoid getting any water in your ear until you finish your course of antibiotic pills." B) "I'm going to instill some warm water into your ear to flush out debris and bacteria." C) "I'll prescribe some ear drops for you, and in the mean time, it's important not to use ear swabs." D) "This likely happened because your ears aren't draining like they should, but antibiotics that you'll put in your ears will resolve this."

C Feedback: Antimicrobial ear drops are the standard treatment for otitis externa, and the ears must be protected from trauma during infection. Oral antibiotics are not commonly used, and flushing the ears is not indicated for the condition. A lack of normal ear drainage is not part of the etiology of otitis externa.

1. During a procedure to remove impacted cerumen, the nurse should be assessing the patient for which of the following most critical complications? A) Excessive bleeding from the ear B) Pain related to instillation of warm fluids to soften the cerumen C) Symptomatic bradycardia from vagal nerve innervation D) Respiratory distress related to fluid entering bronchiole tubes

C Feedback: Because the external auditory canal is innervated by the auricular branch of the vagus nerve, coughing or even cardiac deceleration can result from stimulation of the canal by cerumen impactions or removal attempts. Since this just involves irrigation using a bulb syringe and warm tap water, bleeding should not occur. Pain may occur but is not a critical complication. Since the fluid is going into the ear canal, there should be no fluid entering the respiratory system.

Knowing that gluconeogenesis helps to maintain blood levels, a nurse should: A.Document weight changes because of fatty acid mobilization B. Evaluate the patient's sensitivity to low room temperatures because of decreased adipose tissue insulation C.Protect the patient from sources of infection because of decreased cellular protein deposits D. all of the above

D. All of the above

9. A medical student is scheduled to undergo a clinical exam of his assessment skills under the supervision of the attending physician. As a result of this stressor, the student has high serum levels of glucocorticoid hormones such as cortisol. Which of the following statements best captures an aspect of the role of glucocorticoid hormones such as cortisol in the physiological response to stress? A) Blood glucose levels are increased in anticipation of energy expenditure. B) The immune system is bolstered in response to an impending threat. C) Osteoblast activity and protein synthesis are suppressed in order to refocus energy. D) Attention, arousal, and respiratory rate are increased in order to prepare for a response.

C Feedback: Cortisol suppresses osteoblast activity, hematopoiesis, protein and collagen synthesis, and immune responses with the goal of preserving energy for a fight-or-flight response. Blood glucose levels are stabilized, not increased, and the immune system is not prioritized in the stress response. Increased attention, arousal, and respiratory rate are the domain of the locus ceruleus-norepinephrine system.

20. As part of the diagnostic workup for a client's long-standing vertigo, a clinician wants to gauge the eye movements that occur in the client. Which of the following tests is the clinician most likely to utilize? A) Romberg test B) Rotational tests C) Electronystagmography (ENG) D) Caloric stimulation

C Feedback: ENG is an examination that records eye movements in response to vestibular, visual, cervical (vertigo triggered by somatosensory input from head and neck movements), rotational, and positional stimulation. With ENG, the velocity, frequency, and amplitude of spontaneous or induced nystagmus and the changes in these measurements brought by a loss of fixation, with the eyes open or closed, can be quantified. The Romberg test, rotational tests, and caloric stimulation do not allow for these data.

15. A patient with Parkinson disease has challenged himself to maintain mobility for longer than the physician predicts. He strives every day to walk 5 to 10 steps further than the day before. This phenomenon, being researched by social psychologists, is known as A) stubbornness. B) assertiveness. C) hardiness. D) positivism.

C Feedback: Hardiness describes a personality characteristic that includes a sense of having control over the environment, a sense of having a purpose in life, and an ability to conceptualize stressors as a challenge rather than a threat.

12. A female patient with a recent diagnosis of systemic lupus erythematosus (SLE) has been told that this is an autoimmune disease whereby the immune system is attacking the body's cells and tissues. She knows that she has inflammation and tissue damage. She asks her nurse to explain "What cells in the body are triggering this inflammation?" The nurse responds that A) mainly the adrenocorticoids like cortisone are responsible for all your inflammation. B) mineralocorticoids like aldosterone usually begin the process of inflammation. C) the lymphocytes that migrate to the brain, where they secrete cytokines, which trigger inflammation. D) primarily, this acute stress reaction is associated with stimulation of the autonomic nervous system that causes stiffness in the joints.

C Feedback: Immune cells such as monocytes and lymphocytes can penetrate the blood-brain barrier and take up residence in the brain, where they secrete chemical messengers called cytokines that influence the stress response. This triggers inflammation.

13. As part of a health promotion initiative, a public health nurse is meeting with a group of older adult residents of an assisted living facility. Which of the following teaching points about hearing loss in the elderly should the nurse include in the teaching session? A) "It is actually a myth that seniors have worse and worse hearing as they age." B) "Most hearing loss in older adults is the result of easily fixed problems, such as impacted ear wax." C) "Experts don't quite know what causes seniors to lose their hearing with age, but drugs like aspirin can contribute to the problem." D) "With older adults, the goal is to adjust lifestyle to accommodate diminished hearing rather than trying to treat the hearing loss itself."

C Feedback: Presbycusis is an identified phenomenon that is thought to be multifactorial, and ototoxic drugs are known to contribute significantly to hearing loss in the elderly. Sensorimotor etiologies are most common, and while lifestyle modifications are often necessary, this does not rule out treatment of the hearing loss.

20. A health care professional has recommended biofeedback to a client as a method of dealing with the high levels of stress in her life. Which of the following explanations best characterizes an aspect of biofeedback treatment? A) "You might be asked to use an electrocardiogram as part of the therapy." B) "The goal is to make you aware of your physiological processes." C) "You'll hopefully be able to gain control over skeletal muscle contractions." D) "You'll become aware of the increased skin temperature that accompanies anxiety."

C Feedback: The goal of biofeedback is to gain control over muscle contractions that accompany anxiety and tension. Electrocardiograms are not used in biofeedback, and control, rather than simply awareness, is the ultimate goal. Skin temperature decreases, not increases, with anxiety.

8. A student is attempting to trace the feedback cycle involved in the stress response. Which of the following neural structures is thought to be the central integrating site for the stress response? A) Hypothalamus B) Cerebral cortex C) Locus ceruleus D) Reticular formation

C Feedback: The locus ceruleus (LC) produces norepinephrine (NE), which initiates the autonomic syndrome known as "fight-or-flight." The LC-NE system is connected to the hypothalamus, the limbic system, the hippocampus, and the cerebral cortex, which carry out the specific functions of the stress response.

3. Which of the following physiological processes would be considered a positive feedback mechanism? A) The release of antidiuretic hormone (ADH) from the posterior pituitary gland B) Shivering in response to low environmental temperature C) The platelet aggregation mechanism for closing minute ruptures in very small blood vessels during accidental injury D) Increased production of white blood cells (WBCs) in response to a microorganism

C Feedback: The release of hormones during labor increases rather than mitigates a physiological system. Specifically, uterine contraction stimulates the production of other relevant hormones that cause temporary instability that culminates in childbirth. ADH counters the potential instability of insufficient hydration and/or blood pressure, much as shivering is an attempt to counter low temperature. Increased production of WBCs is a response to the potential homeostatic instability of an infectious process.

26. Which medical term describes a type of vision? a. Blepharoptosis b. Ophthalmalgia c. Otodynia d. Amblyopia e. Dacryorrhea

D. Amblyopia

6. A 45-year-old woman with a diagnosis of shingles is experiencing an acute onset of severe neuropathic pain. Which stage of Selye's characterization of stress response is the woman most likely experiencing at the moment? A) General adaptation syndrome (GAS) B) Exhaustion C) Alarm D) Resistance

C Feedback: The stimulation of the sympathetic nervous system, such as that during an episode of pain, characterizes the alarm stage. Resistance and exhaustion would likely follow, and GAS encompasses the whole continuum, not just this particular stage.

1. An 81-year-old male patient who has a diagnosis of orthostatic hypotension is experiencing an episode of particularly low blood pressure. The man's body has responded by increasing levels of angiotensin II in the bloodstream, a hormone that decreases the glomerular filtration rate in the kidneys and contributes to an increase in blood pressure. Which of the following phenomena best describes what has occurred? A) Positive feedback B) Adaptation C) Negative feedback D) Homeostasis

C Feedback: his regulation and attempt to normalize blood pressure are an example of a negative feedback mechanism, in which physiological processes result in the maintenance of homeostasis. This process of normalization is not an example of positive feedback, and homeostasis is the goal rather than the process. This process involves adaptation, but negative feedback is a more precise characterization of the process.

A 78-year-old female client has been scheduled for outpatient cataract surgery. While taking a presurgery history, which statement by the client correlates to the surgical procedure? A) "One of my eyes has redness and purulent drainage." B) "I had intense eye pain coupled with photosensitivity." C) "I have blurred vision in both my eyes and my visual is distorted." D) "I feel like I have a buildup of pressure in my eyeball."

C) "I have blurred vision in both my eyes and my visual is distorted."

34. A condition in which blood and other fluids leak from intraocular vessels and destroy the visual cells leading to permanent visual loss is called: a. Presbyopia b. Glaucoma c. Macular degeneration d. Cataracts e. Achromatopsia

C. Macular degeneration

45. Visual examination of the ear is called: a. Audiometry b. Ophthalmoscopy c. Otoscopy d. Endoscopy e. Tympanometry

C. Otoscopy

Which complication of acromegaly can be life threatening? a) Splayed teeth result in impaired chewing b) Bone overgrowth causes arthralgias c) Cardiac structures increase in size d) Vertebral changes result in kyphosis

Cardiac structures increase in size While all the complications can exist, it is the enlargement of the heart and accelerated atherosclerosis that may lead to an early death. The teeth become splayed, causing a disturbed bite and difficulty in chewing. Vertebral changes often lead to kyphosis, or hunchback. Bone overgrowth often leads to arthralgias and degenerative arthritis of the spine, hips, and knees. Virtually every organ of the body is increased in size.

Damage to which part of the eye is responsible for loss of color vision?

Central retina Explanation: The blood supply for the retina is derived from the central retinal artery, which supplies blood flow for the entire inside of the retina. As with a camera, the simple lens system of the eye inverts the image of the external world on each retina. Errors in refraction occur when the visual image is not focused on the retina because of individual differences in the size or shape of the eyeball or cornea.

The nursing instructor is teaching a class on diabetes and discusses complications of the disease. She further states that diabetic retinopathy is one of the leading causes of blindness. What does she tell the students are major risk factors for developing diabetic retinopathy? Select all that apply.

Chronic hyperglycemia Hypertension Smoking

The nurse is assessing a patient's risk for sensorineural hearing loss. Which of the following places the patient at greatest risk?

Chronic noise exposure

The nurse is assessing a client's risk for sensorineural hearing loss. Which condition or situation places the client at greatest risk?

Chronic noise exposure Explanation: Chronic exposure to noise is a risk factor for sensorineural hearing losss. Impacted earwax and otitis media are risk factors for conductive hearing loss.

The nurse is planning to collect a 24-hour urine sample for hormone assay. In which situation does the nurse collaborate with the health care provider to find an alternate type of testing?

Client has anuria.

The nurse is caring for a client whose hearing is impaired due to impacted earwax. The nurse understands that the associated deafness is due to which of the following?

Conduction disorder

The nurse is caring for a client whose hearing is impaired due to impacted earwax. The nurse understands that the associated deafness is due to:

Conduction disorder Explanation: Impacted earwax is one cause for conductive hearing loss. It is not a cause of the other answer choices.

A client presents to the clinic complaining, "I have something in my eye." When questioned, the client admits to a scratching and burning sensation and light sensitivity. The health care provider suspects the client has developed:

Conjunctivitis Explanation: Conjunctivitis causes bilateral tearing, itching, burning, foreign body sensation, and morning eyelash crusting and eye redness. The primary symptom of retinal detachment is painless changes in vision. Commonly, flashing lights or sparks, followed by small floaters or spots in the field of vision, are early symptoms. Attacks of glaucoma (increased intraocular pressure) are manifested by ocular pain, excruciating headache, blurred or iridescent vision, and corneal edema with hazy cornea, dilated (mydriasis), and fixed pupil; with repeated or prolonged attacks, the eye becomes reddened. With corneal edema, the cornea appears dull, uneven, and hazy; visual acuity decreases; and iridescent vision (i.e., rainbows around lights) occurs.

Where in the eye does the major refraction of light begin?

Convex corneal surface Explanation: In the eye, the major refraction of light begins at the convex corneal surface. Further refraction occurs as light moves from the posterior corneal surface to the aqueous humor, from the aqueous humor to the anterior lens surface, from the anterior lens surface to the posterior lens surface, and from the posterior lens surface to the vitreous humor.

Which hormone requires a carrier protein for transport through the blood?

Cortisol

Select the hormone that requires protein as a transport carrier. a) Growth hormone b) Insulin c) Cortisol d) Parathormone

Cortisol Specific carrier proteins synthesized in the liver carry steroid hormones, such as cortisol, and thyroid hormone. The other listed options are peptide hormones and protein hormones that are water soluble and usually circulate unbound in the blood. (less)

A client is diagnosed with adrenocorticotropic hormone deficiency (ACTH) and is to begin replacement therapy. Regarding which type of replacement will the nurse educate the client? a) Replacement therapy with synthetic thyroid hormone b) Replacement therapy with prolactin c) Cortisol replacement therapy. d) Growth hormone replacement therapy

Cortisol replacement therapy. Cortisol replacement is started when ACTH deficiency is present; thyroid replacement when TSH deficiency is detected; and sex hormone replacement when LH and FSH are deficient. GH replacement is indicated for pediatric GH deficiency, and is increasingly being used to treat GH deficiency in adults.

A patient's chart documents the finding of cholesteatoma. The nurse interprets this to mean that the patient has which of the following? a) Pain in the middle ear b) Drainage from the ear c) Build up of cerumen d) Cystlike mass in the middle ear

Cystlike mass in the middle ear Cholesteatoma is a cystlike mass of the middle ear that often extends to involve the temporal bone.

A 32-year-old man is complaining of burning, itching, photophobia, and severe pain in his right eye after swimming in the ocean. To determine that the eye condition was a corneal rather than a conjunctival disease, which of the following would be the distinguishing symptom? A) Burning B) Itching C) Photophobia D) Severe pain

D

A 4-month-old infant and his mother are at an appointment with a pediatrician to follow up his nonaccommodative strabismus and to determine a treatment plan. Which of the following treatments is most likely to prevent future loss of vision? A) Prescribing glasses once the infant is 6 months of age B) Use of beta-adrenergic blockers and latanoprost eye drops C) Regularly scheduled eye exams and monitoring of self-correction of his eyes D) Surgical correction of the musculature

D

A 46-year-old male has presented to the emergency department because of the eye pain, severe headache, and blurred vision that have followed an eye exam at an optometrist's office earlier in the day. The client tells the triage nurse that he received eye drops during the exam "to keep my pupils wide open." What differential diagnosis will the care team first suspect? A) Infectious conjunctivitis B) Keratitis C) Corneal trauma D) Angle-closure glaucoma

D

A client with a history of an endocrine disorder exhibits signs and symptoms of hormone deficiency. Which of the following processes would the client's care team most likely rule out first as a contributing factor? A) The client's target cells lack sufficient receptors for the hormone in question. B) Hormone production is sufficient, but affinity on the part of the target cells is lacking. C) The process of down-regulation has resulted in decreased hormone sensitivity. D) Up-regulation has increased the sensitivity of the body to particular hormone levels.

D

When explaining about structural classifications to a group of students, the instructor discusses the peptides and proteins. They talk about small hormones and hormones as large and complex as growth hormone (GH), which has approximately how many amino acids involved? A) 50 amino acids B) 100 amino acids C) 150 amino acids D) 200 amino acids

D

Which of the following statements best captures the relationship between the hypothalamus and the pituitary gland as it relates to endocrine function? A) The hypothalamus directly measures the levels of most hormones throughout the body and inhibits or stimulates the pituitary accordingly. B) The pituitary gland coordinates and dictates the release of hormones from the hypothalamus that act on their intended target cells. C) The pituitary gland and hypothalamus have two-way communication that mediates the signals from neuronal inputs. D) The hypothalamus receives input from numerous sources throughout the body and directs the pituitary to then control many target glands and cells.

D

20. ENT, is the abbreviation for: a. Eyes, nose, and throat b. Eyes, nose, and tympanic membrane c. Ears, nose and tympanic membrane d. Ears, nose and throat e. Eyelids, nose, and throat

D. Ears, nose and throat

An elderly woman has been diagnosed with macular degeneration following a visit to an ophthalmologist. Which of the woman's following statements best demonstrates an accurate understanding of her new diagnosis? A."I suppose this goes to show that I should have controlled my blood pressure better." B. "I think this is something that I might have caught from my husband." C."My friend had this problem and a transplant did wonders for her vision." D. "I suppose that this may be one of the things that happen when you get older."

D Although some risk factors have been identified for macular degeneration, most diagnoses are attributed to increased age. The pathogenesis does not involve infection or hypertension, and a corneal transplant is not a recognized treatment modality.

One of the causes of conductive hearing loss is: A. Sudden loud noise B. Ototoxic medication C. Auditory nerve damage D. Excess middle ear fluid

D Conductive hearing loss occurs when auditory stimuli are not adequately transmitted through the auditory canal, tympanic membrane, middle ear, or ossicle chain to the inner ear; it is usually the result of middle ear fluid or infections. Other causes include ear canal cerumen, or foreign bodies, tympanic membrane thickening or damage, or bony structure (ossicles and oval window) damage of the middle ear caused by otosclerosis or Paget disease. Sensorineural deafness occurs when sound waves are conducted to the inner ear but abnormalities of the cochlear apparatus or auditory nerve decrease or distort the transfer of information to the brain. Causes of sensorineural deafness include genetic, infectious, traumatic loud noises, and ototoxic factors.

Diabetic and hypertensive retinopathy are both characterized by the appearance of: A. Macular edema B. Cloudy corneas C. Microinfarctions D. Intraretinal hemorrhages

D Diabetic (background) retinopathy involves thickening of the retinal capillary walls, ruptured capillaries, microaneurysms, intraretinal hemorrhages, cotton-wool exudates, and microinfarcts. Decreased vision in persons with background retinopathy is commonly due to macular edema secondary to leakage of plasma from the small macular blood vessels. Chronic systemic hypertension results in intraretinal arteriole thickening, reduced capillary perfusion pressure, microaneurysms, intraretinal hemorrhages, cotton-wool exudates, and edema. Malignant hypertension causes localized optic disk edema (papilledema) produced by escaped fluid. Cloudy corneas are characteristic of anterior chamber disorders rather than retinopathy.

A client develops fever, headache, and burning/itching in the periorbital area. After a few days, a vesicular rash appears around the eyelid margins. The health care provider will likely prescribe: A. Topical antimicrobial for infection caused by overuse of contact lens B. Oral antibiotics to treat chlamydial infection C. Topical mast cell stabilizer to treat allergies D. Antiviral medication for herpes zoster ophthalmicus

D Herpes zoster ophthalmicus usually presents with malaise, fever, headache, and burning and itching of the periorbital area. These symptoms commonly precede the ocular eruption by a day or two. The rash, which is initially vesicular, becomes pustular and then develops crusts. Treatment includes the use of oral and intravenous antiviral drugs. Initiation of treatment within the first 72 hours after the appearance of the rash reduces the incidence of ocular complications but not the postherpetic neuralgia. Chlamydial conjunctivitis is commonly spread by contact with genital secretions. It is treated with antimicrobial medications. Causes of ulcerative keratitis include infectious agents, exposure to trauma, and use of extended-wear contact lenses. The first manifestations of recurrent herpes keratitis are irritation, photophobia, and tearing. A history of fever blisters or other herpetic infection is often noted. Allergic conjunctivitis encompasses a spectrum of conjunctival conditions usually characterized by itching. Allergic conjunctivitis also has been successfully treated with topical mast cell stabilizers, histamine type 1 (H1) receptor antagonists, and topical nonsteroidal anti-inflammatory drugs.

During a routine 2-month checkup at the pediatric clinic, a mother expresses concern that her son looks "cross-eyed." She asks if she need to put patches over his good eye. Assessment reveals full eye movement, and the child uses each eye independently. The health care provider explains that the best treatment for the infant's eye problem is: A. A prescription for drops to put in the eyes twice/day. B. "Pretend like you are going to poke him in the eye so that he will blink more and tighten up some muscles." C. To buy some prescription eye glasses so that lazy eye will get stronger. D. To prepare for some surgery to correct this problem early on to correct the eye muscle disorder.

D The disorder may be nonaccommodative, accommodative, or a combination of the two. Infantile esotropia is the most common cause of nonaccommodative strabismus. It occurs in the first 6 months of life, with large-angle deviations, in otherwise developmentally and neurologically normal infants. Eye movements are full, and the child often uses each eye independently to alter fixation (cross-fixation). Infantile esotropia is usually treated surgically by weakening the medial rectus muscle on each eye while the infant is under general anesthesia. Recurrences are common with infantile esotropia, and multiple surgeries are often required.

Frustrated by her worsening tinnitus, a 55-year-old female client has sought care. Which of the following teaching points should the clinician provide to the client? A."I know this can be very difficult to live with, but it normally fades over time." B."I will prescribe some medication that will probably help quite well." C. "This might be a sign of a more serious neurologic problem that we will assess for." D. "Initially, there are some changes in your diet that you should implement."

D Tinnitus is not necessarily self-limiting, although it is not normally an indicator of neurologic disease. Pharmacologic treatments are limited, but dietary changes have met with success in many clients. (coffee, salt, saturated fats, sugars, alcohol)

11. A rupture of the cornea is called: a. Choroidorrhexis b. Choroiditis c. Keratitis d. Keratorrhexis e. Tympanorrhexis

D. Keratorrhexis

28. What is an inflammation of the middle ear called? a. Labyrinthitis b. Tympanitis c. Mastoiditis d. Otitis media e. Conjunctivitis

D. Otitis media

3. What is an usual intolerance and sensitivity to light called? a. Chromatophilia b. Achromatopsia c. Chromatopsia d. Photophobia e. Esotropia

D. Photophobia

48. Impairment of vision due to old age is known as: a. Anisocoria b. Emmetropia c. Diplopia d. Presbyopia e. Esotropia

D. Presbyopia

4. What structure is composed of nerve endings and responsible for the reception and transmission of light impulses? a. Sclera b. Choroid c. Iris d. Retina e. Cornea

D. Retina

8. Distraught at the persistent ringing in his ears and his inability to alleviate it, a 50-year-old man has visited his health care provider. After diagnostic testing, no objective cause (like impacted cerumen or vascular abnormality) was found. Given these testing results, which of the following teaching points by the care provider is most appropriate? A) "This is most often the result of a psychological disturbance, and therapy is often useful in relieving tinnitus." B) "There are many drugs such as blood pressure pills, relaxants, heart medications, and antihistamines that can cause tinnitus." C) "A specialist can listen with a sensitive microphone to determine whether you are actually hearing these sounds." D) "There are some treatments like tinnitus retraining therapy, which includes the extended use of low-noise generators, which has shown good success."

D Feedback: Current treatment modalities for tinnitus address the symptoms of the problem rather than curing the underlying etiology. While therapy can be of some use, it is inaccurate to characterize tinnitus as a psychological disturbance. Medications, including antihistamines, anticonvulsant drugs, calcium channel blockers, benzodiazepines, and antidepressants, have been used for tinnitus alleviation; they are not implicated as a cause. While listening to differentiate between objective and subjective tinnitus is possible, the absence of objective sounds does not mean that tinnitus does not exist, rather that it is subjective. The use of tinnitus retraining therapy, which includes directive counseling and extended use of low-noise generators to facilitate auditory adaptation to the tinnitus, has met with considerable success.

18. A 30-year-old woman has presented to her family doctor complaining of three distressing episodes over the last several months during which she got extremely dizzy, had loud ringing in her ears, and felt like her ears were full of fluid. She states that her hearing diminishes, and she feels nauseous during these episodes. What diagnosis is the physician most likely to first suspect? A) Acute otitis media B) Acute vestibular neuronitis C) Benign paroxysmal positional vertigo (BPPV) D) Ménière disease

D Feedback: Ménière disease is characterized by fluctuating episodes of tinnitus, feelings of ear fullness, and violent rotary vertigo that often renders the person unable to sit or walk. There is a need to lie quietly with the head fixed in a comfortable position, avoiding all head movements that aggravate the vertigo. Symptoms referable to the autonomic nervous system, including pallor, sweating, nausea, and vomiting, usually are present. The more severe the attack, the more prominent are the autonomic manifestations. A fluctuating hearing loss occurs with a return to normal after the episode subsides. Her symptomatology is not characteristic of AOM, acute vestibular neuronitis, or BPPV.

5. Which of the following would be considered an abnormal finding when the nurse practitioner uses an otoscope to look at a toddler's ear? The tympanic membrane is described as A) "transparent." B) "a shallow, oval cone pointing inward toward apex." C) "small, whitish cord seen traversing the middle ear from back to front." D) "yellow, amber discoloration noted."

D Feedback: Otoscopic signs of middle ear effusion will be seen in the tympanic membrane as a yellow, amber discoloration. All of the other distracters are normal findings.

4. An occupational health officer who works in the context of a large police force is attempting to understand the role that stress may play in the health of his clients. According to Selye, which of the following statements best captures an aspect of the phenomenon of stress? A) The alarm stage involves the release of cortisol and catecholamines. B) The nature of a stress response is determined by the objective severity of the stressor. C) Systemic illnesses can sometimes result from the resistance stage of stress response. D) Periods of stress can be developmentally positive or negative.

D Feedback: Selye noted that stress can result in positive growth and development, and that stress is not unanimously detrimental to health and development. The alarm stage is associated with the release of cortisol and catecholamines, and a stress response is dependent on properties of the stressor and the individual's conditioning; the severity is not objectively determined. Illness is often a consequence of the exhaustion stage.

3. During descent, an airplane passenger is complaining that his "ears are plugged." What aspect of the structure and function of the ear best accounts for the passenger's complaint? A) The inner ear adjusts its volume in response to atmospheric pressure, increasing during low pressure and decreasing in high pressure. B) The eustachian tubes must remain patent to equalize pressure between the middle ear and inner ear. C) The tympanic membrane is selectively permeable in order to accommodate pressure changes, and this capacity is often impaired during upper respiratory infections. D) Air must be able to flow between the middle ear and nasopharynx in order to accommodate pressure changes.

D Feedback: The eustachian tubes between the middle ear and nasopharynx must be patent to allow for changes in atmospheric pressure. Pressure is not accommodated by changing the volume of the middle ear, and the tympanic membrane is not selectively permeable to air.

An elderly woman has been diagnosed with macular degeneration following a visit to an ophthalmologist. Which of the woman's following statements best demonstrates an accurate understanding of her new diagnosis? A) "I suppose this goes to show that I should have controlled my blood pressure better." B) "I think this is something that I might have caught from my husband." C) "My friend had this problem and a transplant did wonders for her vision." D) "I suppose that this may be one of the things that happen when you get older."

D) "I suppose that this may be one of the things that happen when you get older."

Diabetic and hypertensive retinopathy are both characterized by the appearance of: A) Macular edema B) Cloudy corneas C) Microinfarctions D) Intraretinal hemorrhages

D) Intraretinal hemorrhages

During a routine 2-month checkup at the pediatric clinic, a mother expresses concern that her son looks "cross-eyed." She asks if she need to put patches over his good eye. Assessment reveals full eye movement, and the child uses each eye independently. The health care provider explains that the best treatment for the infant's eye problem is: A) A prescription for drops to put in the eyes twice/day. B) "Pretend like you are going to poke him in the eye so that he will blink more and tighten up some muscles." C) To buy some prescription eye glasses so that lazy eye will get stronger. D) To prepare for some surgery to correct this problem early on to correct the eye muscle disorder.

D) To prepare for some surgery to correct this problem early on to correct the eye muscle disorder.

24. What sstructure of the inner ear helps maintain body equilibrium? a. Cochlea b. Stapes c. Incus d. Semicircular canal e. Tympanic membrane

D. Semicircular canal

A patient experiences an increase in cortisol as a result of Cushing's disease. Which of the following hormonal responses demonstrates the negative feedback mechanism?

Decreased adrenocorticotropic hormone (ACTH)

A patient experiences an increase in cortisol as a result of Cushing's disease. Which of the following hormonal responses demonstrates the negative feedback mechanism?

Decreased adrenocorticotropic hormone (ACTH) Explanation: Negative feedback occurs when secretion of one hormone causes a reduction in the secretion of the hormone that stimulates production of the first hormone. In this case, ACTH manufactured by the anterior pituitary gland would normally stimulate the release of cortisol, but with the increase of cortisol produced by the secreting tumor, enough cortisol already floods the system that there should be a reduction in the ACTH level. Page 762

A patient experiences an increase in thyroid hormone as a result of a thyroid tumor. Which of the following hormonal responses demonstrates the negative feedback mechanism? Decreased thyroid-stimulating hormone (TSH) Increased follicle-stimulating hormone Decreased adrenocorticotropic hormone Increased thyrotropin-releasing hormone

Decreased thyroid-stimulating hormone (TSH)

A patient experiences an increase in thyroid hormone as a result of a thyroid tumor. Which of the following hormonal responses demonstrates the negative feedback mechanism?

Decreased thyroid-stimulating hormone (TSH) Explanation: Negative feedback occurs when secretion of one hormone causes a reduction in the secretion of the hormone that stimulates production of the first hormone. In this case, TSH, which is manufactured by the anterior pituitary gland, would normally stimulate release of thyroid hormones, but with the increase of those hormones by the secreting tumor, enough thyroid hormones flood the system that there should be a reduction in TSH levels. Page 764

A patient experiences an increase in thyroid hormone as a result of a thyroid tumor. Which of the following hormonal responses demonstrates the negative feedback mechanism? a) Increased follicle-stimulating hormone b) Decreased adrenocorticotropic hormone c) Decreased thyroid-stimulating hormone (TSH) d) Increased thyrotropin-releasing hormone

Decreased thyroid-stimulating hormone (TSH) Negative feedback occurs when secretion of one hormone causes a reduction in the secretion of the hormone that stimulates production of the first hormone. In this case, TSH, which is manufactured by the anterior pituitary gland, would normally stimulate release of thyroid hormones, but with the increase of those hormones by the secreting tumor, enough thyroid hormones flood the system that there should be a reduction in TSH levels.

The nurse is evaluating the hearing of a newborn with the understanding that an infant may be born with hearing deficits related to which family history? Select all that apply.

Developmental malformation of the inner ear Maternal rubella Genetic causes Explanation: Hearing loss in infants may be caused by developmental malformation of the inner ear, genetic causes, and intrauterine exposure to maternal rubella. Maternal immunization for influenza is not associated with hearing deficits in the infant.

The nurse is caring for a client who has been diagnosed with presbycusis. The nurse understands that with this disorder, the client experiences which of the following?

Difficulty hearing high-pitched frequencies

The nurse is caring for a client who has been diagnosed with presbycusis. The nurse understands that with this disorder, the client experiences which of the following?

Difficulty hearing high-pitched frequencies.

The nurse understands that a distinguishing pathology of benign paroxysmal positional vertigo is which of the following? a) Proprioception deficit b) Orthostatic hypotension c) Visual deficit d) Disorder of otoliths

Disorder of otoliths Benign paroxysmal positional vertigo is caused by the otoliths or free-floating debris causing a portion of the vestibular system to become more sensitive, such that any movement of the head in the plane parallel to the posterior duct may cause vertigo and nystagmus.

The nurse knows that patients with which of the following diabetic complications have the greatest risk for the development of foot ulcers? a) Microanghiopathy b) Distal symmetric neuropathy c) Previous incidents of diabetic ketoacidosis d) Autonomic neuropathy

Distal symmetric neuropathy Distal symmetric neuropathy is a major risk factor for foot ulcers due to the fact that people with sensory neuropathies have impaired pain sensation and can be unaware of foot injuries and infections. Autonomic neuropathy, microangiopahty, or diabetic ketoacidosis, while not affecting risk for foot injuries, suggest that the patient's diabetes is inadequately controlled.

A pancreatitis patient is admitted with weight loss, nausea, and vomiting. To maintain nutrition, the physician orders parental nutrition to be started. Knowing that a major side effect of parenteral nutrition is a hyperosmolar hyperglycemic state, the nurse should assess the patient for which clinical manifestations (listed below)? a) Irritability, bradycardia, wheezing noted on inspiration. b) Dry lips, excess urine output, and seizures. c) Fever, chills, elevated BP of 170/101. d) Facial ticks, shuffling gait, stiff joints.

Dry lips, excess urine output, and seizures. Hyperosmolar hyperglycemic state is characterized by high blood glucose (> 600 mg/dL), dehydration (dry lips), depression of sensorium, hemiparesis, seizures, and coma. Also weakness, polyuria, excessive thirst. HHS may occur in various conditions, including type 2 diabetes, acute pancreatitis, severe infection, MI, and treatment with oral or parenteral nutrition solutions

17. What is the term that refers to a twitching of the eyelid? a. Blepharotomy b. Blepharoptosis c. Blepharoplasty d. Blepharitis e. Blepharospasm

E. Blepharospasm

6. What is the term for hearing loss from an impairment in the transmission of sound due to an obstruction? a. Anacusis b. Meniere disease c. Strabismus d. Suppurative e. Conductive

E. Conductive

31. Which procedures is used to insert PE tubes in the eardrum? a. Pneumatic otoscopy b. Stapedotomy c. Salpingostomy d. Otoplasty e. Myringotomy

E. Myringotomy

19. Plastic surgery of the ear is called: a. Blepharoplasty b. Keratotomy c. Otoscopy d. Salpingoscopy e. Otoplasty

E. Otoplasty

25. What does the term otalgia refer to? a. Softening of the ear b. Rupture of the eardrum c. Suture of the ear d. Difficulty hearing e. Pain in the ear

E. Pain in the ear

An adult client with a possible growth hormone (GH)-secreting tumor is undergoing testing. If a glucose load is given, which response by the body would confirm the client has a GH-secreting tumor?

Elevated GH level after a glucose load

A nurse examines the laboratory values of a client in heart failure. Which value indicates a compensatory hormone mechanism?

Elevated atrial natriuretic hormone

A nurse examines the laboratory values of a patient in heart failure. Which of the following values indicates a compensatory hormone mechanism? Elevated atrial natriuretic hormone Decreased hematocrit Elevated serum pH Decreased red blood cell count

Elevated atrial natriuretic hormone

A nurse examines the laboratory values of a patient in heart failure. Which of the following values indicates a compensatory hormone mechanism?

Elevated atrial natriuretic hormone Explanation: In heart failure, the patient experiences fluid backlog in the heart as venous blood continues to return, but cardiac output is reduced. This stretches the atria, which secrete atrial natriuretic hormone (or peptide) to stimulate vasodilation and increased renal excretion of sodium and water. This reduces the volume and the strain in the heart. Page 754

A nurse examines the laboratory values of a patient in heart failure. Which of the following values indicates a compensatory hormone mechanism?

Elevated atrial natriuretic hormone Explanation: In heart failure, the patient experiences fluid backlog in the heart as venous blood continues to return, but cardiac output is reduced. This stretches the atria, which secrete atrial natriuretic hormone (or peptide) to stimulate vasodilation and increased renal excretion of sodium and water. This reduces the volume and the strain in the heart.

Which of the following manifestations would the nurse observe when assessing a patient diagnosed with a pheochromocytoma of the adrenal medulla?

Elevated catecholamine levels and tachycardia Explanation: The adrenal medulla secretes the neurotransmitters epinephrine (adrenalin) and norepinephrine (a precursor to epinephrine). A secreting tumor will cause the patient to experience manifestations of sympathetic stimulation such as episodic anxiety, tachycardia, and hypertension. A tumor of the adrenal medulla will not impact thyroid or aldosterone secretion. Page 756

Which of the following manifestations would the nurse observe when assessing a patient diagnosed with a pheochromocytoma of the adrenal medulla?

Elevated catecholamine levels and tachycardia Explanation: The adrenal medulla secretes the neurotransmitters epinephrine (adrenalin) and norepinephrine (a precursor to epinephrine). A secreting tumor will cause the patient to experience manifestations of sympathetic stimulation such as episodic anxiety, tachycardia, and hypertension. A tumor of the adrenal medulla will not impact thyroid or aldosterone secretion.

A client with a suspected diagnosis of primary hypothyroidism would most likely demonstrate which of the following serum laboratory values? a) Decreased thyroid-stimulating hormone (TSH) related to poor thyroid function b) Decreased thyroid-stimulating hormone (TSH) and thyroxine (T4), and low T3 c) Elevated thyroid-stimulating hormone (TSH) and decreased thyroxine (T4) d) Elevated thyroid-stimulating hormone (TSH), thyroxine (T4), and T3

Elevated thyroid-stimulating hormone (TSH) and decreased thyroxine (T4) A low serum T4 and elevated TSH levels are characteristic of primary hypothyroidism. Elevated TSH, T4, and T3 may indicate hyperthyroidism. Decreased levels may be due to suppression by medication.

A health care provider suspects that a thyroid nodule may be malignant. The nurse knows to prepare information for the patient based on the usual test that will be ordered to establish a diagnosis. What is that test?

Fine-needle biopsy of the thyroid gland

A 38-year-old woman takes clomiphene, an infertility drug that works by competing with, and thereby blocking, cellular receptors for estrogen. Which of the following statements is most likely to be true of this client?

Estrogen will continue to pass freely through the cellular membranes. Explanation: Because estrogen is a steroid hormone, its receptors in target cells are located inside the cell membrane, and their blockage does not affect the movement of the hormone into and out of the cell. Receptors are specific for each hormone, so no hormones other than estrogen will be blocked. Up-regulation occurs when hormone levels are decreased, and in this case the estrogen level will increase. Second messengers, such as cAMP, are only activated by peptide hormones and catecholamines. Page 758

A 15-year-old who has just been diagnosed with type 1 diabetes says she read on the Internet that diabetes is the leading cause of acquired blindness among Americans. She asks you if she will lose her sight. In addition to explaining that new treatment technologies are being worked on every day, which of the following would be the most appropriate response? a) Tell her to expect that she will begin to lose her eyesight by the time she is 25. b) Tell her there is about a 50 percent chance that she will suffer some diabetes-related sight loss by the time she is 50. c) Explain that almost all people with type 1 diabetes do experience some degree of vision loss. d) Reassure her that only type 2 diabetes is a risk factor for blindness.

Explain that almost all people with type 1 diabetes do experience some degree of vision loss. Nearly all people with type 1 diabetes and more than 60 percent of people with type 2 diabetes have some degree of retinopathy. Pregnancy, puberty, and cataract surgery can accelerate these changes. (less)

A 15-year-old child with type 1 diabetes asks nurse about the potential to "lose sight." Which response would be the most appropriate?

Explain that many people with diabetes experience some complications like retinopathy, but these are best prevented with tight control of glucose levels.

A group of student nurses is reviewing the anatomy of the eye for an upcoming exam. One of the student nurses asks where the tarsal plate is located. What is the response?

Eyelid

Which of the following nursing interventions is the highest priority intervention for a nurse caring for a 26-year-old client diagnosed with benign paroxysmal positional vertigo implement?

Fall precautions

Which of the following nursing interventions is the highest priority intervention for a nurse caring for a 26-year-old client diagnosed with benign paroxysmal positional vertigo implement? a) Adminster ordered antibiotic b) Weber test c) Skin assessment d) Fall precautions

Fall precautions The patient with benign paroxysmal positional vertigo experiences brief periods of vertigo when getting in and out of bed, bending over and straightening up, and extending head to look up. This places the patient at increased risk for falls and necessitates implementation of fall precautions. -The Weber test and administration of an antibiotic are not indicated when caring for a patient with benign paroxysmal positional vertigo. -Skin assessment, while always a part of the nursing assessment, has a lower priority than implementation of safety measures for this client.

Which of the following is the role of glucagon in initiating the breakdown of glycogen? a) Receptor antagonist b) Second messenger c) First messenger d) Intracellular catalyst

First messenger Glucagon acts on the surface receptors of liver cells as first messenger. Glucagon starts the process of glycogen breakdown by signaling the intracellular signal system, called the second messenger.

Which of the following is the role of glucagon in initiating the breakdown of glycogen?

First messenger Explanation: Glucagon acts on the surface receptors of liver cells as first messenger. Glucagon starts the process of glycogen breakdown by signaling the intracellular signal system, called the second messenger.

While discussing the regulation of hormone levels, the instructor gives an example of hormones regulated by feedback mechanisms. Which example of this regulation is best?

Following a meal that was high in carbohydrates, a person's blood glucose elevates, which stimulates the release of insulin from the pancreas.

While discussing the regulation of hormone levels, the instructor gives an example of hormones regulated by feedback mechanisms. Which example of this regulation is best?

Following a meal that was high in carbohydrates, a person's blood glucose elevates, which stimulates the release of insulin from the pancreas. Explanation: The levels of hormones such as insulin and antidiuretic hormone (ADH) are regulated by feedback mechanisms that monitor substances such as glucose (insulin) and water (ADH) in the body. None of the other examples are accurate examples of this feedback mechanism. Page 759

While discussing the regulation of hormone levels, the instructor gives an example of hormones regulated by feedback mechanisms. Which example of this regulation is best? a) When a female is thinking about getting pregnant, the body knows to release female sex hormones in greater proportion than usual. b) In children, the body knows to release growth hormones while they are sleeping instead of when they are awake and running around. c) Following a meal that was high in carbohydrates, a person's blood glucose elevates, which stimulates the release of insulin from the pancreas. d) When a person's body pH is decreasing, the stomach lining becomes more acidic to offset this.

Following a meal that was high in carbohydrates, a person's blood glucose elevates, which stimulates the release of insulin from the pancreas. The levels of hormones such as insulin and antidiuretic hormone (ADH) are regulated by feedback mechanisms that monitor substances such as glucose (insulin) and water (ADH) in the body. None of the other examples are accurate examples of this feedback mechanism

While discussing the regulation of hormone levels, the instructor gives an example of hormones regulated by feedback mechanisms. Which example of this regulation is best?

Following a meal that was high in carbohydrates, a person's blood glucose elevates, which stimulates the release of insulin from the pancreas. Explanation: The levels of hormones such as insulin and antidiuretic hormone (ADH) are regulated by feedback mechanisms that monitor substances such as glucose (insulin) and water (ADH) in the body. None of the other examples are accurate examples of this feedback mechanism.

A nurse caring for a client in the hospital with new onset of Ménière disease reviews the treatment options with the client. Which medication will the nurse teach the client about to decrease the volume of endolymph?

Furosemide Explanation: The use of diuretics, such as furosemide, stimulates the kidneys to excrete more fluid into the urine, thereby decreasing the fluid volume in the body, including the endolymph. Decreasing the endolymphatic compartment and in particular the endolymphatic sac within the inner ear helps to alleviate hearing loss, vertigo, and tinnitus.

A hypernatremic client is researching his condition and learns that aldosterone binds to an intracellular receptor. What do intracellular receptors generally stimulate in a cell?

Gene expression Explanation: Aldosterone is a lipid-soluble hormone that activates gene transcription factors in the cytosol. Most lipid-soluble hormones work in this way. Water-soluble hormones activate ion channel receptors, cAMP, and G-proteins. Page 759

Type 1A diabetes is now considered an autoimmune disorder. What factors are considered necessary for type 1A diabetes to occur?

Genetic predisposition, environmental triggering event, and a T-lymphocyte-mediated hypersensitivity reaction against some beta-cell antigen

Type 1A diabetes is now considered an autoimmune disorder. What factors are considered necessary for type 1A diabetes to occur? a) Diabetogenic gene from both parents, environmental triggering event, and a B lymphocyte reaction to alpha cell antigens b) Genetic predisposition, environmental triggering event, and a T-lymphocyte-mediated hypersensitivity reaction against some beta-cell antigen c) Diabetogenic gene from both parents, physiologic triggering event, and an allergic reaction to pancreatic delta cells d) Genetic predisposition, physiologic triggering event, allergic reaction to pancreatic alpha cells

Genetic predisposition, environmental triggering event, and a T-lymphocyte-mediated hypersensitivity reaction against some beta-cell antigen Type 1A diabetes is thought to be an autoimmune disorder resulting from a genetic predisposition (i.e., diabetogenic genes); an environmental triggering event, such as an infection; and a T-lymphocyte-mediated hypersensitivity reaction against some beta-cell antigen. The other answers are incorrect

The nurse is caring for a client who received regular insulin at 7 am. Four hours later the nurse finds the client diaphoretic, cool, and clammy. Which of these interventions is the priority? a) Bathe the client with tepid water. b) Place the client in the supine position. c) Repeat the dose of insulin. d) Give the client a concentrated carbohydrate.

Give the client a concentrated carbohydrate. The client is displaying symptoms of hypoglycemia, which include headache, difficulty in problem solving, altered behavior, coma, and seizures. Hunger may occur. Activation of the sympathetic nervous system may cause anxiety, tachycardia, sweating, and cool and clammy skin.

A patient with diabetes asks the nurse for advice in controlling between-meal blood glucose levels. Which of the following might the nurse suggest as a long-acting insulin to provide a consistent basal level? a) Glulisine (Apidra) b) Regular c) Glargine (Lantus) d) Aspart (Novolog)

Glargine (Lantus) Glargine is a long-acting peakless insulin that provides consistent basal levels over a 20- to 24-hour period. Regular insulin has a duration of 5 to 7 hours . Aspart and glulisine have the most rapid onset and shortest duration of action, making them optimal for postprandial glucose control

The nursing student who is studying pathophysiology correctly identifies the condition that characteristically has an increase in the aqueous humor that fills the anterior and posterior chambers of the eye. What is this disease called?

Glaucoma Explanation: Glaucoma is a chronic degenerative optic neuropathy characterized by optic disk cupping and visual field loss. It is usually associated with an elevation in intraocular pressure. Amblyopia is lazy eye. Retinopathy is a disorder of the retinal vessels that interrupts blood flow to the visual receptors leading to visual impairment. A cataracts is a lens opacity that interferes with the transmission light to the retina.

The nursing student who is studying pathophysiology correctly identifies the condition that characteristically has an increase in the aqueous humor which fills the anterior and posterior chambers of the eye as which disease?

Glaucoma.

The nurse and nursing student are performing medication reconciliation. Which medications taken by the client with diabetes does the nurse teach the student are implicated in causing hyperglycemia? Select all that apply.

Glucocorticoids Loop diuretics Oral contraceptives

Which statement best describes an aspect of the normal process of glucose metabolism?

Glucose that exceeds metabolic needs is converted and stored by the liver.

A woman in her 28th week of pregnancy develops gestational diabetes mellitus and has persistent elevated blood glucose when using the diet plan. What medications are appropriate? Select all that apply.

Glyburide Insulin

A client is diagnosed with type 2 diabetes mellitus and begins to follow a nutritional plan at home. What result at the follow-up visit indicates a successful outcome?

Glycosylated hemoglobin 5.2% (0.52)

A client tells the health care provider that he has been very compliant over the last 2 months in the management of his diabetes. The best diagnostic indicator that would support the client's response would be:

Glycosylated hemoglobin, hemoglobin A1C (HbA1C)

A client tells the health care provider that he has been very compliant over the last 2 months in the management of his diabetes .The best diagnostic indicator that would support the client's response would be: a) Fasting blood glucose level b) Capillary blood glucose sample c) Urine test d) Glycosylated hemoglobin, hemoglobin A1C (HbA1C)

Glycosylated hemoglobin, hemoglobin A1C (HbA1C) Glycosylated hemoglobin, hemoglobin A1C (HbA1C), and A1C are terms used to describe hemoglobin into which glucose has been incorporated. Glycosylation is essentially irreversible, and the level of A1C present in the blood provides an index of blood glucose levels over the previous 6 to 12 weeks. In uncontrolled diabetes or diabetes with hyperglycemia, there is an increase in the level of A1C. The other options would not reflect the 2-month period

A 25-year-old female client exhibits exophthalmos of both eyes. The health care provider recognizes this as a manifestation of: a) Myxedema b) Acquired hypothyroidism c) Graves disease d) Hashimoto thyroiditis

Graves disease Graves disease is a state of hyperthyroidism in which opthalmopathies, such as exophthalmos, typically occur. The other conditions are states of hypothyroidism and are not associated with this abnormality.

A middle-aged female client complains of anxiety, insomnia, weight loss, the inability to concentrate, and eyes feeling "gritty." Thyroid function tests reveal the following: thyroid-stimulating hormone (TSH) 0.02 U/ml, thyroxine 20 g/dl, and triiodothyronine 253 ng/dl. A 6-hour radioactive iodine uptake test showed a diffuse uptake of 85%. Based on these assessment findings, the nurse should suspect:

Graves' disease.

When educating a patient about glargine (Lantus), the nurse should explain that this medication: a) Has a rapid onset and peaks in about 5 minutes after injection so they will need to eat food immediately after injection. b) Since this medication will have a peak effect within 30 minutes, it can be taken after a meal. c) Has a prolonged absorption rate and provides a relatively constant concentration for 12-24 hours. d) Is a combination with short acting and intermediate acting insulin so it is safe to take anytime throughout the day.

Has a prolonged absorption rate and provides a relatively constant concentration for 12-24 hours. Lantus is long-acting insulin that has a slow, prolonged absorption rate and provides a relatively constant concentration over 12-24 hours. A=rapid acting insulin; B= Short acting insulin; C=Intermediate-acting insulin.

When educating a patient about glargine(Lantus), the nurse should explain that this medication: a) Has a rapid onset and peaks in about 5 minutes after injection so they will need to eat food immediately after injection. b) Since this medication will have a peak effect within 30 minutes, it can be taken after a meal. c) Has a prolonged absorption rate and provides a relatively constant concentration for 12-24 hours. d) Is a combination with short acting and intermediate acting insulin so it is safe to take anytime throughout the day.

Has a prolonged absorption rate and provides a relatively constant concentration for 12-24 hours. Lantus is long-acting insulin that has a slow, prolonged absorption rate and provides a relatively constant concentration over 12-24 hours. A=rapid acting insulin; B= Short acting insulin; C=Intermediate-acting insulin.

When the assessment of thyroid autoantibodies is performed, what is the suspected diagnosis? a) Congenital hypothyroidism b) Hashimoto thyroiditis c) Thyroid tumor d) Goiter

Hashimoto thyroiditis The assessment of thyroid autoantibodies (e.g., antithyroid peroxidase antibodies in Hashimoto thyroiditis) is important in the diagnostic workup and consequent follow-up of thyroid clients.

The nurse is assisting with a client who is undergoing caloric testing. The nurse expects which of the following actions will be needed to complete the test?

Have ice water available.

The nurse is assisting with a client who is undergoing caloric testing. The nurse expects which action will be needed to complete the test?

Have ice water available. Explanation: Caloric testing involves elevating the head 30 degrees and irrigating each external auditory canal separately with 30 to 50 mL of ice water. The client does not stand for the test. No electrodes are use for this test.

A client has been admitted with diabetic ketoacidosis. The emergency department starts an IV to improve circulatory volume. If there is a sudden change in extracellular fluid osmolality which results in a too rapid blood glucose lowering, the nurse will likely observe which clinical manifestations?

Headaches, dizziness, change in level of consciousness.

The parents of a 1-month-old child tell the nurse at the pediatrician's office that their child doesn't seem right; the child doesn't always look toward them when they speak. What assessment does the nurse expect the physician to conduct?

Hearing screening

The parents of a 1-month-old child tell the nurse at the pediatrician's office that their child doesn't seem right; the child doesn't always look toward them when they speak. What assessment does the nurse expect the physician to conduct?

Hearing screening.

The nurse is performing an assessment for a client who has hyperthyroidism that is untreated. When obtaining vital signs, what is the expected finding? a) Blood pressure 180/110 mm Hg b) Heart rate 110 and bounding c) Respiratory rate 14 d) Temperature 96oF

Heart rate 110 and bounding Cardiovascular and respiratory functions are strongly affected by thyroid function. With an increase in metabolism, there is a rise in oxygen consumption and production of metabolic end products, with an accompanying increase in vasodilation. Blood volume, cardiac output, and ventilation are all increased. Heart rate and cardiac contractility are enhanced as a means of maintaining the needed cardiac output. Blood pressure is likely to change little because the increase in vasodilation tends to offset the increase in cardiac output.

Which laboratory values for a newly admitted client indicate a diagnosis of diabetes mellitus? Select all that apply.

Hemoglobin A1C 9.1% (.09) 2 hour oral GTT 245 mg/dL (13.6 mmol/L)

x Which statement best explains the function of hormone receptors? Hormone receptors recognize a specific hormone and translate the hormonal signal into a cellular response. Hormone receptors produce antibodies to block certain hormones from entering the cell. Hormone receptors are located within the red blood cells and carry hormones to the target cells. Hormone receptors respond to decreasing hormone levels by producing a decrease in receptor numbers.

Hormone receptors recognize a specific hormone and translate the hormonal signal into a cellular response.

Select the statement that best explains the function of hormone receptors.

Hormone receptors recognize a specific hormone and translate the signal into a cellular response.

Select the statement that best explains the function of hormone receptors.

Hormone receptors recognize a specific hormone and translate the signal into a cellular response. Explanation: Hormone receptors are complex molecular structures that are located either on the surface of or inside target cells. The function of these receptors is to recognize a specific hormone and translate the hormonal signal into a cellular response. The other choices are not true. Page 760

Select the statement that best explains the function of hormone receptors. a) Hormone receptors respond to decreasing hormone levels by producing a decrease in receptor numbers. b) Hormone receptors recognize a specific hormone and translate the signal into a cellular response. c) Hormone receptors are located within the red blood cells and carry hormones to the target cells. d) Hormone receptors produce antibodies to block certain hormones from entering the cell.

Hormone receptors recognize a specific hormone and translate the signal into a cellular response. Hormone receptors are complex molecular structures that are located either on the surface of or inside target cells. The function of these receptors is to recognize a specific hormone and translate the hormonal signal into a cellular response. The other choices are not true.

Select the statement that best explains the function of hormone receptors.

Hormone receptors recognize a specific hormone and translate the signal into a cellular response. Explanation: Hormone receptors are complex molecular structures that are located either on the surface of or inside target cells. The function of these receptors is to recognize a specific hormone and translate the hormonal signal into a cellular response. The other choices are not true.

When hypofunction of an endocrine organ is suspected, which type of diagnostic test can be administered to measure and assess target gland response?

Hormone stimulation

Select the most appropriate statement that describes the function of hormones.

Hormones function as modulators of cellular and systematic responses.

The nurse is teaching a client diagnosed with Addison disease about the importance of lifetime oral replacement therapy. Select the pharmacologic agent that would be prescribed. a) Hydrocortisone b) Potassium supplements c) Insulin d) Ketoconazole

Hydrocortisone Hydrocortisone is usually the drug of choice in treating Addison disease. In mild cases, hydrocortisone alone may be adequate. Ketoconazole causes excessive breakdown of glucocorticoids and can also result in adrenal insufficiency. Clients with Addison disease usually have elevated potassium levels, and insulin is not the treatment for Addison disease.

Which condition should a nurse expect to find in a client diagnosed with hyperparathyroidism?

Hypercalcemia

During periods of fasting and starvation, the glucocorticoid and other corticosteroid hormones are critical for survival because of their stimulation of gluconeogenesis by the liver. When the glucocorticoid hormones remain elevated for extended periods of time, what can occur?

Hyperglycemia

During periods of fasting and starvation, the glucocorticoid and other corticosteroid hormones are critical for survival because of their stimulation of gluconeogenesis by the liver. When the glucocorticoid hormones remain elevated for extended periods of time, what can occur? a) Hyperglycemia b) Hepatomegaly c) Portal hypertension d) Adrenal hyperplasia

Hyperglycemia In predisposed persons, the prolonged elevation of glucocorticoid hormones can lead to hyperglycemia and the development of diabetes mellitus and starvation. They stimulate gluconeogenesis by the liver, sometimes producing a 6- to 10-fold increase in hepatic glucose production. A prolonged increase in glucocorticoid hormones does not cause hepatomegaly, portal hypertension, or adrenal hyperplasia.

A client with hyperthyroidism took aspirin for a headache. Which complications could develop? Select all that apply.

Hyperthermia Tachycardia Agitation

A 30-year-old male who manages his type 1 diabetes with glyburide presents at the emergency room reporting headache, confusion, and tachycardia. He has come from a party at which he drank two beers to celebrate running his first half-marathon. Which of the following is likely to be the cause of his complaints?

Hypoglycemia

A man is brought into the emergency department by paramedics who state that the client passed out on the street. The man smells of alcohol, and when roused says he has not eaten since yesterday. He is wearing a medic alert bracelet that says he is a diabetic. What would the nurse suspect as a diagnosis? a) Hypernatremia b) Hyperglycemia c) Hyponatremia d) Hypoglycemia

Hypoglycemia Alcohol decreases liver gluconeogenesis, and people with diabetes need to be cautioned about its potential for causing hypoglycemia, especially if alcohol is consumed in large amounts or on an empty stomach

The hypophysis is a unit formed by the pituitary and the hypothalamus. These two glands are connected by the blood flow in what system?

Hypophyseal portal system

The hypophysis is a unit formed by the pituitary and the hypothalamus. These two glands are connected by the blood flow in what system?

Hypophyseal portal system Explanation: The hypothalamus and pituitary (i.e., hypophysis) form a unit that exerts control over many functions of several endocrine glands as well as a wide range of other physiologic functions. These two structures are connected by blood flow in the hypophyseal portal system, which begins in the hypothalamus and drains into the anterior pituitary gland, and by the nerve axons that connect the supraoptic and paraventricular nuclei of the hypothalamus with the posterior pituitary gland. The other answers are not correct. Page 762

The hypophysis is a unit formed by the pituitary and the hypothalamus. These two glands are connected by the blood flow in what system?

Hypophyseal portal system Explanation: The hypothalamus and pituitary (i.e., hypophysis) form a unit that exerts control over many functions of several endocrine glands as well as a wide range of other physiologic functions. These two structures are connected by blood flow in the hypophyseal portal system, which begins in the hypothalamus and drains into the anterior pituitary gland, and by the nerve axons that connect the supraoptic and paraventricular nuclei of the hypothalamus with the posterior pituitary gland. The other answers are not correct.

A health care provider is assessing a client for a potential endocrine disorder. Assessment findings identify abnormalities with emotion, pain, and body temperature. Which mechanism of endocrine control will require further laboratory/diagnostic assessment?

Hypothalamus

A healthcare provider is assessing a client for a potential endocrine disorder. Assessment findings identify abnormalities with emotion, pain, and body temperature. Which mechanism of endocrine control will require further lab/diagnostic assessment? Hypothalamus Anterior pituitary Cerebellum Cerebral cortex

Hypothalamus

Which gland acts as a signal relaying bridge between multiple body systems and the pituitary gland?

Hypothalamus Explanation: The activity of the hypothalamus is regulated by both hormonally mediated signals (e.g., negative feedback signals) and by neuronal input from a number of sources. Neuronal signals are mediated by neurotransmitters such as acetylcholine, dopamine, norepinephrine, serotonin, ?-aminobutyric acid (GABA), and opioids. Cytokines that are involved in immune and inflammatory responses, such as the interleukins, also are involved in the regulation of hypothalamic function. This is particularly true of the hormones involved in the hypothalamic-pituitary-adrenal axis. Thus, the hypothalamus can be viewed as a bridge by which signals from multiple systems are relayed to the pituitary gland. This cannot be said of the other options. Page 759

The physician is assessing a client with a preliminary diagnosis of endocrine disorder. Further assessment findings identify abnormalities with emotion, pain, body temperature, and neural input. The physician determines the need to further assess the:

Hypothalamus Explanation: The hypothalamus is the coordinating center of the brain for endocrine, behavioral, and autonomic nervous system function. It is at the level of the hypothalamus that emotion, pain, body temperature, and other neural input are communicated to the endocrine system. The anterior pituitary regulates several physiological processes, including stress, growth, reproduction, and lactation. The cerebellum is involved in motor control, and the cerebral cortex is associated with sensory, motor, and association. Page 754

The physician is assessing a client with a preliminary diagnosis of endocrine disorder. Further assessment findings identify abnormalities with emotion, pain, body temperature, and neural input. The physician determines the need to further assess the: a) Cerebellum b) Cerebral cortex c) Hypothalamus d) Anterior pituitary

Hypothalamus The hypothalamus is the coordinating center of the brain for endocrine, behavioral, and autonomic nervous system function. It is at the level of the hypothalamus that emotion, pain, body temperature, and other neural input are communicated to the endocrine system. The anterior pituitary regulates several physiological processes, including stress, growth, reproduction, and lactation. The cerebellum is involved in motor control, and the cerebral cortex is associated with sensory, motor, and association

Which gland acts as a signal relaying bridge between multiple body systems and the pituitary gland?

Hypothalamus Explanation: The activity of the hypothalamus is regulated by both hormonally mediated signals (e.g., negative feedback signals) and by neuronal input from a number of sources. Neuronal signals are mediated by neurotransmitters such as acetylcholine, dopamine, norepinephrine, serotonin, ?-aminobutyric acid (GABA), and opioids. Cytokines that are involved in immune and inflammatory responses, such as the interleukins, also are involved in the regulation of hypothalamic function. This is particularly true of the hormones involved in the hypothalamic-pituitary-adrenal axis. Thus, the hypothalamus can be viewed as a bridge by which signals from multiple systems are relayed to the pituitary gland. This cannot be said of the other options.

The physician is assessing a client with a preliminary diagnosis of endocrine disorder. Further assessment findings identify abnormalities with emotion, pain, body temperature, and neural input. The physician determines the need to further assess the:

Hypothalamus Explanation: The hypothalamus is the coordinating center of the brain for endocrine, behavioral, and autonomic nervous system function. It is at the level of the hypothalamus that emotion, pain, body temperature, and other neural input are communicated to the endocrine system. The anterior pituitary regulates several physiological processes, including stress, growth, reproduction, and lactation. The cerebellum is involved in motor control, and the cerebral cortex is associated with sensory, motor, and association.

57. double a. Acous/o k. kerat/o b. Blephar/o l. myring/o c. Chromat/o m. ophthalm/o d. Conjunctiv/o n. ot/o e. Choroid/o o. salping/o f. Core/o p. -metry g. Corne/o q. -opsia h. Dacry/o r. -ptosis i. dipl/o s. -rrhea j. irid/o t. -tropia

I. dipl/o

A client with pheochromocytoma has been admitted for an adrenalectomy to be performed the following day. To prevent complications, the nurse should anticipate preoperative administration of which of the following?

IV corticosteroids

The ophthalmologist is performing a client's annual eye exam and notes an increase in intraocular pressure. The condition most likely is the result of:

Imbalance between aqueous production and outflow

The ophthalmologist is performing a client's annual eye exam and notes an increase in intraocular pressure. The condition most likely is the result of:

Imbalance between aqueous production and outflow.

The nurse has received a prescription to administer a ceruminolytic agent twice daily to an elderly client. The nurse understands that the rationale for use of this agent is to remove which of the following?

Impacted ear wax

The nurse is caring for an elderly client with a suspected diagnosis of presbycusis. Select the most likely manifestation that the nurse would note. a) Inability to understand words during a conversation b) Feeling of continuous pressure in both ears c) Feeling a continuous drainage coming from both ears d) Buzzing or ringing in the ears

Inability to understand words during a conversation Presbycusis is a common report among the elderly. It is manifested by the inability to understand or hear what is being said in a conversation that progresses to not being able to hear. Pressure may result from increased fluid, and buzzing or ringing may be a complication of medication.

When reviewing laboratory results for a patient with a possible diagnosis of hypoparathyroidism, the nurse knows that this condition is characterized by which of the following?

Inadequate secretion of parathormone

The nurse is caring for a pediatric client who has a diagnosis of acute otitis media (AOM). Which of the following is an appropriate option for pain management associated with this condition?

Incision of the tympanic membrane.

When the nurse is performing a health history for a client who is being admitted for hyperthyroidism, what symptoms does the client report that the nurse would find associated with this disorder? a) Increase in appetite b) Constipation c) Weight gain d) Fatigue

Increase in appetite Thyroid hormone enhances gastrointestinal function, causing an increase in motility and production of GI secretions that often results in diarrhea. An increase in appetite and food intake accompanies the higher metabolic rate that occurs with increased thyroid hormone levels. At the same time, weight loss occurs because of the increased use of calories.

An adult patient is scheduled for testing of a suspected growth hormone (GH)- secreting tumor. Which of the following results from the glucose suppression test would confirm the condition? a) Hyperinsulinemia b) Hypoglycemia c) Increased GH secretion d) Decreased GH levels

Increased GH secretion A suppression test is intended to determine if an organ that is oversecreting will respond to feedback to suppress that hormone. The glucose suppression test is used for an adult with acromegaly from excessive GH production. Normally, glucose would cause a decrease in GH secretion, but there is increased production of GH in response to the glucose in the test when the patient has acromegaly.

An adult patient is scheduled for testing of a suspected growth hormone (GH)- secreting tumor. Which of the following results from the glucose suppression test would confirm the condition?

Increased GH secretion Explanation: A suppression test is intended to determine if an organ that is oversecreting will respond to feedback to suppress that hormone. The glucose suppression test is used for an adult with acromegaly from excessive GH production. Normally, glucose would cause a decrease in GH secretion, but there is increased production of GH in response to the glucose in the test when the patient has acromegaly. Page 764

An adult patient is scheduled for testing of a suspected growth hormone (GH)- secreting tumor. Which of the following results from the glucose suppression test would confirm the condition?

Increased GH secretion Explanation: A suppression test is intended to determine if an organ that is oversecreting will respond to feedback to suppress that hormone. The glucose suppression test is used for an adult with acromegaly from excessive GH production. Normally, glucose would cause a decrease in GH secretion, but there is increased production of GH in response to the glucose in the test when the patient has acromegaly.

The nurse is discussing positive feedback mechanisms. Which example best explains this mechanism?

Increased estradiol production causes increased follicle-stimulating hormone (FSH) production.

An instructor is teaching the class about positive feedback mechanism. The best example would be:

Increased estradiol production causes increased gonadotropin (FSH) production. Explanation: The positive feedback mechanism regulates hormones as follows: a rising level of a hormone causes another gland to release a hormone that is stimulating to the first. The other choices represent negative feedback mechanisms of control. Page 762

An instructor is teaching the class about positive feedback mechanism. The best example would be: a) Increased estradiol production causes increased gonadotropin (FSH) production. b) Decreased cortisone levels increase blood levels of adrenocorticotropic hormone (ACTH). c) Decreased thyroid hormone (TH) increases thyroid-stimulating hormone (TSH) levels. d) Increased glucose levels in the blood decreases growth hormone (GH) levels.

Increased estradiol production causes increased gonadotropin (FSH) production. The positive feedback mechanism regulates hormones as follows: a rising level of a hormone causes another gland to release a hormone that is stimulating to the first. The other choices represent negative feedback mechanisms of control.

A client comes to the clinic with fatigue and muscle weakness. The client also states she has been having diarrhea. The nurse observes the skin of the client has a bronze tone and when asked, the client says she has not had any sun exposure. The mucous membranes of the gums are bluish-black. When reviewing laboratory results from this client, what does the nurse anticipate seeing? a) Increased levels of ACTH b) Increase in sedimentation rate c) Elevated WBC count d) Positive C-reactive protein

Increased levels of ACTH Hyperpigmentation results from elevated levels of ACTH. The skin looks bronzed or suntanned in exposed and unexposed areas, and the normal creases and pressure points tend to become especially dark. The gums and oral mucous membranes may become bluish-black. The amino acid sequence of ACTH is strikingly similar to that of melanocyte stimulating hormone; hyperpigmentation occurs in greater than 90 percent of persons with Addison's disease and is helpful in distinguishing the primary and secondary forms of adrenal insufficiency.

An older client is complaining of having difficulty seeing. The nurse suspects that the client has cataracts. What are the common manifestations of a cataract? Select all that apply.

Increasingly blurred vision Visual distortion Explanation: Age-related cataracts, which are the most common type, are characterized by increasingly blurred vision and visual distortion. Visual acuity for far and near objects decreases. Dilation of the pupil in dim light improves vision. With nuclear cataracts (those involving the lens nucleus), the refractive power of the anterior segment often increases to produce an acquired myopia.

The nurse is conducting a community education class on amblyopia. The nurse determines that the participants are understanding the concepts when they identify which of the following as the time when amblyopia is generally diagnosed?

Infancy

The nurse is caring for a client who has been diagnosed with Meniere disease. The nurse expects which medication will be prescribed to manage the symptoms?

Promethazine Explanation: Pharmacologic management includes suppressant drugs such as promethazine. The other medications are not used to treat Meniere disease.

The nurse is conducting a community education class on amblyopia. The nurse determines that the participants understand the concepts when they identify which time period as the time when amblyopia is generally diagnosed?

Infancy Explanation: Amblyopia (i.e., lazy eye) is a decrease in visual acuity resulting from abnormal visual development in infancy and early childhood. There is no known connection between the condition and either injury or diabetes.

The nurse is conducting a community education class on amblyopia. The nurse determines that the participants are understanding the concepts when they identify which of the following as the time when amblyopia is generally diagnosed?

Infancy.

Select the most appropriate intervention for the nurse to teach a client diagnosed with distal symmetric neuropathy related to diabetes. a) Inspect the feet for blisters daily b) Decrease daily walking activity c) Wear comfortable, open-toe shoes d) Rotate insulin injection sites once a week

Inspect the feet for blisters daily A client with neuropathy is at risk for damage to his or her feet, such as blisters or ulcers, as the clients are unable to feel this damage. Clients need to inspect their feet daily, where foot coverings (such as closed-toe shoes) to prevent injuries, and continue the exercise patterns to promote improved circulation.

A 51 year-old woman has been experiencing signs and symptoms of perimenopause and has sought help from her family physician. A deficiency in estrogen levels has been determined to be a contributing factor. Which of the following phenomena could potentially underlie the woman's health problem?

Insufficient estrogen production within the smooth endoplasmic reticulum of the relevant cells.

A client with diabetes asks the nurse for advice in controlling between-meal blood glucose levels. Which of these might the nurse suggest as a long-acting insulin to provide a consistent basal level?

Insulin glargine

The diagnosis of type 1 diabetes would be confirmed by: a) Insulin is produced but unavailable for use in the body. b) Insulin is present in large amounts for use by the body. c) Insulin is not available for use by the body. d) Small amounts of insulin are produced daily.

Insulin is not available for use by the body. Type 1 diabetes is a catabolic disorder characterized by an absolute lack of insulin. In type 2 diabetes, some insulin is produced.

Which of the following statements is correct about hormone transport? Select all that apply.

Insulin is unbound. Glucocorticoids are bound. Prolactin is unbound. Explanation: Steroids and thyroid hormone are transported bound to carrier molecules. Peptides and protein hormones such as insulin, glucagon, parathyroid hormone, prolactin, and ACTH usually circulate unbound.

Which statements are correct about hormone transport? Select all that apply.

Insulin is unbound. Glucocorticoids are bound. Prolactin is unbound.

The nurse knows that metabolic abnormalities that lead to type 2 diabetes include which physiologic problems? Select all that apply.

Insulin resistance Beta cell failure Hepatic glycogenolysis

The nurse is admitting a client documented to have papilledema. The nurse recognizes that papilledema is most commonly caused by which condition?

Intracranial pressure Explanation: The central retinal artery enters the eye through the optic papilla in the center of the optic nerve. An accompanying vein exits the eye along the same path. The entrance and exit of the central retinal artery and vein through the tough scleral tissue at the optic papilla can be compromised by any condition causing persistent increased intracranial pressure. The most common of these conditions are cerebral tumors, subdural hematomas, hydrocephalus, and malignant hypertension. Usually, the thin-walled, low-pressure veins are the first to collapse, with the consequent backup and slowing of arterial blood flow. Under these conditions, capillary permeability increases and leakage of fluid results in edema of the optic papilla, called papilledema.

The nurse documents the presence of nystagmus when assessing a client. This can be interpreted as:

Involuntary eye movements that preserve eye fixation on stable objects in the visual field

The nurse is caring for a client undergoing hyperbaric oxygen therapy to promote wound healing. The nurse understands that the client may experience which condition related to this treatment?

Middle ear barotrauma Explanation: One cause of middle ear barotrauma is hyperbaric oxygen therapy (increased atmospheric pressure and partial oxygen pressure in tissues) used for many people with multiple disorders.

Neurotransmitters like catecholamines (ex. dopamine and epinephrine) have a reaction time of:

Milliseconds. Explanation: The neurotransmitters, which control the opening of ion channels, have a reaction time of milliseconds. Page 753

The nurse documents the presence of nystagmus when assessing a client. This can be interpreted as: a) Fixed eye movements that preserve eye fixation on unstable objects in the visual field b) Voluntary eye movements that preserve eye fixation on stable objects in the visual field c) Involuntary eye movements that preserve eye fixation on stable objects in the visual field d) Unilateral eye movement that preserve eye fixation on unstable objects in the visual field

Involuntary eye movements that preserve eye fixation on stable objects in the visual field Nystagmus refers to the involuntary eye movements that preserve eye fixation on stable objects in the visual field during angular and rotational movements of the head. As the body rotates, the vestibuloocular reflexes cause a slow compensatory drifting of eye movement in the opposite direction, thus stabilizing the binocular fixation point.

Why is it important for a patient with a diagnosis of papilledema to follow up?

It indicates a need for diagnosis of the cause.

During an eye assessment the nurse notes inflammation of the client's cornea. The nurse should document this as which of the following?

Keratitis

During an eye assessment the nurse notes inflammation of the client's cornea. The nurse should document this as which condition?

Keratitis Explanation: Keratitis, or inflammation of the cornea, can be caused by infections, hypersensitivity reactions, ischemia, trauma, defects in tearing, or trauma.

During an eye assessment the nurse notes inflammation of the client's cornea. The nurse should document this as which of the following?

Keratits.

Diabetics are at higher risk than are the majority of the population for injury to organ systems in the body. Which organs are most at risk?

Kidneys and eyes

A client suffering from chronic sinusitis arrives at the urgent care center complaining of a sudden onset of dizziness and difficulty walking straight. What diagnosis does the admitting nurse suspects this client has?

Labyrinthitis Explanation: Chronic infection in the sinuses can cause infection of the inner ear (labyrinthitis), producing symptoms such as vertigo, dizziness, loss of balance, nausea, and vomiting.

A male patient has been diagnosed with a low sperm count. Which of the following endocrine imbalances could contribute to this condition? Select all that apply. Lack of follicle-stimulating hormone (FSH) Excess calcitonin Insufficient androgens Excess thyroid hormone Increased growth hormone

Lack of follicle-stimulating hormone (FSH) Insufficient androgens

The preferred preparation for treating hypothyroidism includes which of the following?

Levothyroxine (Synthroid)

A 5-month-old infant is being treated for acute otitis media (AOM) for the second time in the past 10 weeks. Which action should the nurse recommend to prevent future recurrences?

Limiting the infant's exposure to large group settings Explanation: Ways to reduce the risk of developing AOM include minimal exposure to group settings. MMR and hepatitis vaccines are irrelevant to the etiology of AOM. Ear rinsing is not recommended.

A 5-month-old infant is being treated for acute otitis media (AOM) for the second time in the past 10 weeks. Which action should the nurse recommend to prevent future recurrences?

Limiting the infant's exposure to large group settings.

The nurse has just completed teaching a client newly diagnosed with type 1 diabetes about rapid-acting insulin. The nurse determines that teaching was effective when the client selects: a) Glargine b) Regular c) Lispro d) NPH

Lispro There are four principal types of insulin: short acting, rapid acting, intermediate acting, and long acting. Regular insulin is short-acting insulin whose effects begin within 30 minutes after subcutaneous injection and generally last for 5 to 8 hours. The rapid-acting insulins (lispro, aspart, and glulisine) have a more rapid onset, peak, and duration of action than short-acting regular insulin and are administered immediately before a meal. Intermediate- to long-acting insulins include NPH, glargine, and detemir. These insulins have slower onsets and a longer duration of action.

A client with bipolar disorder has developed hypothyroidism with a goiter. When the nurse obtains a medication history, which medication taken by the client does the nurse understand may cause this disorder? a) Phenytoin (Dilantin) b) Levothyroxine (Synthroid) c) Sertraline (Zoloft) d) Lithium carbonate

Lithium carbonate Certain goitrogenic agents, such as lithium carbonate (used in the treatment of manic-depressive states) and the antithyroid drugs propylthiouracil and methimazole, in continuous dosage can block hormone synthesis and produce hypothyroidism with goiter.

While teaching a science class, the instructor mentions that both autocrine and paracrine hormonal actions occur without entering the bloodstream. A student aks, "What cells do paracrine actions affect?" Which response is correct?

Local

A client who is being treated in the critical care unit is experiencing a slowing of blood flow to the central retinal artery, ultimately resulting in edema of the client's optic papilla. Which assessment would most likely address the etiology of this health problem?

Monitoring of the client's intracranial pressure Explanation: The client is experiencing papilledema, which is most often caused by an increase in ICP. It is noninfectious and is unrelated to systemic blood pressure or lymphedema.

Which of the following best describes the half-life of a highly protein bound drug such as thyroxine (99% protein bound)? The half-life would be:

Much longer to reduce the concentration of the hormone by one half. Explanation: The half-life of a hormone—the time it takes for the body to reduce the concentration of the hormone by one half—is positively correlated with its percentage of protein binding. Thyroxine, which is more than 99% protein bound, has a half-life of 6 days whereas Aldosterone, 15% bound, has a half-life of only 25 minutes. Page 757

A young teenager is brought to the clinic for an eye exam. She has been complaining that she is having difficulty in school as she cannot see the blackboard as clearly as she used to. She tells the nurse that it is blurry. What should the nurse suspect is wrong with this child? a) Myopia b) Hyperopia c) Presbyopia d) Immature cataracts

Myopia Myopia is a condition also referred to as nearsightedness where the person can see close objects without problems, but distant objects are blurred. It can be corrected with an appropriate concave-surface lens. -Hyperopia is when near images are blurred. -Presbyopia refers to a decrease in accommodation and is usually seen due to aging. - Immature cataracts deal with a change in the lens opacity which leads to interference of the transmission of light to the retina.

In which situation would an increase of glucagon be expected? a) NPO before surgery b) Fever c) High anxiety d) Hyperglycemia

NPO before surgery The primary role of glucagon is to raise plasma glucose levels. When fasting, the blood glucose level can drop and if there is no food to raise it, glucagon will be released to access liver glycogen stores. A client with hyperglycemia would have low glucagon secretion. High anxiety would increase secretion of catecholamines such as epinephrine. Fever is a stressor that raises growth hormone levels.

What is the most common mechanism of hormone control?

Negative feedback Explanation: With negative feedback, the most common mechanism of hormone control, some feature of hormone action directly or indirectly inhibits further hormone secretion so that the hormone level returns to an ideal level or set point. Page 762

What is the most common mechanism of hormone control? a) Positive feedback b) Hypothalamic-pituitary-target cell feedback c) Negative feedback d) Hypothalamic-pituitary-adrenal axis

Negative feedback With negative feedback, the most common mechanism of hormone control, some feature of hormone action directly or indirectly inhibits further hormone secretion so that the hormone level returns to an ideal level or set point.

What is the most common mechanism of hormone control?

Negative feedback Explanation: With negative feedback, the most common mechanism of hormone control, some feature of hormone action directly or indirectly inhibits further hormone secretion so that the hormone level returns to an ideal level or set point.

The hormone levels in the body need to be kept within an appropriate range. How is this accomplished for many of the hormones in the body? Positive feedback loop Negative feedback loop Regulated feedback loop Sensory feedback loop

Negative feedback loop

The nurse is caring for a client who has been diagnosed with Meniere disease. The nurse expects which of the following medications will be prescribed to manage the symptoms?

Promethazine (Phenergan).

The nurse is assessing the hearing of a 25-year-old client who reports chronic painless tinnitus and difficulty hearing. The nurse suspects the client may have which diagnosis?

Otosclerosis Explanation: Otosclerosis may begin at any time in life but usually does not appear until after puberty, most frequently between the ages of 20 and 30 years. The symptoms of otosclerosis involve an insidious hearing loss, tinnitus, and vertigo. Acute otitis media is characterized by otalgia, fever, hearing loss, and inflammation. Otitis media effusion is an accumulation of fluid in the middle ear without signs of infection. It may cause hearing loss. Mastoiditis can occur when typanic membranes perforate.

The nurse is providing education to a client with otosclerosis. Which statement by the client would cause the nurse to decide that the client needs further education?

Otosclerosis usually appears prior to puberty. Explanation: Initially the client with otosclerosis is unable to hear a whisper and this results in progressive deafness. It is a familial disease that typically presents after puberty.

The cornea functions as a protective membrane. What layers of tissues form the cornea? Select all that apply.

Outer epithelial layer Substantia propria or stroma Inner endothelial layer Explanation: Three layers of tissue form the cornea: an extremely thin outer epithelial layer, which is continuous with the bulbar conjunctiva; a middle layer called the substantia propria or stroma; and an inner endothelial layer, which lies next to the aqueous humor of the anterior chamber. The three layers of the cornea are separated by two important basement membranes: Bowman and Descemet membranes.

Conductive hearing loss can occur for a variety of reasons, including foreign bodies in the ear canal, damage to the ear drum, or disease. What disease is associated with conductive hearing loss?

Paget disease

A teen with newly diagnosed pituitary dwarfism asks the nurse why growth hormone must be taken as an injection rather than a pill like the steroids used for arthritis. Which response best answers the client's question?

Peptide proteins will be digested instead of entering the bloodstream.

The nurse is conducting the Rinne test. What is included in this test?

Placing a vibrating tuning fork on the client's mastoid bone and in front of the ear in an alternating fashion Explanation: The Rinne test compares air and bone conduction. The test is done by alternately placing the tuning fork on the mastoid bone and in front of the ear canal. In conductive losses, bone conduction exceeds air conduction. In sensorineural losses, the opposite occurs.

A client has been experiencing elevated blood glucose levels. The nurse anticipates that which assessment data correlates with hyperglycemia?

Polydipsia

A client is diagnosed with diabetic ketoacidosis (DKA) in the emergency department. Which clinical manifestations will the client likely exhibit?

Polyuria, polydipsia, voming, and fatigue

A patientreceives steroids for several months to treat an inflammatory condition. Which of the following actions by the primary healthcare provider indicates an understanding of the negative feedback mechanism when the patient no longer needs the medication?

Prescribing a tapering dose of the medication over weeks Explanation: Negative feedback occurs when secretion of one hormone causes a reduction in the secretion of the hormone that stimulates production of the first hormone. In this case, adrenocorticotropic hormone (ACTH) manufactured by the anterior pituitary would normally stimulate release of cortisol, but with the increase of cortisol produced by the secreting tumor, enough already floods the system there has been a reduction in ACTH levels. Sudden withdrawal of the medication would leave the patient without glucocorticoids and risk an Addisonian crisis. Gradual withdrawal of the medication allows the pituitary to measure the drop in cortisol levels and begin secreting ACTH. Page 762

A patientreceives steroids for several months to treat an inflammatory condition. Which of the following actions by the primary healthcare provider indicates an understanding of the negative feedback mechanism when the patient no longer needs the medication?

Prescribing a tapering dose of the medication over weeks Explanation: Negative feedback occurs when secretion of one hormone causes a reduction in the secretion of the hormone that stimulates production of the first hormone. In this case, adrenocorticotropic hormone (ACTH) manufactured by the anterior pituitary would normally stimulate release of cortisol, but with the increase of cortisol produced by the secreting tumor, enough already floods the system there has been a reduction in ACTH levels. Sudden withdrawal of the medication would leave the patient without glucocorticoids and risk an Addisonian crisis. Gradual withdrawal of the medication allows the pituitary to measure the drop in cortisol levels and begin secreting ACTH.

While reviewing the concept of nuclear receptors with a group of pathophysiology students, the instructor uses the example of clients with type 2 diabetes mellitus taking pioglitazone, a thiazolidinedione medication. Because of the peroxisome proliferator-activated receptors (PPARs), the drug has which effect on the clients' diabetes? Select the best answer.

Promote glucose uptake and increase the synthesis of certain proteins involved in fat metabolism, which reduces levels of certain types of lipids.

While reviewing the concept of nuclear receptors with a group of pathophysiology students, the instructor uses the example of clients with type 2 diabetes mellitus taking pioglitazone, a thiazolidinedione medication. Because of the peroxisome proliferator-activated receptors (PPARs), the drug has which effect on the clients' diabetes? Select the best answer. Increases the clients' metabolic rate, thereby giving them more energy, which results in weight loss. Help the body burn fats more effectively thereby preventing any cardiovascular problems like myocardial infarction. Promote glucose uptake and increase the synthesis of certain proteins involved in fat metabolism, which reduces levels of certain types of lipids. This medicines increases insulin resistance in muscle and fat and increases the amount of glucose produced by the liver.

Promote glucose uptake and increase the synthesis of certain proteins involved in fat metabolism, which reduces levels of certain types of lipids.

While reviewing the concept of nuclear receptors with a group of pathophysiology students, the instructor uses the example of clients with type 2 diabetes mellitus taking pioglitazone, a thiazolidinedione medication. Because of the peroxisome proliferator-activated receptors (PPARs), the drug has which effect on the clients' diabetes? Select the best answer.

Promote glucose uptake and increase the synthesis of certain proteins involved in fat metabolism, which reduces levels of certain types of lipids. Explanation: The peroxisome proliferator-activated receptors (PPARs) bind intracellular lipid metabolites and regulate the transcription of genes involved in lipid metabolism and adipose tissue metabolism. Pioglitazone is not given solely to increase metabolic rate or to just burn fats, even though this may occur. Thiazolidimedione medications lower insulin resistance in muscle and fat. They also reduce glucose produced by the liver. Page 759

While reviewing the concept of nuclear receptors with a group of pathophysiology students, the instructor uses the example of clients with type 2 diabetes mellitus taking pioglitazone, a thiazolidinedione medication. Because of the peroxisome proliferator-activated receptors (PPARs), the drug has which effect on the clients' diabetes? Select the best answer. a) Increases the clients' metabolic rate, thereby giving them more energy, which results in weight loss. b) This medicines increases insulin resistance in muscle and fat and increases the amount of glucose produced by the liver. c) Help the body burn fats more effectively thereby preventing any cardiovascular problems like myocardial infarction. d) Promote glucose uptake and increase the synthesis of certain proteins involved in fat metabolism, which reduces levels of certain types of lipids.

Promote glucose uptake and increase the synthesis of certain proteins involved in fat metabolism, which reduces levels of certain types of lipids. The peroxisome proliferator-activated receptors (PPARs) bind intracellular lipid metabolites and regulate the transcription of genes involved in lipid metabolism and adipose tissue metabolism. Pioglitazone is not given solely to increase metabolic rate or to just burn fats, even though this may occur. Thiazolidimedione medications lower insulin resistance in muscle and fat. They also reduce glucose produced by the liver

While reviewing the concept of nuclear receptors with a group of pathophysiology students, the instructor uses the example of clients with type 2 diabetes mellitus taking pioglitazone, a thiazolidinedione medication. Because of the peroxisome proliferator-activated receptors (PPARs), the drug has which effect on the clients' diabetes? Select the best answer.

Promote glucose uptake and increase the synthesis of certain proteins involved in fat metabolism, which reduces levels of certain types of lipids. Explanation: The peroxisome proliferator-activated receptors (PPARs) bind intracellular lipid metabolites and regulate the transcription of genes involved in lipid metabolism and adipose tissue metabolism. Pioglitazone is not given solely to increase metabolic rate or to just burn fats, even though this may occur. Thiazolidimedione medications lower insulin resistance in muscle and fat. They also reduce glucose produced by the liver.

While reviewing the concept of nuclear receptors with a group of pathophysiology students, the instructor uses the example of clients with type 2 diabetes mellitus taking pioglitazone, a thiazolidinedione medication. Because of the peroxisome proliferator-activated receptors (PPARs), the drug has which effect on the clients' diabetes?

Promotes glucose uptake and increase the synthesis of certain proteins involved in fat metabolism, which reduces levels of certain types of lipids.

Select the most accurate statement regarding measurements of urinary hormone:

Provide a better measure of hormone levels during a designated period. Explanation: Measurements of urinary hormone or hormone metabolite excretion often are done on a 24-hour urine sample and provide a better measure of hormone levels during that period than hormones measured in an isolated blood sample. The advantages are relative ease of obtaining urine samples and blood sampling is not required. The disadvantages are that timed urine collections often are difficult to obtain and urine samples may be accidentally discarded or inaccurately preserved; drugs or disease states that alter hormone metabolism may interfere with the test results. Page 764

Cortical blindness is the bilateral loss of the primary visual cortex. What is retained in cortical blindness?

Pupillary reflexes Explanation: Crude analysis of visual stimulation at reflex levels, such as eye- and head-orienting responses to bright moving lights, pupillary reflexes, and blinking at sudden bright lights, may be retained even though vision has been lost.

Cortical blindness is the bilateral loss of the primary visual cortex. What is retained in cortical blindness?

Pupillary reflexes Explanation: Crude analysis of visual stimulation at reflex levels, such as eye- and head-orienting responses to bright moving lights, pupillary reflexes, and blinking at sudden bright lights, may be retained even though vision has been lost.

A client seeks medical care when he wrecks his car because of poor eyesight. At the time of admission, his blood glucose level was 390 mg/dL. The client is diagnosed with diabetes (type 2). The ophthalmologist must perform an urgent intravitreal injection. The nurse explains this to the client by stating the doctor will:

Put a needle with syringe into your eyeball and inject some medication to decrease active bleeding.

Many hormones are measured for diagnostic reasons by using the plasma levels of the hormones. What is used today to measure plasma hormone levels?

Radioimmunoassay methods

Many hormones are measured for diagnostic reasons by using the plasma levels of the hormones. What is used today to measure plasma hormone levels?

Radioimmunoassay methods Explanation: Real progress in measuring plasma hormone levels came more than 40 years ago with the use of competitive binding and the development of radioimmunoassay methods. The other answers are incorrect. Page 764

The nurse screening for diabetes mellitus at a health fair obtains these results. Which client should be referred to a primary health care provider for further evaluation?

Random blood glucose 195 mg/dL (10.82 mmol/L)

The nurse screening for diabetes mellitus at a health fair obtains these results. Which client should be referred to a primary healthcare provider for further evaluation? a) Fasting blood glucose 89 mg/dl b) Urine ketones 0 c) Hemoglogin A1c 6.0% d) Random blood glucose 195 mg/dl

Random blood glucose 195 mg/dl Laboratory values that are considered normal are hemoglobin A1c less than 6.5 percent, fasting plasma glucose of (FPG) less than 100 mg/dl or less than 140 mg/dl 2 hours after an oral glucose tolerance test (GTT). Urine should be free of ketones. A hemoglobin A1c value that is greater than or equal to 6.5 percent, a fasting blood glucose greater than 126mg/dl, or a blood glucose level greater than 200 mg/dl 2 hours after a glucose tolerance test (GTT) indicate diabetes mellitus. Values between these levels are considered to place clients at increased risk for diabetes mellitus and require further evaluation. A random blood glucose should is expected to correlate with the two-hour GTT results and should be below 200 mg/dl.

A woman in her 28th week of pregnancy tests positive for gestational diabetes mellitus and begins to follow a nutritional plan at home. What result at the follow-up visit indicates a successful outcome? a) Gained 5 pounds in one week b) 1% ketones present in urine c) Glycosylated hemoglobin 7.2% d) Random blood glucose 85 mg/dl

Random blood glucose 85 mg/dl The goals of the nutritional plan for gestational diabetes mellitus (GDM) include normal glucose levels, no ketosis, proper weight gain for the pregnancy, and adequate nutrition for fetal health.

Which manifestation would a nurse expect when assessing a child with insufficient growth hormone (GH) secretion?

Rank below 10% on the growth chart

An example of a single hormone that can exert effects in different tissues, erythropoietin, made in the kidney stimulates the bone marrow to produce: a) Natural killer cells. b) Red blood cells. c) Platelets. d) Mast cells.

Red blood cells. A characteristic of hormones is that a single hormone can exert various effects in different tissues. For example, erythropoietin, a traditional circulating hormone, is made in the kidney and stimulates erythropoesis in the bone marrow.

What are the clinical manifestations of Ménière disease? Select all that apply.

Severe ringing in the ear(s) Feeling of fullness in the ear(s) Episodes of disabling vertigo Dizziness triggered by turning the head Explanation: Ménière's disease is characterized by severe, disabling episodes of tinnitus; feelings of ear fullness; and violent rotary vertigo. Benign paroxysmal positional vertigo (BPPV) is a condition believed to be caused by free-floating particles in the posterior semicircular canal. It presents as a sudden onset of dizziness or vertigo that is provoked by certain changes in head position.

A client with diabetes carries insulin with him at all times. At 11:35, he obtains a blood glucose reading of 218 mg/dL (12.1 mmol/L) and self-administers a dose of insulin in anticipation of eating lunch at noon. What type of insulin did he most likely inject?

Short acting

A parent arrives in the endocrinology clinic with her 8-year-old son, concerned about his rapid development and tall stature. What significant assessment finding does the nurse recognize is important to report to the physician related to the development of precocious puberty? a) Multiple dental caries b) Enlarged head circumference c) Significant genital enlargement d) The child is 20 pounds over his target weight for height

Significant genital enlargement Diagnosis of precocious puberty is based on physical findings of early thelarche, adrenarche, and menarche. The most common sign in boys is early genital enlargement. Radiologic findings may indicate advanced bone age. People with precocious puberty are unusually tall for their age as children but short as adults because of the early closure of the epiphyses.

A client with severe hypothyroidism is presently experiencing hypothermia. What nursing intervention is a priority in the care of this client? a) Keeping the client in a hypothermic state in order to prevent renal failure b) Active rewarming of the client to increase body temperature rapidly c) Slow rewarming of the client to prevent vasodilation and vascular collapse d) Placing the client on a hyperthermia blanket and using heated saline in order to induce vasodilation

Slow rewarming of the client to prevent vasodilation and vascular collapse If hypothermia is present, active rewarming of the body is contraindicated because it may induce vasodilation and vascular collapse. Prevention is preferable to treatment and entails special attention to high risk populations, such as women with a history of Hashimoto thyroiditis.

The nurse is caring for a client diagnosed with diabetes mellitus who is reporting burning pain of his feet. The nurse would interpret this as: a) Somatic neuropathy b) Peripheral vascular disease c) Autonomic neuropathy d) Intermittent claudication

Somatic neuropathy Somatic neuropathy is usually bilateral and symmetric and associated with diminished perception of vibration, pain, and temperature, particularly in the lower extremities. The autonomic neuropathies result in disorders of vasomotor function, decreased cardiac responses, inability to empty the bladder, gastrointestinal motility problems, and sexual dysfunction. Peripheral vascular disease is a general complication of diabetes, and intermittent claudication is pain associated with arterial insufficiency in the legs

Which events are involved in the negative feedback mechanism that keeps the serum thyroid hormone level within appropriate range? Select all that apply. You Selected: Stimulation of sensors in the hypothalamus Correct response: Stimulation of sensors in the hypothalamus Stimulation of sensors in the anterior pituitary gland Decrease in secretion of thyroid stimulating hormone (TSH) Increased levels of thyroid hormone

Stimulation of sensors in the hypothalamus Stimulation of sensors in the anterior pituitary gland Decrease in secretion of thyroid stimulating hormone (TSH) Increased levels of thyroid hormone

A 21 year-old female is suspected of having inadequate function of her hypothalamic-pituitary-thyroid system. Her care provider is planning to inject thyrotropin-releasing hormone (TRH) and then measure her levels of TSH. Which of the following diagnostic tests is being performed? a) Metabolite excretion test b) Suppression test c) Stimulation test d) Radioimmunoassay (RIA) test

Stimulation test A stimulation test involves the introduction of an element that stimulates the production of another factor or hormone followed by measurement of that hormone. This is not the case in a suppression test, RIA test of metabolite excretion test.

The physician suspects a client may be experiencing hypofunction of an endocrine organ. Select the most appropriate test to determine organ function.

Stimulation tests Explanation: Stimulation tests are used when hypofunction of an endocrine organ is suspected. Suppression tests are used when hyperfunction of an endocrine organ is suspected. Genetic testing is used for DNA analysis, and imaging may be used as a follow-up after the diagnosis.

A mother brings her toddler to the clinic and tells the nurse that she thinks something is wrong with the baby's eyes. Upon further assessment the nurse notices that the child has upper deviation in movement in only one eye. What diagnosis should the nurse suspect for this child?

Strabismus Explanation: Strabismus, or squint, refers to any abnormality of eye coordination or alignment that results in loss of binocular vision. Amblyopia refers to lazy eye. Glaucoma is a chronic degenerative optic neuropathy. Cataract affects the lens of the eye.

When a client reports that he feels like he is spinning but the room is stable, the nurse should document the finding as which of the following?

Subjective vertigto

When a client reports that he feels like he is spinning but the room is stable, the nurse should document the finding as which of the following?

Subjective vertigto.

The nurse cares for a client with severe chronic obstructive pulmonary disease who has been taking oral steroids for several years. Which response in the client's endocrine control will the nurse expect?

Suppression of the client's hypothalamic-pituitary-target cell system

The nurse is teaching a client with diabetes and the family about the signs and symptoms of hypoglycemia. The client asks what produces signs and symptoms of headache, disturbed behavior, coma, and seizures. The best response would be: a) Hypoglycemia causes ketone breakdown. b) Hepatic glycogenolysis. c) The brain relies on blood glucose as its main energy source. d) Insulin produces a rebound hyperglycemia.

The brain relies on blood glucose as its main energy source. Because the brain relies on blood glucose as its main energy source, hypoglycemia produces behaviors related to altered cerebral function. Headache, difficulty in problem solving, disturbed or altered behavior, coma, and seizures may occur.

The parents of a 6-month-old child who was born with a blocked tear duct are concerned because their child has been waking up the past few mornings with inflammation in the left eye and yellow, dried secretions preventing eye opening in spite of massaging the tear duct. The nurse would anticipate which of the following?

The child has developed dacryocystitis. Explanation: Some babies are born without spontaneous opening of the tear duct(s) before birth, causing the inability of the tears to drain and increasing the risk of infection of the lacrimal sac (dacryocystitis). Signs and symptoms of dacryocystitis include excess secretions in the affected eye, which may cause crusting; excessive tearing; swelling; pain; and tenderness. The initial treatments would include warm compresses to the affected eye, antibiotics, and if appropriate, analgesics. Consideration of surgical probing of the tear duct is reserved for chronic cases of dacryocystitis.

A very tall 8-year-old child is brought by a parent to the clinic for a follow-up visit after diagnostic testing reveals a diagnosis of gigantism. What objective assessment data gathered would the nurse recognize correlates with the child's condition? a) The client has increased cortisol levels, truncal obesity, and altered fat distribution. b) The child has excess levels of growth hormones and the epiphyses of the long bones are not fused. c) The child has an increase in the TSH level and has a goiter. d) The child has fusion of the epiphyses of the long bones and increased growth hormone levels.

The child has fusion of the epiphyses of the long bones and increased growth hormone levels. Growth hormone excess occurring before puberty and the fusion of the epiphyses of the long bones results in gigantism. Excessive secretion of GH by somatotrope adenomas causes gigantism in the prepubertal child. It occurs when the epiphyses are not fused and high levels of IGF-1 stimulate excessive skeletal growth. When GH excess occurs in adulthood or after the epiphyses of the long bones have fused, the condition is referred to as acromegaly.

The nurse is providing discharge instructions for a client with Graves' disease who has ophthalmopathy. What should the nurse be sure to include in the instructions to decrease exacerbation of this clinical manifestation? a) The client should be strongly encouraged not to drink any alcohol. b) The client should be strongly urged not to smoke. c) The client should be informed that he should not be in contact with other people during the acute phase. d) The client should be informed that if he begins to feel symptoms getting worse, he should take an extra dose of their medication.

The client should be strongly urged not to smoke. Ophthalmopathy can also be aggravated by smoking, which should be strongly discouraged. It is not necessary for the client to avoid contact with others. Alcohol is not contraindicated but should be limited when taking any medication regimen. The client should not adjust the doses of medications without first consulting the physician.

The nurse is caring for a client with decreased serum protein levels secondary to liver failure. When administering medications that are highly protein bound, the nurse anticipates the resulting drug level will respond in which of these ways?

The drug level will be elevated as lack of protein allows more free drug to circulate. Explanation: Peptide hormones and protein hormones usually circulate unbound in the blood. Specific carrier proteins synthesized in the liver carry steroid hormones and thyroid hormone, for example. Drugs that compete with a hormone for binding with transport carrier molecules increase hormone action by increasing the availability of the active unbound hormone. Page 757

A client undergoing an evaluation of their hormone levels asks "What regulates the hormone levels?" Which response by the nurse would be considered most accurate? The hypothalamic-pituitary-target cell system Exogenous forms of hormones The positive feedback loop The hypophysial portal system

The hypothalamic-pituitary-target cell system

When describing to a client newly diagnosed with diabetes how insulin is regulated, the nurse will draw upon her knowledge of which hormonal regulation mechanism?

The hypothalamic-pituitary-target cell system

A client who is referred to the endocrinologist's office for an evaluation of his hormone levels asks what regulates the hormone levels. The best response would be that hormone levels in the body are primarily regulated by:

The hypothalamic-pituitary-target cell system Explanation: The levels of many of the hormones are regulated by feedback mechanisms that involve the hypothalamic-pituitary-target cell system. Positive feedback control refers to rising levels of a hormone that causes another gland to release a hormone that is stimulating to the first. The hypophyseal portal system connects the supraoptic and paraventricular nuclei of the hypothalamus with the posterior pituitary gland. Exogenous forms of hormones (given as drug preparations) can influence the normal feedback control of hormone production and release. Page 759

When describing to a newly diagnosed diabetic client how insulin is regulated, the nurse will draw upon her knowledge of which hormonal regulation mechanism?

The hypothalamic-pituitary-target cell system The hypophysis (pituitary plus hypothalamus) and hypothalamus stimulatory hormones regulate the release and synthesis of anterior pituitary hormones. The levels of hormones such as insulin and antidiuretic hormone (ADH) are regulated by feedback mechanisms that monitor substances such as glucose (insulin) and water (ADH) in the body. The levels of many of the hormones are regulated by feedback mechanisms that involve the hypothalamic-pituitary-target cell system.

A client who is referred to the endocrinologist's office for an evaluation of his hormone levels asks what regulates the hormone levels. The best response would be that hormone levels in the body are primarily regulated by: a) The hypophysial portal system b) The hypothalamic-pituitary-target cell system c) Exogenous forms of hormones d) The positive feedback loop

The hypothalamic-pituitary-target cell system The levels of many of the hormones are regulated by feedback mechanisms that involve the hypothalamic-pituitary-target cell system. Positive feedback control refers to rising levels of a hormone that causes another gland to release a hormone that is stimulating to the first. The hypophyseal portal system connects the supraoptic and paraventricular nuclei of the hypothalamus with the posterior pituitary gland. Exogenous forms of hormones (given as drug preparations) can influence the normal feedback control of hormone production and release.

A client who is referred to the endocrinologist's office for an evaluation of his hormone levels asks what regulates the hormone levels. The best response would be that hormone levels in the body are primarily regulated by:

The hypothalamic-pituitary-target cell system Explanation: The levels of many of the hormones are regulated by feedback mechanisms that involve the hypothalamic-pituitary-target cell system. Positive feedback control refers to rising levels of a hormone that causes another gland to release a hormone that is stimulating to the first. The hypophyseal portal system connects the supraoptic and paraventricular nuclei of the hypothalamus with the posterior pituitary gland. Exogenous forms of hormones (given as drug preparations) can influence the normal feedback control of hormone production and release.

Which of the following statements best captures the relationship between the hypothalamus and the pituitary gland as it relates to endocrine function?

The hypothalamus receives input from numerous sources throughout the body and directs the pituitary to then control many target glands and cells. Explanation: The hypothalamus can be viewed as a bridge by which signals from multiple systems are relayed to the pituitary gland. The hypothalamus collects data from sources throughout the body rather than directly measuring levels and communication normally flows from the hypothalamus to the pituitary. Page 759

A newborn is screened for congenital hypothyroidism and is found to have the disorder. When educating the mother about the importance of the infant's taking thyroid hormone supplement, what should be included in the education? a) The infant will have to take the medication only until he reaches puberty. b) Once the dosage levels are adjusted, the infant will remain on the same amount throughout life. c) The medication dosage will increase every 6 months after laboratory testing. d) The infant will have dosage levels adjusted as he grows.

The infant will have dosage levels adjusted as he grows. Congenital hypothyroidism is treated by hormone replacement. Evidence indicates that it is important to normalize T4 levels as rapidly as possible because a delay is accompanied by poorer psychomotor and mental development. Dosage levels are adjusted as the child grows. When early and adequate treatment regimens are followed, the risk of mental retardation in infants detected by screening programs is essentially nonexistent.

A patient is managing his type 2 diabetes with exercise and diet. He has a fasting blood sugar level (FBS) of 80 mg/dL and a hemoglobin A1C of 5%. Based on these findings, which of the following can the nurse assume? a) The patient is at risk for an insulin reaction. b) The patient is at risk for developing hyperglycemia. c) The patient is achieving normal glycemic control. d) The patient needs to modify his diet related to the low readings.

The patient is achieving normal glycemic control. The reading for the FBS is appropriate (<100 mg/dL is normal) and the hemoglobin A1C level (<6.5) shows good control of glucose levels over a 6- to 12-week period. This patient should continue his current exercise and diet routine, which is working well for him.

The nurse is teaching a client who has been newly diagnosed with hypothyroidism about the function of the thyroid. Which of these does the nurse explain to the client is the role of the thyroid gland? The thyroid gland is responsible for increasing the metabolic rate. The thyroid gland releases neurotransmitters when the "flight or flight" mechanism is stimulated. The thyroid gland is responsible for regulating serum calcium levels. The thyroid gland promotes development of secondary sex characteristics.

The thyroid gland is responsible for increasing the metabolic rate.

The nurse is teaching a client who has been newly diagnosed with hypothyroidism about the function of the thyroid. Which of these does the nurse explain to the client is the role of the thyroid gland?

The thyroid gland is responsible for increasing the metabolic rate. Explanation: The thyroid gland produces thyroid hormones, T3 and T4. These hormones increase the metabolic rate; increase protein and bone turnover; increase responsiveness to catecholamines; are necessary for fetal and infant growth and development. The parathyroid gland regulates calcium metabolism. The adrenal glands regulate flight or fight and the testes or ovaries regulate development of secondary sex characteristics. Page 756

The nurse is teaching a client who has been newly diagnosed with hypothyroidism about the function of the thyroid. Which of these does the nurse explain to the client is the role of the thyroid gland? a) The thyroid gland is responsible for increasing the metabolic rate. b) The thyroid gland promotes development of secondary sex characteristics. c) The thyroid gland is responsible for regulating serum calcium levels. d) The thyroid gland releases neurotransmitters when the "flight or flight" mechanism is stimulated.

The thyroid gland is responsible for increasing the metabolic rate. The thyroid gland produces thyroid hormones, T3 and T4. These hormones increase the metabolic rate; increase protein and bone turnover; increase responsiveness to catecholamines; are necessary for fetal and infant growth and development. The parathyroid gland regulates calcium metabolism. The adrenal glands regulate flight or fight and the testes or ovaries regulate development of secondary sex characteristics.

For complete function of the eyes, it is necessary that the two eyes point toward the same fixation point and that the retinal and central nervous system visual acuity mechanisms function. How many pairs of extraocular muscles does binocular vision depend on?

Three

Binocular vision depends on the coordination of how many pairs of extraocular nerves that provide for the conjugate eye movements?

Three Explanation: Binocular vision depends on the coordination of three pairs of extraocular nerves that provide for the conjugate eye movements, with optical axes of the two eyes maintained parallel with one another as the eyes rotate in their sockets.

The nurse administers a glucocorticoid medication to a client with pneumonia. Which of these does the nurse teach the client is the purpose of the medication?

To decrease airway inflammation Explanation: Glucocorticoids affect metabolism of all nutrients and have anti-inflammatory effects, which can assist with airway inflammation. Page 756

The nurse explains to a client in labor who has demonstrated ineffective contractions impeding progression of labor that the health care provider has added oxytocin infusion to the orders. Which of these does the nurse teach the client is the purpose of oxytocin?

To stimulate contraction of the uterus Explanation: The role of oxytocin is to stimulate contraction of the pregnant uterus and milk ejection from breasts after childbirth. An infusion of oxytocin will promote effective contractions. Page 756

The nurse explains to a client in labor who has demonstrated ineffective contractions impeding progression of labor that the health care provider has added oxytocin infusion to the orders. Which of these does the nurse teach the client is the purpose of oxytocin? a) To stimulate contraction of the uterus b) To protect the fetus from effects of forceful contractions c) To reduce pain of uterine contractions d) To contribute to maturation of fetal lungs

To stimulate contraction of the uterus The role of oxytocin is to stimulate contraction of the pregnant uterus and milk ejection from breasts after childbirth. An infusion of oxytocin will promote effective contractions.

A 2-year-old child who has had otitis media (OM) for 4 months and been treated with several courses of antibiotics now appears to have some hearing loss. The nurse anticipates that the most appropriate treatment for the child would be:

Tympanostomy tube insertion Explanation: Tympanostomy tubes should be the next course of action with the child having hearing loss and OM persisting for 4 months or longer. The parents should be taught to continue to monitor for signs of OM.

A 2-year-old child who has had otitis media (OM) for 4 months and been treated with several courses of antibiotics now appears to have some hearing loss. The nurse anticipates that the most appropriate treatment for the child would be: a) Tympanostomy tube insertion b) Monitoring for complications of OM c) The child will need a hearing aid d) Retreating with a stronger antibiotic

Tympanostomy tube insertion Tympanostomy tubes should be the next course of action with the child having hearing loss and OM persisting for 4 months or longer. The parents should be taught to continue to monitor for signs of OM.

A 2-year-old child who has had otitis media (OM) for 4 months and been treated with several courses of antibiotics now appears to have some hearing loss. The nurse anticipates that the most appropriate treatment for the child would be:

Tympanostomy tube insertion.

A client with a history of an endocrine disorder exhibits signs and symptoms of hormone deficiency. Which of the following processes would the client's care team most likely rule out first as a contributing factor? a) Hormone production is sufficient, but affinity on the part of the target cells is lacking. b) The client's target cells lack sufficient receptors for the hormone in question. c) Up-regulation has increased the sensitivity of the body to particular hormone levels. d) The process of down-regulation has resulted in decreased hormone sensitivity.

Up-regulation has increased the sensitivity of the body to particular hormone levels. Up-regulation is a response to low hormone levels in which the number of receptors increases. As such, it would not likely result in signs and symptoms of deficiency but is rather a compensatory mechanism that counters a deficiency. Insufficient numbers of receptors, low affinity and down-regulation could all contribute to signs and symptoms of a hormone deficiency.

A client with type 1 diabetes mellitus wishes to stop taking insulin injections. What option is appropriate? a) Using an insulin infusion pump b) Taking the herb chromium picolinate c) Increasing daily aerobic activity d) Taking metformin (Glucophage)

Using an insulin infusion pump Clients with type 1 diabetes mellitus require exogenous insulin because they have absolute lack of their own secretion. Insulin is a protein that would be digested if taken orally. To avoid injections, clients could use an insulin pump which provides continuous infusion through a catheter placed under the skin that is changed every few days. While some herbal preparations may help lower blood glucose, they do not replace insulin. While aerobic activity helps mobilize glucose, it does not replace insulin. Metformin (Glucophage) is an insulin sensitizer that enhances insulin utilization, but does not replace insulin

The nurse is preparing a client for a surgical procedure to create an endolymphatic shunt. The nurse understands that this procedure aims to relieve which symptom?

Vertigo Explanation: A surgical method to treat Meniere disease includes the creation of an endolymphatic shunt in which excess endolymph from the inner ear is diverted into the subarachnoid space or the mastoid area. This relieves symptoms associated with Meniere disease, such as vertigo. The other symptoms are not manifestations of Meniere disease.

The nurse is preparing a client for a surgical procedure to create an endolymphatic shunt. The nurse understands that this procedure aims to relieve which of the following symptoms? a) Strabismus b) Vertigo c) Otalgia d) Diplopia

Vertigo A surgical method to treat Meniere disease includes the creation of an endolymphatic shunt in which excess endolymph from the inner ear is diverted into the subarachnoid space or the mastoid area. This relieves symptoms associated with Meniere disease, such as vertigo. The other symptoms are not manifestations of Meniere disease.

The nurse assessing a patient with Ménière disease expects the patient to have which of the following symptoms?

Vertigo.

A client is having difficulty with balance. The nurse understands that the area of the ear that impacts balance is:

Vestibular apparatus Explanation: The vestibular system maintains and assists recovery of stable body and head position and balance through control of postural reflexes. The vestibular system includes the three semicircular canals.

A young patient is brought to the clinic with symptoms of a runny nose, small amount of drainage from both eyes, fever, and malaise. The mother states to the nurse, "I knew she was going to be sick. That swimming pool she played in 2 days ago was filthy." What disease should the nurse suspect this child is experiencing?

Viral conjunctivitis

A client presents to the emergency department complaining of loss of part of his vision. An MRI with contrast reveals multiple aneurysms of the circle of Willis. The client is diagnosed with "bitemporal heteronymous anopia." For this client, what effect will this have on his vision?

With both eyes open, the client has full binocular visual fields

The obstetrical nurse is caring for a client who has been treated for gestational diabetes. When teaching the client about the causes of gestational diabetes, the nurse should include which of these risk factors in the teaching? a) Woman with a family history of diabetes b) First birth occurring during the teenage years c) Woman who has had a child under 5 pounds d) First pregnancy

Woman with a family history of diabetes Gestational diabetes occurs most commonly in African American, Hispanic/Latino American and Native American women. It most frequently affects women with a family history of diabetes, a history of stillbirth or spontaneous abortion, women who previously gave birth newborn with fetal anomaly or had a previous large- or heavy-for-date infant, those who are obese, those of advanced maternal age, or those who have had five or more pregnancies.

Which statement by the patient indicates a need for additional instruction in administering insulin? a. "I need to rotate injection sites among my arms, legs, and abdomen each day." b. "I can buy the 0.5 mL syringes because the line markings will be easier to see." c. "I should draw up the regular insulin first after injecting air into the NPH bottle." d. "I do not need to aspirate the plunger to check for blood before injecting insulin."

a. "I need to rotate injection sites among my arms, legs, and abdomen each day."

A patient receives aspart (NovoLog) insulin at 8:00 AM. Which time will it be most important for the nurse to monitor for symptoms of hypoglycemia? a. 10:00 AM b. 12:00 AM c. 2:00 PM d. 4:00 PM

a. 10:00 AM

Which information will the nurse include in teaching a female patient who has peripheral arterial disease, type 2 diabetes, and sensory neuropathy of the feet and legs? a. Choose flat-soled leather shoes. b. Set heating pads on a low temperature. c. Use callus remover for corns or calluses. d. Soak feet in warm water for an hour each day.

a. Choose flat-soled leather shoes.

In order to assist an older diabetic patient to engage in moderate daily exercise, which action is most important for the nurse to take? a. Determine what type of activities the patient enjoys. b. Remind the patient that exercise will improve self-esteem. c. Teach the patient about the effects of exercise on glucose level. d. Give the patient a list of activities that are moderate in intensity.

a. Determine what type of activities the patient enjoys.

The nurse has administered 4 oz of orange juice to an alert patient whose blood glucose was 62 mg/dL. Fifteen minutes later, the blood glucose is 67 mg/dL. Which action should the nurse take next? a. Give the patient 4 to 6 oz more orange juice. b. Administer the PRN glucagon (Glucagon) 1 mg IM. c. Have the patient eat some peanut butter with crackers. d. Notify the health care provider about the hypoglycemia.

a. Give the patient 4 to 6 oz more orange juice.

A 54-year-old patient is admitted with diabetic ketoacidosis. Which admission order should the nurse implement first? a. Infuse 1 liter of normal saline per hour. b. Give sodium bicarbonate 50 mEq IV push. c. Administer regular insulin 10 U by IV push. d. Start a regular insulin infusion at 0.1 units/kg/hr.

a. Infuse 1 liter of normal saline per hour.

A 32-year-old patient with diabetes is starting on intensive insulin therapy. Which type of insulin will the nurse discuss using for mealtime coverage? a. Lispro (Humalog) b. Glargine (Lantus) c. Detemir (Levemir) d. NPH (Humulin N)

a. Lispro (Humalog)

Which nursing action can the nurse delegate to unlicensed assistive personnel (UAP) who are working in the diabetic clinic? a. Measure the ankle-brachial index. b. Check for changes in skin pigmentation. c. Assess for unilateral or bilateral foot drop. d. Ask the patient about symptoms of depression.

a. Measure the ankle-brachial index.

Which patient action indicates a good understanding of the nurse's teaching about the use of an insulin pump? a. The patient programs the pump for an insulin bolus after eating. b. The patient changes the location of the insertion site every week. c. The patient takes the pump off at bedtime and starts it again each morning. d. The patient plans for a diet that is less flexible when using the insulin pump.

a. The patient programs the pump for an insulin bolus after eating.

A female patient is scheduled for an oral glucose tolerance test. Which information from the patient's health history is most important for the nurse to communicate to the health care provider? a. The patient uses oral contraceptives. b. The patient runs several days a week. c. The patient has been pregnant three times. d. The patient has a family history of diabetes.

a. The patient uses oral contraceptives.

An unresponsive patient with type 2 diabetes is brought to the emergency department and diagnosed with hyperosmolar hyperglycemic syndrome (HHS). The nurse will anticipate the need to a. give a bolus of 50% dextrose. b. insert a large-bore IV catheter. c. initiate oxygen by nasal cannula. d. administer glargine (Lantus) insulin.

b. insert a large-bore IV catheter

The nurse is explaining to the parent of a 5-year-old that the child has otitis media with effusion (OME), noted by otoscopic exam, following an upper respiratory infection. Unlike acute otitis media (AOM), OME does not require treatment with antibiotics because it is usually:

asymptomatic and often self-limiting. Explanation: It is very important to distinguish appropriately AOM from OME to ensure the proper treatment and to avoid the unnecessary use of antimicrobials. AOM often presents with sudden onset of ear pain, fever, hearing loss, and middle ear infection, whereas OME often presents as an asymptomatic effusion of the middle ear.

After change-of-shift report, which patient should the nurse assess first? a. 19-year-old with type 1 diabetes who has a hemoglobin A1C of 12% b. 23-year-old with type 1 diabetes who has a blood glucose of 40 mg/dL c. 40-year-old who is pregnant and whose oral glucose tolerance test is 202 mg/dL d. 50-year-old who uses exenatide (Byetta) and is complaining of acute abdominal pain

b. 23-year-old with type 1 diabetes who has a blood glucose of 40 mg/dL

The nurse is preparing to teach a 43-year-old man who is newly diagnosed with type 2 diabetes about home management of the disease. Which action should the nurse take first? a. Ask the patient's family to participate in the diabetes education program. b. Assess the patient's perception of what it means to have diabetes mellitus. c. Demonstrate how to check glucose using capillary blood glucose monitoring. d. Discuss the need for the patient to actively participate in diabetes management.

b. Assess the patient's perception of what it means to have diabetes mellitus.

Which action should the nurse take after a 36-year-old patient treated with intramuscular glucagon for hypoglycemia regains consciousness? a. Assess the patient for symptoms of hyperglycemia. b. Give the patient a snack of peanut butter and crackers. c. Have the patient drink a glass of orange juice or nonfat milk. d. Administer a continuous infusion of 5% dextrose for 24 hours.

b. Give the patient a snack of peant butter and crackers

Which information will the nurse include when teaching a 50-year-old patient who has type 2 diabetes about glyburide (Micronase, DiaBeta, Glynase)? a. Glyburide decreases glucagon secretion from the pancreas. b. Glyburide stimulates insulin production and release from the pancreas. c. Glyburide should be taken even if the morning blood glucose level is low. d. Glyburide should not be used for 48 hours after receiving IV contrast media.

b. Glyburide stimulates insulin production and release from the pancreas.

Which laboratory value reported to the nurse by the unlicensed assistive personnel (UAP) indicates the most urgent need for the nurse's assessment of the patient? a. Bedtime glucose of 140 mg/dL b. Noon blood glucose of 52 mg/dL c. Fasting blood glucose of 130 mg/dL d. 2-hr postprandial glucose of 220 mg/dL

b. Noon blood glucose of 52 mg/dL

The nurse is taking a health history from a 29-year-old pregnant patient at the first prenatal visit. The patient reports no personal history of diabetes but has a parent who is diabetic. Which action will the nurse plan to take first? a. Teach the patient about administering regular insulin. b. Schedule the patient for a fasting blood glucose level. c. Discuss an oral glucose tolerance test for the twenty-fourth week of pregnancy. d. Provide teaching about an increased risk for fetal problems with gestational diabetes.

b. Schedule the patient for a fasting blood glucose level.

Which patient action indicates good understanding of the nurse's teaching about administration of aspart (NovoLog) insulin? a. The patient avoids injecting the insulin into the upper abdominal area. b. The patient cleans the skin with soap and water before insulin administration. c. The patient stores the insulin in the freezer after administering the prescribed dose. d. The patient pushes the plunger down while removing the syringe from the injection site.

b. The patient cleans the skin with soap and water before insulin administration.

79. complete hearing loss a. achromatopsia k. macular degeneration b. anacusis l. meniere disease c. astigmatism m. myringoplasty d. audiometry n. myringotomy e. cataract o. otoscopy f. diabetic retinopathy p. photophobia g. exotropia q. rinne test h. glaucoma r. strabismus i. hordeolum s. tinnitus j. iridoplegia t. vertigo

b. anacusis

A 34-year-old has a new diagnosis of type 2 diabetes. The nurse will discuss the need to schedule a dilated eye exam a. every 2 years. b. as soon as possible. c. when the patient is 39 years old. d. within the first year after diagnosis.

b. as soon as possible

52. eyelid a. Acous/o k. kerat/o b. Blephar/o l. myring/o c. Chromat/o m. ophthalm/o d. Conjunctiv/o n. ot/o e. Choroid/o o. salping/o f. Core/o p. -metry g. Corne/o q. -opsia h. Dacry/o r. -ptosis i. dipl/o s. -rrhea j. irid/o t. -tropia

b. blephar/o

The nurse identifies a need for additional teaching when the patient who is self-monitoring blood glucose a. washes the puncture site using warm water and soap. b. chooses a puncture site in the center of the finger pad. c. hangs the arm down for a minute before puncturing the site. d. says the result of 120 mg indicates good blood sugar control.

b. chooses a puncture site in the center of the finger pad

After the nurse has finished teaching a patient who has a new prescription for exenatide (Byetta), which patient statement indicates that the teaching has been effective? a. "I may feel hungrier than usual when I take this medicine." b. "I will not need to worry about hypoglycemia with the Byetta." c. "I should take my daily aspirin at least an hour before the Byetta." d. "I will take the pill at the same time I eat breakfast in the morning."

c. "I should take my daily aspirin at least an hour before the Byetta."

After change-of-shift report, which patient will the nurse assess first? a. 19-year-old with type 1 diabetes who was admitted with possible dawn phenomenon b. 35-year-old with type 1 diabetes whose most recent blood glucose reading was 230 mg/dL c. 60-year-old with hyperosmolar hyperglycemic syndrome who has poor skin turgor and dry oral mucosa d. 68-year-old with type 2 diabetes who has severe peripheral neuropathy and complains of burning foot pain

c. 60-year-old with hyperosmolar hyperglycemic syndrome who has poor skin turgor and dry oral mucosa

Which statement by a nurse to a patient newly diagnosed with type 2 diabetes is correct? a. Insulin is not used to control blood glucose in patients with type 2 diabetes. b. Complications of type 2 diabetes are less serious than those of type 1 diabetes. c. Changes in diet and exercise may control blood glucose levels in type 2 diabetes. d. Type 2 diabetes is usually diagnosed when the patient is admitted with a hyperglycemic coma.

c. Changes in diet and exercise may control blood glucose levels in type 2 diabetes

The health care provider suspects the Somogyi effect in a 50-year-old patient whose 6:00 AM blood glucose is 230 mg/dL. Which action will the nurse teach the patient to take? a. Avoid snacking at bedtime. b. Increase the rapid-acting insulin dose. c. Check the blood glucose during the night d. Administer a larger dose of long-acting insulin.

c. Check the blood glucose during the night

A few weeks after an 82-year-old with a new diagnosis of type 2 diabetes has been placed on metformin (Glucophage) therapy and taught about appropriate diet and exercise, the home health nurse makes a visit. Which finding by the nurse is most important to discuss with the health care provider? a. Hemoglobin A1C level is 7.9%. b. Last eye exam was 18 months ago. c. Glomerular filtration rate is decreased. d. Patient has questions about the prescribed diet.

c. Glomerular filtration rate is decreased.

A patient who was admitted with diabetic ketoacidosis secondary to a urinary tract infection has been weaned off an insulin drip 30 minutes ago. The patient reports feeling lightheaded and sweaty. Which action should the nurse take first? a. Infuse dextrose 50% by slow IV push. b. Administer 1 mg glucagon subcutaneously. c. Obtain a glucose reading using a finger stick. d. Have the patient drink 4 ounces of orange juice.

c. Obtain a glucose reading using a finger stick.

72. defective curvature of the cornea and lens that results in a distorted image a. achromatopsia k. macular degeneration b. anacusis l. meniere disease c. astigmatism m. myringoplasty d. audiometry n. myringotomy e. cataract o. otoscopy f. diabetic retinopathy p. photophobia g. exotropia q. rinne test h. glaucoma r. strabismus i. hordeolum s. tinnitus j. iridoplegia t. vertigo

c. astigmatism

68. color a. Acous/o k. kerat/o b. Blephar/o l. myring/o c. Chromat/o m. ophthalm/o d. Conjunctiv/o n. ot/o e. Choroid/o o. salping/o f. Core/o p. -metry g. Corne/o q. -opsia h. Dacry/o r. -ptosis i. dipl/o s. -rrhea j. irid/o t. -tropia

c. chromat/o

A 48-year-old male patient screened for diabetes at a clinic has a fasting plasma glucose level of 120 mg/dL (6.7 mmol/L). The nurse will plan to teach the patient about a. self-monitoring of blood glucose. b. using low doses of regular insulin. c. lifestyle changes to lower blood glucose. d. effects of oral hypoglycemic medications.

c. lifestyle changes to lower blood glucose

A 26-year-old female with type 1 diabetes develops a sore throat and runny nose after caring for her sick toddler. The patient calls the clinic for advice about her symptoms and a blood glucose level of 210 mg/dL despite taking her usual glargine (Lantus) and lispro (Humalog) insulin. The nurse advises the patient to a. use only the lispro insulin until the symptoms are resolved. b. limit intake of calories until the glucose is less than 120 mg/dL. c. monitor blood glucose every 4 hours and notify the clinic if it continues to rise. d. decrease intake of carbohydrates until glycosylated hemoglobin is less than 7%.

c. monitor blood glucose every 4 hours and notify the clinic if it continues to rise.

The nurse has been teaching a patient with type 2 diabetes about managing blood glucose levels and taking glipizide (Glucotrol). Which patient statement indicates a need for additional teaching? a. "If I overeat at a meal, I will still take the usual dose of medication." b. "Other medications besides the Glucotrol may affect my blood sugar." c. "When I am ill, I may have to take insulin to control my blood sugar." d. "My diabetes won't cause complications because I don't need insulin."

d. "My diabetes won't cause complications because I don't need insulin."

The anatomy instructor is presenting a lecture on hormone synthesis. The instructor determines that instruction was effective when students identify the site of prohormone to hormone conversion as the:

golgi complex

56. tear; lacrimal apparatus (duct, sac, or gland) a. Acous/o k. kerat/o b. Blephar/o l. myring/o c. Chromat/o m. ophthalm/o d. Conjunctiv/o n. ot/o e. Choroid/o o. salping/o f. Core/o p. -metry g. Corne/o q. -opsia h. Dacry/o r. -ptosis i. dipl/o s. -rrhea j. irid/o t. -tropia

h. dacry/o

77. increased intraocular pressure caused by the failure of aqueous humor to drain, resulting in atrophy of the optic nerve and eventual blindness a. achromatopsia k. macular degeneration b. anacusis l. meniere disease c. astigmatism m. myringoplasty d. audiometry n. myringotomy e. cataract o. otoscopy f. diabetic retinopathy p. photophobia g. exotropia q. rinne test h. glaucoma r. strabismus i. hordeolum s. tinnitus j. iridoplegia t. vertigo

h. glaucoma

When educating a client about glargine, the nurse should explain that this medication:

has a prolonged absorption rate and provides a relatively constant concentration for 12-24 hours.

87. incision of the eardrum to relieve pressure and release pus from the middle ear or to surgically insert tympanostomy tubes in the eardrum a. achromatopsia k. macular degeneration b. anacusis l. meniere disease c. astigmatism m. myringoplasty d. audiometry n. myringotomy e. cataract o. otoscopy f. diabetic retinopathy p. photophobia g. exotropia q. rinne test h. glaucoma r. strabismus i. hordeolum s. tinnitus j. iridoplegia t. vertigo

n. myringotomy

58. ear a. Acous/o k. kerat/o b. Blephar/o l. myring/o c. Chromat/o m. ophthalm/o d. Conjunctiv/o n. ot/o e. Choroid/o o. salping/o f. Core/o p. -metry g. Corne/o q. -opsia h. Dacry/o r. -ptosis i. dipl/o s. -rrhea j. irid/o t. -tropia

n. ot/o

A client tells the health care provider he has noticed a recent change in his vision: he can bring distant images into focus, but near images become blurred. The client has most likely developed:

hyperopia. Explanation: Hyperopia or farsightedness occurs when the accommodative changes of the lens can bring distant images into focus, but near images become blurred. Persons with myopia or nearsightedness can see close objects without a problem because accommodative changes in their lens bring near objects into focus, but distant objects are blurred. Astigmatism is a refractive defect of the corneal surface.

A client presents with a huge lower jaw, bulging forehead, large hands and feet, and frequent headaches. What could be causing this client's symptoms?

hyperpituitarism

76. small, purulent, inflammatory infection of a sebaceous gland of the eyelid; also called sty a. achromatopsia k. macular degeneration b. anacusis l. meniere disease c. astigmatism m. myringoplasty d. audiometry n. myringotomy e. cataract o. otoscopy f. diabetic retinopathy p. photophobia g. exotropia q. rinne test h. glaucoma r. strabismus i. hordeolum s. tinnitus j. iridoplegia t. vertigo

i. hordeolum

The ophthalmologist is performing a client's annual eye exam and notes an increase in intraocular pressure. The condition most likely is the result of:

imbalance between aqueous production and outflow. Explanation: Increases in intraocular pressure occur when there is an imbalance between aqueous production and outflow. A cell breakdown in the macula lutea results in macular degeneration. Cataracts may cause blurring of vision but do not alter pressure.

83. loss of central vision and a common cause of visual impairment in persons over age 50 a. achromatopsia k. macular degeneration b. anacusis l. meniere disease c. astigmatism m. myringoplasty d. audiometry n. myringotomy e. cataract o. otoscopy f. diabetic retinopathy p. photophobia g. exotropia q. rinne test h. glaucoma r. strabismus i. hordeolum s. tinnitus j. iridoplegia t. vertigo

k. macular degeneration

80. rare disorder of unknown etiology within the labyrinth of the inner ear that can be lead to progressive hearing loss a. achromatopsia k. macular degeneration b. anacusis l. meniere disease c. astigmatism m. myringoplasty d. audiometry n. myringotomy e. cataract o. otoscopy f. diabetic retinopathy p. photophobia g. exotropia q. rinne test h. glaucoma r. strabismus i. hordeolum s. tinnitus j. iridoplegia t. vertigo

l. meniere disease

69. tympanic membrane (eardrum) a. Acous/o k. kerat/o b. Blephar/o l. myring/o c. Chromat/o m. ophthalm/o d. Conjunctiv/o n. ot/o e. Choroid/o o. salping/o f. Core/o p. -metry g. Corne/o q. -opsia h. Dacry/o r. -ptosis i. dipl/o s. -rrhea j. irid/o t. -tropia

l. myring/o

A middle-aged female client has been diagnosed with thyroid condition. The nurse educates the client about the prescription and needed follow-up lab work, which will help regulate the dosage. The client asks, "Why do I not return to the clinic for weeks, since I am starting the medication tomorrow morning?" The nurse bases the answer on the knowledge that thyroid hormones:

may take days for the full effect to occur, based on the mechanism of action. Explanation: Hormones produce their effects through interaction with high-affinity receptors, which in turn are linked to one or more effector systems within the cell. These mechanisms involve many of the cell's metabolic activities, ranging from ion transport at the cell surface to stimulation of nuclear transcription of complex molecules. The rate at which hormones react depends on their mechanism of action. Thyroid hormone, which controls cell metabolism and synthesis of intracellular signaling molecules, requires days for its full effect to occur. None of the other distractors are accurate reasons to have the client return to the clinic weeks after starting the medication.

Which of the following best describes the half-life of a highly protein-bound drug such as thyroxine (99% protein bound)? The half-life would be:

much longer to reduce the concentration of the hormone by one half.

An incoherent client with a history of hypothyroidism is brought to the emergency department by the rescue squad. Physical and laboratory findings reveal hypothermia, hypoventilation, respiratory acidosis, bradycardia, hypotension, and nonpitting edema of the face and periorbital area. Knowing that these findings suggest severe hypothyroidism, the nurse prepares to take emergency action to prevent the potential complication of:

myxedema coma.

62. tube (usually fallopian or eustachian (auditory) tubes) a. Acous/o k. kerat/o b. Blephar/o l. myring/o c. Chromat/o m. ophthalm/o d. Conjunctiv/o n. ot/o e. Choroid/o o. salping/o f. Core/o p. -metry g. Corne/o q. -opsia h. Dacry/o r. -ptosis i. dipl/o s. -rrhea j. irid/o t. -tropia

o. salping/o

The nurse is performing her morning assessment on a 69 year old patient on a medical-surgical unit. Upon assessing her pupils the nurse notices that they are equal and responsive to light but not to accommodation.The nurse is aware that with aging the lens becomes thicker and its capsule less elastic so she believes this to be the case. When accommodation is diminished in an older person as a result of aging this is referred to as what?

presbyopia

The nurse is performing her morning assessment on a 69 year old patient on a medical-surgical unit. Upon assessing her pupils the nurse notices that they are equal and responsive to light but not to accommodation.The nurse is aware that with aging the lens becomes thicker and its capsule less elastic so she believes this to be the case. When accommodation is diminished in an older person as a result of aging this is referred to as what?

presbyopia.

The nurse is performing her morning assessment on a 69-year-old client on a medical-surgical unit. Upon assessing her pupils the nurse notices that they are equal and responsive to light but not to accommodation. The nurse is aware that with aging the lens becomes thicker and its capsule less elastic so she believes this to be the case. When accommodation is diminished in an older person as a result of aging this is referred to as:

presbyopia. Explanation: The term presbyopia refers to a decrease in accommodation that occurs because of aging. Hyperopia is the same as farsightedness and myopia refers to nearsightedness (when the person can see close objects without problems but distant objects are blurred).

84. hearing acuity test that is performed with a vibrating tuning fork placed on the mastoid process and then in front of the external auditory canal and used to test bone and air conduction a. achromatopsia k. macular degeneration b. anacusis l. meniere disease c. astigmatism m. myringoplasty d. audiometry n. myringotomy e. cataract o. otoscopy f. diabetic retinopathy p. photophobia g. exotropia q. rinne test h. glaucoma r. strabismus i. hordeolum s. tinnitus j. iridoplegia t. vertigo

q. rinne test

75. muscular eye disorder in which the eyes turn from the normal position so that they deviate in different directions a. achromatopsia k. macular degeneration b. anacusis l. meniere disease c. astigmatism m. myringoplasty d. audiometry n. myringotomy e. cataract o. otoscopy f. diabetic retinopathy p. photophobia g. exotropia q. rinne test h. glaucoma r. strabismus i. hordeolum s. tinnitus j. iridoplegia t. vertigo

r. strabismus

During a near-miss accident while cycling, a client marvels at how fast he was able to react. He attributes this to his "fight or flight" response but then wonders why it lasts for only a short period. The client had a short burst of catecholamine activity because catecholamines are:

rapidly degraded by enzymes in circulation and at the tissue.

When comparing the endocrine and nervous system functions, the nurse knows that the endocrine system:

releases hormones into the blood that is transported throughout the body. glands are widely scattered throughout the body. takes longer to respond to innervations but has prolonged actions when they arrive.

An elderly woman comes to the clinic complaining of seeing flashing lights and small spots. She tells the nurse that this has been going on for over 24 hours but now it is as if she sees a dark curtain whenever she opens her eyes. She asks the nurse if this means she is going to be blind.What diagnosis should the nurse suspect ?

retinal detachment

An elderly woman comes to the clinic complaining of seeing flashing lights and small spots. She tells the nurse that this has been going on for over 24 hours but now it is as if she sees a dark curtain whenever she opens her eyes. She asks the nurse if this means she is going to be blind. What diagnosis should the nurse suspect ? a) conjunctivitis b) retinal detachment c) cataracts d) glaucoma

retinal detachment The primary symptom of retinal detachment consists of painless changes in vision. Commonly, flashing lights or sparks, followed by small floaters or spots in the filed of vision occur. As the detachment progresses the person perceives a shadow or dark curtain across the visual field.

An elderly woman comes to the clinic complaining of seeing flashing lights and small spots. She tells the nurse that this has been going on for over 24 hours but now it is as if she sees a dark curtain whenever she opens her eyes. She asks the nurse if this means she is going to be blind.What diagnosis should the nurse suspect ?

retinal detachment.

While reviewing the major actions of follicle-stimulating hormone (FSH), the faculty points out that in males, this hormone is responsible for the:

sperm production. Explanation: FSH in males stimulates sperm production. GH stimulates growth of bone and muscle. LH stimulates secretions of testosterone. Androgens affect the development of male sex organs. Page 756

A mother brings her toddler to the clinic and tells the nurse that she thinks something is wrong with the baby's eyes. Upon further assessment the nurse notices that the child has upper deviation in movement in only one eye. What should the nurse suspect is wrong with the child? a) cataract b) glaucoma c) amblyopia d) strabismus

strabismus Strabismus or squint, refers to any abnormality of eye coordination or alignment that results in loss of binocular vision. -Amblyopia refers to lazy eye. -Glaucoma is a chronic degenerative optic neuropathy and - cataract affects the lens of the eye.

A nurse is instructing a client with newly diagnosed hypoparathyroidism about the regimen used to treat this disorder. The nurse should state that the physician probably will order daily supplements of calcium and:

vitamin D.

A client is scheduled for a suppression test as part of the diagnostic testing for his suspected endocrine disorder. The results of this test will help the care team determine:

whether the client is producing excessive hormone levels.

A client has been diagnosed with bacterial conjunctivitis. This type of infection is usually characterized by large amounts of what color drainage?

yellow-green Explanation: The infection usually is characterized by large amounts of yellow-green drainage. The eyelids are sticky, and there may be excoriation of the lid margins.

A nurse on a medical unit is providing care for a 37-year-old female patient who has a diagnosis of Graves' disease. Which of the following treatments would the nurse most likely anticipate providing for the client? a) Administration of somatostatin analogs to inhibit GH production b) Administration of levothyroxine to supplement thyroid function c) Calcium channel blocking medications to reduce heart rate and cardiac risks d) β-adrenergic-blocking medications to reduce sympathetic nervous stimulation

β-adrenergic-blocking medications to reduce sympathetic nervous stimulation The hyperthyroidism that constitutes Graves' disease can often be mitigated by the administration of β-adrenergic-blocking medications. Levothyroxine would be used to address hypothyroidism and calcium channel blockers are not an identified treatment modality for Graves' disease. Somatostatin analogs are used to treat GH excess

A patient with hyperthyroidism took aspirin for a headache. Which of the following complications could develop? Select all that apply. a) Tachycardia b) Edema c) Hyperlipidemia d) Diarrhea e) Hyperthermia

• Hyperthermia • Tachycardia • Diarrhea Thyroxine has a normal half-life of approximately 6 days because it is almost completely protein bound in the blood. Aspirin competes with thyroxine for the protein carrier causing the released thyroxine to exert its effect all at once. The patient could develop a thyroid crisis, called a thyroid storm, with signs and symptoms of tachycardia, hyperthermia, heart failure, shock, diarrhea, vomiting, delirium, and coma. Hyperlipidemia and edema could occur as a result of hypothyroidism.

Which of the following clients are at risk for developing and acute episode of angle-closure glaucoma? Select all that apply.

• A 60-year-old female emotionally devastated after divorcing her husband of 35 years • A soldier ordered to stay in a pitch-black cave to observe enemy militants for an extended period of time • A surgical client who has received many doses of IV atropine to keep heart rate above 50 Manifestations of acute angle-closure glaucoma are related to sudden, intermittent increases in intraocular pressure. These occur after prolonged periods in the dark, emotional upset, and other conditions that cause extensive and prolonged dilation of the pupil. Administration of pharmacologic agents, such as atropine, that cause pupillary dilation (mydriasis) also can precipitate an acute episode. -Age-related macular degeneration (AMD) is the most common cause of reduced vision in the elderly. In addition to older age, identifiable risk factors include cigarette smoking, obesity, and low dietary intake of lutein, omega 3 fatty acids, zinc, and vitamins A, C, and E. Persons with high grades of myopia or nearsightedness may have abnormalities in the peripheral retina that predispose to sudden detachment.

A client has been diagnosed with dysfunction of the anterior pituitary gland. The nurse is aware that which of the following hormones may be affected? Select all that apply. a) Growth hormone (GH) b) Antidiuretic hormone (ADH) c) Luteinizing hormone (LH) d) Norepinephrine e) Adrenocorticotropic hormone (ACTH) f) Thyroid-stimulating hormone (TSH)

• Adrenocorticotropic hormone (ACTH) • Thyroid-stimulating hormone (TSH) • Growth hormone (GH) • Luteinizing hormone (LH) The anterior lobe of the pituitary gland produces adrenocorticotropic hormone (ACTH), thyroid-stimulating hormone (TSH), growth hormone (GH), the gonadotropic hormones (follicle-stimulating hormone [FSH] and luteinizing hormone [LH]), and prolactin. ADH is produced in the posterior pituitary, and norepinephrine is produced in the adrenal medulla.

A patient with diabetes has been injured. The nurse will tell the patient that healing of the injury may be delayed due to which of the following diabetic complications? Select all that apply. a) Fluid imbalances b) Ketoacidosis c) Somogyi effect d) Vascular impairment e) Chronic neuropathies

• Chronic neuropathies • Vascular impairment Suboptimal response to injury and infection in a person with diabetes is caused by chronic complications, including vascular disease and neuropathies, poorly controlled hyperglycemia, and altered immune cell and neutrophil function. Sensory loss decreases awareness of minor trauma and infection, and vascular disease impairs circulation of blood with substances needed for effective healing. The Somogyi effect is an acute complication of diabetes, causing hypoglycemia. Ketoacidosis is an acute complication of hyperglycemia when liver ketone production exceeds cell use. (less)

Type 2 diabetes is caused by metabolic abnormalities in the presence of insulin. What are these metabolic abnormalities? (Select all that apply.) a) Hypersensitivity to insulin b) Deranged secretion of insulin c) Increased glucose production by the liver d) Insulin resistance e) Decreased glucose production by the liver

• Deranged secretion of insulin • Insulin resistance • Increased glucose production by the liver The metabolic abnormalities that lead to type 2 diabetes include (1) insulin resistance, (2) deranged secretion of insulin by the pancreatic beta cells, and (3) increased glucose production by the liver. The other answers are incorrect.

Acute otitis media (AOM) is the disorder in children for which antibiotics are most prescribed. What are the risk factors for AOM? Select all that apply.

• Ethnicity • Premature birth • Genetic syndromes

A 30 year-old man with a diagnosis of type 1 diabetes is aware of the multiple effects that insulin has on his metabolism. Which of the following physiological processes are actions of insulin? Select all that apply. a) Promoting the breakdown of stored triglycerides b) Increasing the metabolic needs of body cells c) Promoting glucose uptake by target cells. d) Inhibiting protein breakdown e) Facilitating triglyceride synthesis from glucose in fat cells

• Facilitating triglyceride synthesis from glucose in fat cells • Inhibiting protein breakdown • Promoting glucose uptake by target cells. The actions of insulin are threefold: (1) it promotes glucose uptake by target cells and provides for glucose storage as glycogen, (2) it prevents fat and glycogen breakdown, and (3) it inhibits gluconeogenesis and increases protein synthesis. Insulin acts to promote fat storage by increasing the transport of glucose into fat cells. It also facilitates triglyceride synthesis from glucose in fat cells and inhibits the intracellular breakdown of stored triglycerides. Insulin also inhibits protein breakdown. It does not directly influence the metabolic needs of body cells.

A nurse is teaching a patient with newly diagnosed type 1 diabetes about the importance of blood glucose control to decrease the risk of which of the following potential chronic complications of diabetes? Select all that apply. a) Nephropathy b) Gastroparesis c) Neuropathy d) Anxiety disorder e) Retinopathy

• Gastroparesis • Nephropathy • Retinopathy • Neuropathy There are many complications that result from consistent hyperglycemia. Damage to the retina leads to loss of vision. Damage to kidneys leads to renal failure. Neuropathies have many serious consequences including loss of feeling in the lower extremities, foot ulcers, and infections that go undetected and may result in amputation. Impaired gastrointestinal function includes the slowed gastric emptying of gastroparesis. Anxiety, although it is a possible consequence of a diagnosis of diabetes, is not directly attributable to elevated blood glucose levels.

A patient hospitalized with type 1 diabetes has been administered a scheduled dose of regular insulin. Which of the following are processes of insulin? Select all that apply. a) Glucose uptake by muscle and adipose tissue b) Promotion of glycogenolysis c) Gluconeogenesis d) Protein synthesis e) Fat storage

• Glucose uptake by muscle and adipose tissue • Protein synthesis • Fat storage There are three actions of insulin: (1) it promotes glucose uptake by target cells and provides for glucose storage as glycogen; (2) it prevents fat and glycogen breakdown; and (3) it inhibits gluconeogenesis and increases protein synthesis. Glucagon, not insulin, promotes glycogenolysis.

A nurse is conducting a "Living Well with Diabetes" seminar for the community outreach center. The nurse is discussing diabetic retinopathy and its overwhelming effect on people living with diabetes. Which of the following modifiable risk factors would the nurse discuss? Select all that apply.

• Hyperglycemia • Hypercholesteremia • Hypertension

An infant born with congenital hypothyroidism who does not receive care from any healthcare provider is likely to develop which of the following complications? Select all that apply. a) Loss of fine motor control and arthritis b) Deformed joints and bone spurs c) Impaired physical growth d) Down's syndrome e) Mental retardation

• Impaired physical growth • Mental retardation Thyroid hormone is essential for normal growth and brain development, almost half of which occurs during the first 6 months of life. If untreated, congenital hypothyroidism causes mental retardation and impairs physical growth. Down's syndrome is a congenital birth defect and not caused by hypothyroidism.

Stepping out of a mall and into the sunshine has caused a man's pupils to constrict. Place the following anatomical components of the man's pupillary reflex in the ascending chronological order that they responded to the light. Use all the options. a) Retinal ganglionic cells b) Oculomotor nuclei c) Pretectal nuclei d) Preganglionic neurons

• Retinal ganglionic cells • Oculomotor nuclei • Pretectal nuclei • Preganglionic neurons Pretectal areas on each side of the brain are connected, explaining the binocular aspect of the light reflex. The afferent stimuli for pupillary constriction arise in the ganglionic cells of the retina and are transmitted to the pretectal nuclei at the junction of the thalamus and the midbrain, and from there to preganglionic neurons in the oculomotor (CN III) nuclei via the pretecto-oculomotor tract

Stepping out of a mall and into the sunshine has caused a man's pupils to constrict. Place the following anatomical components of the man's pupillary reflex in the ascending chronological order that they responded to the light. Use all the options.

• Retinal ganglionic cells • Preganglionic neurons • Oculomotor nuclei • Pretectal nuclei

What are the symptoms of Ménière disease? Select all that apply.

• Severe ringing in the ear(s) • Feeling of fullness in the ear(s) • Episodes of disabling vertigo

The nurse is a conducting discharge teaching to a patient prescribed an ototoxic medication. Which cochlear signs and/or symptoms will the nurse ask the client to report? Select all that apply.

• Tinnitus • Hearing loss


Ensembles d'études connexes

Explore the value of Google Search

View Set

AP Human Geography Unit 6 Vocabulary

View Set

2. ¿Qué es ...? (What is ...?)

View Set

Fundamental of Nursing Chapter 6: Values, Ethics, and Advocacy

View Set

Ethical Hacking and Network Defense - Chapter: 5

View Set

National Code of Ethics for Interpreters in Health Care, Interpreter Module 3: Strategic Mediation, Interpreter training, Community Interpreting Training, Interpreter Training Mid-term, Interpreter Review Module 2: Interpreter Protocols and Skills, C...

View Set